Você está na página 1de 136

Current Affairs

October 2015

Current Affairs 1 October-2015

01 October 2015
Paper 2 Topic: Bilateral, regional and global
groupings and agreements involving India
and/or affecting Indias interests
India to hold G20 Chair in 2018, Delhi may play
host
India is set to be the G20 Chair in 2018, and New
Delhi could host the prestigious annual G20 summit.
This decision was taken by G20 member
nations recently.
G20:
The Group of Twenty is an international forum for
the governments and central bank governors from 20
major economies. The members include 19
individual countries and the European Union (EU).
The G20 started in 1999 as a meeting of
Finance Ministers and Central Bank
Governors in the aftermath of the Asian
financial crisis.
Collectively, the G-20 economies account for
around 85% of the gross world product
(GWP), 80% of world trade, and two-thirds
of the world population.
The G20 is supported by international
organisations, including the Financial
Stability Board, the International Labour
Organisation, the International Monetary
Fund, the Organisation for Economic Cooperation and Development, the United
Nations, the World Bank and the World Trade
Organization.
Notable points:
The G20 operates as a forum and not as an
organisation. Therefore, it does not have any
permanent secretariat or management and
administrative structure.
One of the G20 countries is selected to hold
the Chair in rotation, also known as G20
presidency. The presidency establishes a
temporary secretariat for the duration it holds
the Chair. The secretariat coordinates all
work and organises G20 meetings.
The immediate past, present and next Chair
constitute a troika and ensure continuity in
the G20 work. In the current year (2015), the
presidency is held by Turkey. The Chair was
handed over to it by Australia. Turkey will
hand over the Chair for the next year (2016)
to China. After China, Germany will hold it
in 2017.
For selecting presidency, a system has been
in place since 2010, when South Korea held

the Chair. Under it, 19 countries have been


categorised into five regional groupings of a
maximum of four nations each.
Sources: The Hindu, g20.
Paper 2 Topic: Government policies and
interventions for development in various
sectors and issues arising out of their
design and implementation.

Natural gas price cut in line with new


policy
The government has cut the price of natural gas by
18% to $3.81 per million British thermal unit
(mmBtu) on a gross calorific value basis from the
current $4.66 per mmBtu.
The new price will be in place for six
months. This is in line with the policy
adopted by the government in October 2014.
This price cut is likely to help fertiliser and
power firms.
In accordance with the New Domestic Natural Gas
Pricing Guidelines, 2014, issued by the Ministry of
Petroleum and Natural Gas, the price of natural gas
is to be revised every six months on the basis of a
weighted average of rates in countries such as the
U.S., Mexico, Canada and Russia, all gas-surplus
economies.
Salient features of the Domestic Natural Gas Pricing
Guidelines, 2014:
These guidelines will be applicable to all
natural gas produced domestically,
irrespective of the source, whether
conventional, shale, CBM etc.
The prices determined under these guidelines
will be applicable to all consuming sectors
uniformly.
These guidelines will also be applicable for
natural gas produced by ONGC/OIL from
their nominated fields.
Natural Gas:
It is by product of crude oil and is considered
to be a clean fuel.
India imports 25% of its natural gas
requirement. India has significant natural gas
reserves in offshore block.
Sources: The Hindu, PIB.
Paper 2 Topic: Inclusive growth and
issues arising from it.

Maharashtra imposes tax to tackle


drought
In a bid to raise funds to tackle the drought situation,

Current Affairs 2 October-2015

the Maharashtra government has decided to impose


drought tax.
This is the first time since 1973 that a state
government has decided to take such a
drastic step.
This tax is meant to help farmers who have
been hit by one of the worst droughts in
recent times.
This arrangement will be in place for the next
five months and will be reviewed during the
next budget session.
The State government is expected to raise Rs.
1600 crore through this.
Drought tax includes:
Tax on petrol and diesel, VAT on liquor,
cigarettes and beverages, and surcharge on
VAT for gold and diamond jewelleries.
A tax of Rs. 2 per litre would be charged on
petrol and diesel, while Value Added Tax
(VAT) on liquor, cigarettes and beverages has
been raised by 5%. Also, the surcharge on
VAT for gold and diamond jewelleries has
been raised from 1 to 1.20 %.
The State government will also hold a special
Cabinet meeting next month to discuss other means
to increase the inflow of money and cut expenditure.
According to the data provided by the government, it
has spent around Rs. 3,332.57 crore on various
schemes and packages meant to support drought-hit
farmers.
Sources: The Hindu.
Paper 2 Topic: Bilateral, regional and
global groupings and agreements
involving India and/or affecting Indias
interests.

India climbs 16 places in WEFs Global


Competitiveness Index
After five years of decline, India has moved up 16
positions to 55th place on a global index of worlds
most competitive economies for the year 2015-16.
Global Competitiveness Report is released
annually by the World Economic Forum
(WEF).
Performance of various countries:
Globally, Switzerland has retained its top
position as the worlds most competitive
economy for seventh year in a row and is
followed by Singapore, the US, Germany and
the Netherlands in the top-five.
These are followed by Japan, Hong Kong,
Finland, Sweden and the UK in the top-ten.

South Africa has re-entered the top 50,


progressing seven places to 49th.
China, holding steady at 28, remains by far
the most competitive among large emerging
economies, although its lack of progress
moving up the ranking shows the challenges
it faces in transitioning its economy.
Performance of India:
In the last years report, India stood at the
71st position.
In terms of competitiveness of its institutions,
India is ranked 60th (out of total 140
countries and up 10 positions from last year),
while for infrastructure it has gained six
places to 81st.
For macroeconomic environment, India is
ranked 91st, helped by a reduction in
commodity prices and improvement in the
governments budget deficit.
Observations made by the WEF:
It says, the most problematic factors for
doing business in India include corruption,
policy instability, inflation and access to
finance.
The areas where India ranks better were
investor protection, gross national savings,
quality of education system, venture capital
availability, hiring and firing practices, GDP
and domestic market size, public trust in
politicians and burden of government
regulation.
Even though infrastructure in India has
improved (81st, up six places), it remains a
major growth bottleneck, electricity in
particular.
Indias performance in the macroeconomic
stability pillar has improved, although the
situation remains worrisome (91st, up 10
places). The inflation also eased to 6% in
2014, due to lower commodity prices, down
from near double-digit levels the previous
year.
The government budget deficit has gradually
dropped since its 2008 peak, although it still
amounted to 7% of GDP in 2014, one of the
highest in the world (131st).
Fewer than one in five Indians access the
Internet on a regular basis, and fewer than
two in five are estimated to own even a basic
cell phone.
Sources: The hindu, toi.

Current Affairs 3 October-2015

Trivia:
For the first time in the history of Indian
judiciary, the 150-year-old Madras High
Court recently allowed live telecast of
contempt proceeding against two Madurai
advocates.
Meghalaya Governor V. Shanmuganathan
was recently sworn in as the 17th Governor
of Manipur.
1. Question

With reference to Business Correspondent


Agents/Bank Mitras under the Pradhan Mantri
Jan Dhan Yojana, which of the following
statements are correct?
1. They offer banking services like cash
deposit ,withdrawal and fund transfer
2. They also offer the service of opening
new accounts
3. Providing insurance and pension related
services too is one of their tasks
4. The Business Correspondent Agents/Bank
Mitras are also playing a major role in
spread of financial literacy
Select the correct answer using the codes given
below.
a) 1, 3 and 4 only
b) 1,2 and 4 only
c) All of the above
d) 1 and 4 only
Incorrect
Ans: c.
http://pib.nic.in/newsite/PrintRelease.aspx?
relid=127071
2. Question
Consider the following statements with regard to
Subhas Chandra Bose:
1. He secured from the Axis powers, a
declaration in favour of Indian
Independence
2. The Provisional Government of Free
India established by him was recognized
by Japan as well as Germany and Fascist
Italy
Which of the above statements are correct?
a) 1 only
b) 2 only
c) Both 1 and 2 only
d) Neither 1 or 2
Ans: b.
Although he could not secure an Axis declaration in
favour of Indian Independence, his Provisional
Government of Free India was recognized by many
Axis-nations.

Page 425-426, From Plassey to Partition Sekhar


Bandopadhyay
3. Question
It is often said that Subhas Chandra Bose died in
an air crash. What was his planned-destination?
a) Soviet Russia
b) Manchuria
c) He was performing a routine sortie
which resulted in the fatal crash
d) Germany
Ans: a.
The failed Imphal military campaign did not deter
Boses spirit. He remained optimistic, thought of
regrouping, and after Japanese surrendered,
contemplated seeking help from Soviet Russia. The
Japanese agreed to provide him transport up to
Manchuria from where he could travel to Russia.
But on his way, he died in an air crash.
So his final destination turned out to be the heavens,
but his planned-destination was Soviet Russia J
Page 426-427, From Plassey to Partition Sekhar
Bandopadhyay
4. Question
The International Migration Outlook is an
annual report released by the
a) World Bank
b) United Nations Economic Commission
for Europe (UNECE)
c) Population Division of the Department
of Economic and Social Affairs under the
United Nations
d) Organisation for Economic
Cooperation and Development
Ans: d.
http://www.thehindu.com/data/emigration-fromindia-rises-but-remains-relativelylow/article7677645.ece
5. Question
Which of the following statements about the
Organisation for Economic Cooperation and
Development are incorrect?
1. India is a key partner but not a member
2. Unlike the G20, it has a permanent
secretariat
Select the correct answer using the codes given
below:
a) 1 Only
b) 2 Only
c) Both 1 and 2
d) Neither 1 or 2
Ans: d.
http://www.oecd.org/about/whodoeswhat/

Current Affairs 4 October-2015


http://www.thehindu.com/data/emigration-fromindia-rises-but-remains-relativelylow/article7677645.ece

02 October 2015
Paper 2 Topic: Welfare schemes for
vulnerable sections of the population by the
Centre and States and the performance of these
schemes.

Mid-day meals rules lay stress on quality,


regularity
The Centre has notified the mid-day meals rules.
The rules were notified by the union human resource
development ministry in accordance with the
provisions of the National Food Security Act 2013
act after consultation with states and other related
central ministries.
Notified rules:
Mid-day meals provided to students at
schools will be tested each month by
accredited laboratories to ensure nutritional
standards and quality.
The meal provided to children will be
evaluated and certified by the government
food research laboratories or any laboratory
accredited or recognised by law.
The rules make it mandatory for schools to
dig into funds meant for other purposes if the
funds meant for providing mid-day meals to
students are exhausted for some reason.
The schools are required to provide an
allowance to students if meals cannot be
given them.
Concerned state governments are required to
fix responsibility on the person or agency if
meals are not provided on three consecutive
school days or five days in a month.
Every child within the age group of six to 14
years studying in Classes 1 to 8, enrolled
with and attending school, shall be provided
hot-cooked meal having nutritional standards
of 450 calories and 12 gm of protein for
primary and 700 calories and 20 gm protein
for upper primary classes free of charge
every day except on school holidays.
The mid-day meals shall be served to the
children only in the school.
The schools will also have to ensure meals
are cooked in a hygienic manner. Schools in
urban area may use the facility of centralised
kitchens wherever required in accordance

with the guidelines issued by the central


government.
The heads of schools are empowered to
utilise any available fund for the continuation
of the scheme. The amount will be
reimbursed to the school account.
Background:
The National Food Security Act, 2013,
contains provisions related to welfare
schemes, including the mid-day meal
scheme.
The Midday Meal Scheme was designed to
improve the nutritional status of school
children nationwide.
The programme supplies free lunches on
working days for children in primary and
upper primary classes in government,
government aided, local body, Education
Guarantee Scheme, and alternative
innovative education centres, Madrasas and
Maqtabs supported under Sarva Shiksha
Abhiyan, and National Child Labour Project
schools run by the ministry of labour.
Sources: The Hindu, Wiki.
Paper 2 Topic: Urbanisation problems
and remedies.

Swachh Bharat: plan to produce power,


compost from solid waste
On the first anniversary of Swachh Bharat Mission,
the Urban Development Ministry has announced
some crucial policy changes since it is planning to
generate electricity and compost from municipal
solid waste.
Ministry of Chemicals and Fertilizers has
decided to bring a proposal before Cabinet to
provide Market Development Assistance on
sale of city compost to farmers.
Ministry of Power is in the process of
amending the Electricity Act 2003 to enable
mandatory purchase of power generated from
municipal solid waste.
About Swachh Bharat Mission:
It was officially launched on 2 October 2014 and is
Indias biggest ever cleanliness drive. The mission
seeks to achieve clean India and aims to provide
access to toilets to all households in the country.
Objectives of the mission:
Eliminate open defecation.
Conversion of insanitary toilets to pour flush
toilets.
Eradication of manual scavenging.

Current Affairs 5 October-2015

100% collection and scientific


processing/disposal reuse/recycle of
Municipal Solid Waste.
To bring about a behavioral change in people
regarding healthy sanitation practices.
Generate awareness among the citizens about
sanitation and its linkages with public health.
Strengthening of urban local bodies to
design, execute and operate systems.
To create enabling environment for private
sector participation in Capital Expenditure
and Operation & Maintenance (O&M) costs.
The components of the programme are:
Construction of individual sanitary latrines
for households below the poverty line with
subsidy (80%) where demand exists.
Conversion of dry latrines into low-cost
sanitary latrines.
Construction of exclusive village sanitary
complexes for women providing facilities for
hand pumping, bathing, sanitation and
washing on a selective basis where there is
not adequate land or space within houses and
where village panchayats are willing to
maintain the facilities.
Setting up of sanitary marts.
Total sanitation of villages through the
construction of drains, soakage pits, solid and
liquid waste disposal.
Intensive campaign for awareness generation
and health education to create a felt need for
personal, household and environmental
sanitation facilities.
Sources: the hindu, wiki.
Paper 3 Topic: Indians achievement in
S&T.

NHAI to sign pact with Isro for satellite


mapping of highways
The National Highways Authority of India (NHAI)
is planning to sign a memorandum of understanding
(MoU) with the Indian Space Research Organisation
(Isro) for adoption of best technologies in the road
sector.
It is also planning to utilise GAGAN and
BHUVAN satellite systems to prepare a
360-degree mapping of all national highways
by 2017 under its road assets management
system (RAMS).
This will facilitate timely repair of roads,
maintenance, monitoring of road progress,
formulation of detailed project reports

(DPR), etc.
It is also mulling signing a pact with the
North East Centre for Technology
Application and Reach (NECTAR) for
deployment of drones and aerial vehicles in
difficult terrains to map the roads under
RAMS.
What is RAMS?
Road Asset Management System (RAMS) is a
strategic and systematic process of maintaining,
upgrading, and operating road assets effectively.
Information collected and made available through
this system will be useful for the agencies
responsible for road development and maintenance,
investors, as well as road users.
The development of a Road Asset
Management System (RAMS) for the entire
National Highways along with collection of
requisite data is part of the Digital India
initiative.
It is a World Bank funded project which
helps in creation of centralised road database
for all NHs in the country.
Initially, data collection for 3,000 km length
of the pilot NH network shall be included in
RAMS. In subsequent stages; it is proposed
to collect data for all NHs in India.
Project Objectives:
Development of single road database for
NHs supported by analytical tools to be used
by NHAI and MORTH
Institutionalise RAMS in NHAI and ministry
of road transport and highways (MORTH) to
assist in planning, programming and
budgeting for road maintenance and
upgrading works
Integrate GIS within RAMS to form publicly
accessible Traveller/Tourist Information
System
Information collected from this project will
be useful for the transport ministry, finance
ministry, NHAI, and funding agencies.
Indian Highways network is the worlds second
largest road network of 4.87 million km. The total
length of National Highways in India is over 97,135
km as on March 2015. National Highways account
for only 2% of total road network, but carries 40%
of the traffic.
About GAGAN:
GAGAN was developed by the Indian Space
Research Organisation (ISRO) and the
Airports Authority of India (AAI) at a cost of
Rs. 774 crore, over 15 years.

Current Affairs 6 October-2015

GAGAN will provide augmentation service


for the GPS over the country, the Bay of
Bengal, South East Asia and Middle East and
up to Africa.
Some of its benefits are improved efficiency,
direct routes, increased fuel savings,
approach with vertical guidance at runways,
significant cost savings because of the
withdrawal of ground aids and reduced
workload of flight crew and air traffic
controllers.
Gagan works by augmenting and relaying
data from GPS satellites with the help of two
augmentation satellites and 15 earth-based
reference stations.
The system utilises the satellite-based wide
area augmentation system (SBAS)
technology which has been developed by
Raytheon.
Sources: the hindu, pib.
1. Question

Consider the following statements about the


Integrated Child Development Scheme:
1. One of its objectives is to improve the
nutritional and health status of children in
the age-group 0-5 years
2. Private players cannot participate in
providing services under this scheme
Which of the above statements are correct?
a) 1 only
b) 2 only
c) Both 1 and 2 only
d) Neither 1 or 2
Ans: b.
One of its objectives is to improve the nutritional
and health status of children in the age-group 0-6
years.
http://www.thehindu.com/news/national/contractorraj-on-in-nutrition-schemes/article7678350.ece

http://wcd.nic.in/icds.htm
2. Question
Consider the following statements:
Assertion (A): A big solar event (Coronal Mass
Ejections) may cause Low-Earth-Orbit satellites
to be dragged back down to earth
Reasons (R): A big solar event causes the Earths
atmosphere to swell up
Which of the above statements is/are true?
a) A and R both are true, and R is the
correct explanation for A
b) A and R both are true, and R is the
NOT the correct explanation for A

c) A is correct, R is incorrect
d) A and R both are incorrect
Ans: a.
A pretty good article on a pretty cool phenomenon
http://www.bbc.com/earth/story/20150923-the-suncreates-space-weather-that-affects-us-all
3. Question
Consider the following statements:
1. It is on average an 11-year cycle
2. Its extremes consist of a solar minimum
and a solar maximum
3. Extreme solar events can happen at any
time during the cycle
Which of the above statements about the Solar
Cycle are correct?
a) 1 and 2 Only
b) 2 and 3 Only
c) 1 and 3 Only
d) All of the above
Ans: d.
http://www.bbc.com/earth/story/20150923-the-suncreates-space-weather-that-affects-us-all
4. Question
Which of the following may be consequences of
a Coronal Mass Ejection (CME)?
1. Unlike when in ordinary circumstances
auroras are visible close only to the north
and south poles, it will trigger auroras
worldwide
2. It will generate an electrical current in any
conductive material near the ground
3. We might lose a lot of money stored in
banks
4. The ionosphere thickens and delays the
transmission of signals back to earth
Select the correct answer using codes given
below:
a) 1, 2 and 3 Only
b) 2, 3 and 4 Only
c) None of the above
d) All of the above
Ans: d.
http://www.bbc.com/earth/story/20150923-the-suncreates-space-weather-that-affects-us-all
5. Question
Consider the following statements about the
World Youth Skills Day (WYSD):
1. It was first celebrated in the year 2015
2. WYSD is in line with the United Nations
Skill Development Goals (SDGs)
3. The Government of India launched the
Skill India programme on this day

Current Affairs 7 October-2015

Which of the above statements are correct?


a) 1 and 2 only
b) 2 and 3 only
c) 1 and 3 Only
d) All the above
Ans: c.
WYSD is in line with the United Nations
Sustainable Development Goals (SDGs).
http://en.unesco.org/events/world-youth-skills-day

http://www.thehindu.com/news/national/uphill-taskfor-skill-india-mission/article7682136.ece

03 October 2015
Paper 3 Topic: Conservation, environmental
pollution and degradation.

India to cut emissions intensity


The Union Environment Ministry has finally
submitted its Intended Nationally Determined
Contributions (INDCs) to the UN Framework
Convention on Climate Change (UNFCCC),
committing to cut the emissions intensity of GDP by
33-35% by 2030 from 2005 levels.
All nations were due to come out with
emission targets ahead of a climate change
conference in Paris in December, where they
are supposed to adopt a landmark deal to
fight climate change.
Including India, 120 countries have now
submitted their INDCs.
What are INDCs?
INDCs lay out the blueprint for tackling climate
change. These are individual country commitments
which are expected to indicate through their form
and strength what shape any 2015 agreement might
take.
Every country is submitting these action
plans, called Intended Nationally Determined
Contributions, or INDCs, in official
language, ahead of an annual climate change
conference in December this time being
held in Paris that is expected to deliver a
global climate agreement.
In preparation, countries have agreed to
publicly outline what post-2020 climate
actions they intend to take under a new
international agreement, known as their
Intended Nationally Determined
Contributions (INDCs).
Through INDCs countries put forward their
agreements in the context of their own
national circumstances, capabilities and

priorities, within the ambition to reduce


global greenhouse gas emissions enough to
keep global temperature rise to 2 degrees
Celsius.
The INDCs will not only contain steps taken
towards emission reductions, but also aim to
address steps taken to adapt to climate
change impacts, and what support the
country needs-or will provide to address
climate change.
INDCs emphasise eight key goals
sustainable lifestyles, cleaner economic
development, reducing emission intensity of
GDP, increasing the share of non-fossil fuel
based electricity, enhancing carbon sink,
adaptation and mobilising finance, technology
transfer and capacity building.
Indias proposed targets:
1. Reduce emissions intensity of its GDP by 33
to 35% by 2030 from 2005 level.
2. Achieve about 40% electric power installed
capacity from non-fossil fuel based energy
resources by 2030 with help of transfer of
technology and low cost international
finance.
3. Create an additional carbon sink of 2.5 to 3
billion tonnes of CO2 equivalent through
additional forest and tree cover by 2030.
How much it will cost?
Preliminary estimates suggest India would
require at least USD 2.5 trillion at current
prices to implement all these plans till 2030.
NITI Aayog has said that the mitigation activities for
moderate low carbon development would cost
around 834 billion dollars till 2030.
Indias ongoing efforts to achieve its climate
objectives:
India had set for itself in the run-up to the
2009 climate conference in Copenhagen, the
previous time the world had attempted to
finalise a climate agreement, but had failed.
At that time, India had said it would cut its
emission intensity by 20 to 25% by the year
2020 compared to 2005.
In its INDC, India says its emission intensity
in 2010 had already been cut by 12% as
compared to 2005.
India has already planned to install 175 GW
of power generation capacity through
renewable energy sources by the year 2022.
It has also planned to increase the coal cess
and increase taxes on petrol and diesel. India
has already cut its petroleum subsidy by 26%

Current Affairs 8 October-2015

over the last one year.


Several of governments flagship
programmes like the Smart Cities Mission,
Atal Mission for Rejuvenation and Urban
Transformation (AMRUT), Swachh Bharat
Mission, National Heritage City
Development and Augmentation Yojana

Sources: the hindu, ie, ndtv.


Paper 3 Topic: Bio diversity.

Nilgiri tahr population over 3,000


In a comprehensive study of the Nilgiri tahr
population in Western Ghats, the WWF-India has
put the population at 3,122.
It has also said that with diligent
conservation and planning, the population
could reach the 5,000 mark.
The study was conducted in the habitat
spread over 5,790 sq km in Tamil Nadu and
Kerala.
During the survey, WWF has discovered 17

(HRIDAY), National Mission for Clean


Ganga, Make in India policy, Soil Health
Card scheme, Pradhan Mantri Krishi
Sinchayee Yojana and many others aim to
achieve the climate objectives.
India is the fourth biggest emitter of greenhouse
gases after China, the United States, and European
Union as a whole.

unrecorded pockets of tahr habitat inhabited


by 131 animals. These potential areas had not
been located in the past owing to their rugged
terrain that remains inaccessible for over half
the year owing to heavy rain, mist and fog.
Then study recommends periodic monitoring
for conservation.
The study also helps in preparing a
comprehensive conservation plan.
The threats identified by the study include:
incidents of forest fire destroying fodder
spread of diseases through unregulated cattle

Current Affairs 9 October-2015

crazing in areas close to the habitat


unsustainable non-timber forest produce
collection
poaching and unabated human intervention in
the habitat in the name of pilgrimage to
sacred places
Nilgiri tahr:
The Nilgiri tahr, known locally as the Nilgiri ibex or
simply ibex, is an ungulate that is endemic to the
Nilgiri Hills and the southern portion of the Western
Ghats in the states of Tamil Nadu and Kerala in
Southern India.
It is the state animal of Tamil Nadu.
It has been classified as endangered as per
IUCN.
Sources: The Hindu, Wiki.
Paper 3 Topic: Science and Technologydevelopments and their applications and
effects in everyday life.

TRAI stands by its decision of making


GPS mandatory in all handsets
TRAI has stood by its proposal of a sunset date for
making GPS mandatory in all handsets, which can
help find out exact location of the caller in an
emergency.
It says feature phones are slowly being
replaced, with users shifting to smartphones,
and introducing a GPS chip in handsets
should not be very costly once it is mandated.
Background:
The Telecom Regulatory Authority of India
(TRAI) in April came out with
recommendations on making 112 as the
single emergency number.
TRAI proposed implementation of public
safety answering point (PSAP) based
integrated emergency communication and
response system (IECRS) in the country,
which will be accessed through a single
emergency number 112.
It had proposed that DoT may consider
mandating a transition to global positioning
systems (GPS) enabled handsets in a certain
timeframe to obtain more accurate location
information of the caller.
However, the proposal was rejected by the
Department of Telecom (DoT), saying there
is a major percentage of low-cost handsets
with subscribers which do not support GPS
and the impact of enabling all mobile phone
handsets with GPS may be studied before

taking a call on the issue.


About TRAI:
It is the independent regulator of the
telecommunications business in India.
It was established in 1997 by an Act of
Parliament to regulate telecom services and
tariffs in India.
In January 2000, TRAI was amended to
establish the Telecom Disputes Settlement
Appellate Tribunal (TDSAT) to take over the
adjudicatory functions of the TRAI. The
TDSAT was set up to resolve any dispute
between a licencor and a licensee, between
two or more service providers, between a
service provider and a group of consumers.
In addition, any direction, TRAI orders or
decisions can be challenged by appealing to
TDSAT.
Sources: ie, pib.
1. Question

Consider the following statements:


1. The Saka calendar is the national calendar
along with the Gregorian calendar, used
by the Government of India for certain
official purposes
2. The Kushana emperor Kanishka is alleged
to have initiated the Saka era
3. The Vikram Samvat is said to have begun
with Vikramadityas accession to the
throne in 57 AD
Which of the above statements are incorrect?
a) 1 and 2 only
b) 2 and 3 only
c) 1 and 3 only
d) 3 Only
Ans: d.
http://www.archive.india.gov.in/knowindia/national_
symbols.php?id=10

http://www.thehindu.com/opinion/lead/kanak-manidixit-writes-on-nepals-historic-constitution-applypanchsheel-on-nepal/article7681981.ece

http://www.britannica.com/topic/chronology#ref523
197
2. Question
Which of the following statements about Bakrid
is/are incorrect?
1. It is performed by Muslims to
commemorate the willingness of Hazrath
Ibrahim to sacrifice his only goat to feed
an emaciated man
2. It is also known as Eid-ul-adha

Current Affairs 10 October-2015

3. Blood of the goat is used to wash away


any sins committed by Muslims
4. Islam permits only goats to be sacrificed
during this celebration
Select the correct answer using the codes given
below:
a) 1, 2 and 3 only
b) 1, 3 and 4 only
c) 1 Only
d) 4 Only
Ans: b.
http://www.thehindu.com/features/fridayreview/religion/religious-discourse-significance-ofbakrid/article7681774.ece

http://islam.about.com/od/hajj/a/adha.htm
3. Question
Which among the following are ports-related
performance indicators utilized by the Ministry of
Shipping?
1. Average turnaround time
2. Average pre-berthing time
3. Average output per ship berth-day
4. Average number of moves per crane-hour
Select the correct answer using the codes given
below:
a) 1 and 3 Only
b) 2 and 3 Only
c) 1, 2 and 3 Only
d) All of the above
Ans: d.
Page 116 Volume 2, Economic Survey 2014-15
http://www.performance.gov.in/?
q=ministry/shipping
4. Question
Consider the following statements about the

United Kingdoms Equalities Act, 2010:


1. It recognizes caste discrimination as an
aspect of race discrimination
2. This act was made to tackle, among
others, the menace of race as well as caste
discrimination, within the UK only
Which of the above statements are incorrect?
a) 1 Only
b) 2 Only
c) Both 1 and 2
d) Neither 1 or 2
Ans: c.
http://www.thehindu.com/news/international/woman
-gets-184k-in-uk-caste-bias-case/article7682113.ece

The primary purpose of the Act is to codify the


complicated and numerous array of Acts and
Regulations, which formed the basis of antidiscrimination law in Great Britain. This was,
primarily, the Equal Pay Act 1970, the Sex
Discrimination Act 1975, the Race Relations Act
1976, the Disability Discrimination Act 1995 and
three major statutory instruments protecting
discrimination in employment on grounds of religion
or belief, sexual orientation and age.
https://en.wikipedia.org/wiki/Equality_Act_2010
5. Question
Which among the following contributes the
maximum number of troops to the United
Nations Peacekeeping Operations?
a) India
b) North America
c) Africa
d) Asia Pacific
Ans: c.
As of August 2015, it is Africa.

Current Affairs 11 October-2015

05 October 2015
Paper 3 Topic: conservation.

Fishermen apprehensive as Kerala


prepares to roll out World Bank-aided
project
The Kerala government is gearing up to implement
an Integrated Coastal Zone Management (ICZM)
project aimed at livelihood improvement of coastal
communities and conservation of the coastal
ecosystem, amid voices of protest from the
fishermen community.
The project director has already been
appointed by the government.
Why the fishermen are opposing?
The fishermen are apprehensive about the project
and its impact on the coastline. They fear the project
would pave the way for a construction spree,
jeopardising the fragile coastal environment and
further endangering their livelihood.
About the Integrated Coastal Zone Management
(ICZM):
ICZM aims to improve livelihood of coastal
communities and conserve the coastal
ecosystem.
The ICZM plan involves identification of
infrastructure requirements and livelihood
improvement means in coastal districts.
Conservation of mangroves is among the
components.
The national component of the project
includes mapping of the countrys coastline
and demarcation of the hazard line.
It is a World Bank assisted project.
It is being implemented by the Department of
Forests and Environment with assistance
from the Union Ministry of Environment,
Forests and Climate Change (MoEFCC).
The National Centre for Sustainable Coastal
Management (NCSCM), Chennai, will
provide scientific and technical inputs.
Kerala will be included in the second phase of the
Rs.1,155.63-crore project that has already covered
Gujarat, Odisha and West Bengal.
Sources: The Hindu, wb.
Paper 3 Topic: Infrastructure.

Indian islands to be developed under


Swiss challenge model
The Centre is going to implement a comprehensive
plan to develop Andaman & Nicobar and
Lakshadweep islands, for an integrated

modernisation of the region, under its Sagarmala


initiative.
The plan is to develop these islands under the
Swiss challenge system.
What is swiss challenge system?
Swiss challenge method is a process of
giving contracts. Any person with credentials
can submit a development proposal to the
government. That proposal will be made
online and a second person can give
suggestions to improve and beat that
proposal.
It is a method where third parties make offers
(challenges) for a project within a designated
period to avoid exaggerated project costs.
Is it new to India?
The Swiss challenge method is one that has
been used in India by various states including
Karnataka, Andhra Pradesh, Rajasthan,
Madhya Pradesh, Bihar, Punjab and Gujarat
for roads and housing projects.
In 2009, the Supreme Court approved the method for
award of contracts.
Sagarmala Initiative:
The Sagarmala project seeks to develop a string of
ports around Indias coast. The objective of this
initiative is to promote Port-led development
along Indias 7500 km long coastline.
It aims to develop access to new
development regions with intermodal
solutions and promotion of the optimum
modal split, enhanced connectivity with main
economic centres and beyond through
expansion of rail, inland water, coastal and
road services.
The Union Ministry of Shipping has been
appointed as the nodal ministry for this
initiative.
The Sagarmala initiative will address challenges by
focusing on three pillars of development, namely:
1. Supporting and enabling Port-led
Development through appropriate policy and
institutional interventions and providing for
an institutional framework for ensuring interagency and ministries/departments/states
collaboration for integrated development,
2. Port Infrastructure Enhancement, including
modernization and setting up of new ports,
and
3. Efficient Evacuation to and from hinterland.
Other objectives:
In addition to strengthening port and
evacuation infrastructure, it also aims at

Current Affairs 12 October-2015

simplifying procedures used at ports for


cargo movement and promotes usage of
electronic channels for information exchange
leading to quick, efficient, hassle-free and
seamless cargo movement.
It also strives to ensure sustainable
development of the population living in the
Coastal Economic Zone (CEZ). This would
be done by synergising and coordinating with
State Governments and line Ministries of
Central Government through their existing
schemes and programmes such as those
related to community and rural development,

Paper 3 Topic: Science and TechnologyAchievements of Indians in science &


technology.

tribal development and employment


generation, fisheries, skill development,
tourism promotion etc.
NSAC:
A National Sagarmala Apex Committee (NSAC) is
envisaged for overall policy guidance and high level
coordination, and to review various aspects of
planning and implementation of the plan and
projects. The NSAC shall be chaired by the Minister
incharge of Shipping, with Cabinet Ministers from
stakeholder Ministries and Chief Ministers/Ministers
incharge of ports of maritime states as members
Sources: the hindu, livemint.

DRDO sets up worlds highest terrestrial


centre in Ladakh
Defence Research and Development Organisation
(DRDO) has established the worlds highest

Current Affairs 13 October-2015

terrestrial centre at 17,600 feet above sea level at


Changla near Pengong lake in Ladakh.
Key features:
The centre will serve as a natural cold
storage for preserving rare and endangered
medical plants for generations to come.
The centre will act as an important utility for
research work in frontal areas of food and
agriculture and bio-medical sciences for well
being of the soldiers deployed in high
altitude cold desert.
Other activities that are proposed to be
undertaken here include human physiological
work, designing, testing, validation and
demonstration of mobile and portable
greenhouses, soil-less micro-farming
technologies for fresh food in remote
landlocked posts besides conservation and
propagation of endangered extreme altitude
medicinal plants and others.
Sources: toi.
1. Question

With reference to the Gond tribe of India,


consider the following statements:
1. Their language, Gondi, lacks a working
script
2. Gondwana is a derived from this tribes
name, and it means Land of the Gonds
3. They venerate Ravana (from the Hindu
literature Ramayana), and worship Him
Which of the above statements are incorrect?
a) 1 and 2 only
b) 1 and 3 only
c) All of the above
d) None of the above
Ans: d.
http://www.thehindu.com/news/national/otherstates/on-the-margins-championing-gondculture/article7686156.ece
2. Question
Consider the following statements:
1. Tea Board of India comes under the
Department of Plantations, Ministry of
Commerce and Industry
2. The Plantations Labour Act, 1951 is
applicable to the tea plantations as well
Which of the above statements are incorrect?
a) 1 only
b) 2 only
c) Both 1 and 2
d) Neither 1 or 2
Ans: a.
There is no such Department of Plantations.

However, the Tea Board of India does come under


the Ministry of Commerce and Industry.
http://www.thehindu.com/business/Industry/centreplans-to-liberalise-tea-act/article7686113.ece
3. Question
Which among the following are mentioned in
List-III of the Seventh Schedule in the Indian
Constitution?
1. Criminal Law
2. Criminal Procedure
3. Markets and fairs
4. Population Control and family planning
5. Elections to the Legislature of the State,
subject to the provisions of any law made
by Parliament
Select the correct answer using the codes given
below:
a) 1, 2 and 3 only
b) 3, 4 and 5 only
c) 1, 2 and 4 Only
d) 2, 3 and 5 only
Ans: c.
List III is the Concurrent List.
From one of our earlier questions, you would know
that Statement 5 is a subject from the State List.
Here itself two options get eliminated. The Centre is
unable to deal with faults in agricultural markets in a
concrete manner because of the subject in Statement
3 being a state subject. Here, option a) gets
eliminated. You are left only with c).
http://lawmin.nic.in/olwing/coi/coienglish/Const.Pock%202Pg.Rom8Fsss(35).pdf

http://www.thehindu.com/news/national/centreclears-controversial-antiterror-bill-ofgujarat/article7686664.ece
4. Question
Consider the following statements about the
India-Africa Forum Summit:
1. The first such summit will be held in New
Delhi in 2015
2. This biennial summit is to be held on a
rotational basis between India and Africa
Which of the above statements are incorrect?
a) 1 Only
b) 2 Only
c) Both 1 and 2
d) Neither 1 or 2
Ans: c.
Mr. Akbaruddin has been entrusted with bringing
together the heads of 54 African nations to the IndiaAfrica summit in Delhi on October 26.
http://www.thehindu.com/news/national/foreign-

Current Affairs 14 October-2015

secretary-jaishankar-puts-his-imprint-in-mea-teamreshuffle/article7686187.ece

https://en.wikipedia.org/wiki/India
%E2%80%93Africa_Forum_Summit
5. Question
Consider the following statements:
1. It consists of a Methane Sensor
2. A Lyman Alpha Photometer which will
help us understand the loss-process of
water from the planet
3. A Magnetometer which will measure
ionosphere magnetic field characteristics
Which of the statements is/are associated with the
Mars Orbiter Mission and is/are true?
a) 1 Only
b) 1 and 2 only
c) 1 and 3 Only
d) All of the above
Ans: b.
Magnetometer is aboard the MAVEN.
http://www.isro.gov.in/pslv-c25-mars-orbitermission/payloads

http://pib.nic.in/newsite/PrintRelease.aspx?
relid=127146
6. Question
Consider the following statements associated
with the diesel engine:
1. It is an internal combustion engine which
utilizes spark plugs for fuel ignition
2. The United Nations Health Agency has
classified its exhaust as carcinogenic to
humans
3. Not only does it emit Carbon Monoxide, it
also emits Diesel Particulate Matter
4. Diesel has a higher boiling point than that
of water
Which of the above statements are correct?
a) 1 and 3 Only
b) 2 and 4 Only
c) 1, 2 and 3 only
d) 2, 3 and 4 Only
Ans: d.
http://www.un.org/apps/news/story.asp?
NewsID=42204#.VgY8xRGqqko

https://en.wikipedia.org/wiki/Diesel_exhaust#Particu
late_matter

http://auto.howstuffworks.com/diesel.htm

06 October 2015
Paper 3 Topic: Environmental pollution.

Nuclear energy not viable, says German


economist
A German green economist has said that nuclear
energy is not viable. He says, India will do better if
it invests in solar and in wind power than in nuclear
energy.
Why he says so?
He cites the following reasons:
Nuclear energy is not green. It is inherently
high-risk. If something goes wrong then it
can be catastrophic like Chernobyl and
Fukushima.
There is also the problem of nuclear
proliferation. There is a very thin wall
between civil and military nuclear
applications.
Increasing cost of plants is also a matter of
concern. The cost is growing. Nuclear power
will not be able to survive without
government subsidies. The market cannot
decide it.
Sources: the Hindu.
Paper 3 Topic: e-technology in the aid
of farmers.

Drones to help gauge crop damage


To fasten payment of crop insurance claims to
farmers, the Centre has launched a pilot programme
Kisan, which will use satellite and drone-based
imaging and other geospatial technology to get
timely and accurate data on crop yields.
The programme envisages use of high
resolution remote sensing data both from
satellite and drone-based imaging,
sophisticated modelling activity and other
geospatial technology for improving the
accuracy of crop yield estimation through
more efficient crop cutting experiments.
Why it was necessary?
Currently, the crop insurance claim is calculated on
the basis of crop cutting experiments. However,
there has always been a problem in getting timely
and accurate data, due to which payment of claims
to farmers were getting delayed and the government
is concerned over the delays in settlements. To
address this issue Kisan programme is being
launched on pilot basis.
Advantages:
Gives accurate data to enable crop insurance
companies to give proper compensation to

Current Affairs 15 October-2015

affected farmers.
Helps develop index-based data for insurance
companies.
The scientific data collected by drones and
collated with satellites imagery will be
matched with traditional crop cutting
experiments to arrive at a foolproof data.
It also helps in block level yield estimation.
Initially, the pilot study will be carried out in rice
and cotton fields in four districts. The programme
will be scaled up across the country after assessing
the results.
The programme will be jointly conducted by
Mahalanobis National Crop Forecast Centre, Indian
Space Research Organisation, India Meteorological
Department, State Agriculture Departments and
Remote Sensing Centres, Climate Change,
Agriculture and Food Security (CCAFS).
Sources: the Hindu.
Paper 3 Topic: e-technology in the aid
of farmers.

App to collect hailstorm data


The centre has launched an Android-based app for
collection of data of hailstorm. This gives access to
large-scale damage to standing crops.
The app will be used by state agriculture
officials for data collection and the data will
help the union agriculture ministry in having
very fast assessment of damage to crops
because of hailstorm.
Even farmers can download the app and send
pictures of hailstorm.
The app has been developed with the support
of ISRO.
The app can be used through smartphones for
collection of hailstorm data along with
photographs and locations and can be
uploaded on real-time to ISROs Bhuvan
server.
The app allows the farmers to immediately send
photos of their crop damage to officials concerned
for immediate relief. This will cut the red tape in
reaching assistance to farmers
Hailstorm:
Any thunderstorm which produces hail that
reaches the ground is known as a hailstorm.
Hail is a form of solid precipitation. It
consists of balls or irregular lumps of ice,
each of which is called a hailstone.
Hail formation requires environments of
strong, upward motion of air with the parent

thunderstorm and lowered heights of the


freezing level. In the mid-latitudes, hail
forms near the interiors of continents, while
in the tropics, it tends to be confined to high

elevations.
Hailstorms cause large scale damage to standing
crops. However, at present, there is no
comprehensive approach to collect hailstorm data.
Sources: The Hindu, Wiki.
Paper 2 Topic: Bilateral, regional and
global groupings and agreements
involving India and/or affecting Indias
interests.

Germany wont sign MLAT, cites death


penalty
Germany has expressed its inability to sign the
Mutual Legal Assistance Treaty (MLAT) with India,
citing its provision for death penalty for heinous
crimes and terror activities.
This is perhaps the first time a country has
refused to sign the treaty on grounds of the
death penalty provision.
India has signed MLAT with 39 countries,
including the United States.
India and Germany have been negotiating
since 2007 to sign the Mutual Legal
Assistance Treaty in criminal matters but
have not been able to reach a conclusion due
to Germanys strong reservation to the
provision of death penalty in Indian law.
About Mutual Legal Assistance Treaty (MLAT):
MLAT is an agreement between two or more
countries for gathering and exchanging
information to enforce public or criminal
laws.
Under the agreement, mechanisms have been
developed among nations for requesting and
obtaining evidence for criminal
investigations and prosecutions.

Current Affairs 16 October-2015

Mutual legal assistance: When evidence or other


forms of legal assistance, such as witness statements
or the service of documents, are needed from a
foreign sovereign, states may attempt to cooperate
informally through their respective police agencies
or, alternatively, resort to what is typically referred
to as requests for mutual legal assistance.
Abolition of death penalty in India:
The Law Commission of India has already
recommended for abolition of death penalty except
in terror-related cases. However, the Home Ministry
is believed to be against it maintaining that time was
not ripe yet to remove it completely from the statute
book keeping in mind the threat from terrorism.
Sources: The Hindu, Wiki.
Paper 3 Topic: Science and Technologydevelopments and their applications and
effects in everyday life.

3 win Nobel Prize in Medicine for


parasite-fighting therapies
Three scientists from the US, Japan and China have

won the Nobel Prize in medicine for discovering


drugs to fight malaria and other tropical diseases
that affect hundreds of millions of people every year.
The three scientists are:
1. Santoshi omura from Japan
2. Youyou tu from China
3. William campbell from Ireland
Facts:
Campbell and Omura were cited for
discovering avermectin, derivatives of which
have helped lower the incidence of river
blindness and lymphatic filariasis, two
diseases caused by parasitic worms that
affect millions of people in Africa and Asia.
Tu discovered artemisinin, a drug that has
helped significantly reduce the mortality
rates of malaria patients. Tu Youyou is the
first-ever Chinese medicine laureate.
These discoveries have provided humankind with
powerful new means to combat these debilitating
diseases that affect hundreds of millions of people
annually.

River blindness is an eye and skin disease that ultimately leads to blindness. About 90% of the disease
occurs in Africa, according to the World Health Organization.

Current Affairs 17 October-2015

Lymphatic filariasis can lead to swelling of the limbs and genitals, called elephantiasis, and its
primarily a threat in Africa and Asia. The WHO says 120 million people are infected with the disease,
without about 40 million disfigured and incapacitated.

Malaria is a mosquito-borne disease that still kills around 500,000 people a year, mostly in Africa,
despite efforts to control it.

Current Affairs 18 October-2015

Nobel Prize in Physiology or Medicine:


The Nobel Prize in Physiology or Medicine is awarded once a year for outstanding discoveries in the
fields of life sciences and medicine. It is awarded by the Swedish Karolinska Institute to scientists
and doctors.
It is one of five Nobel Prizes established in 1895 by Swedish chemist Alfred Nobel, the inventor of
dynamite, in his will.
It carries a cash prize of 8 million Swedish kronor (about $960,000).
Sources: the Hindu, wiki.
Paper 2 Topic: Government policies and
interventions for development in various
sectors and issues arising out of their
design and implementation.

Road Safety Bill will give govt power to


order recall of vehicles: Gadkari
In the wake of the Volkswagen emissions scandal,
Road Transport Minister Nitin Gadkari has
underscored a clause in the pending Road Safety Bill
that allows the government to direct a manufacturer
to recall motor vehicles if a defect in that particular
type of vehicle may cause harm to the driver or
occupants or road users.
According to the Bill, a manufacturer can be
ordered to recall a particular type of motor
vehicle if a specified number of users
complain about a defect that can cause harm
to them, their passengers, or any other road
users.
The Bill could not be introduced in the last
session Parliament.
It is also to be noted that the Road safety Act
falls in the purview of concurrent list and

both state governments and the Centre have


rights.
Road Transport and Safety Bill:
It is a Bill which aims to provide a framework for
safer, faster, cost effective and inclusive movement
of passengers and freight in the country thus
enabling the mission of Make in India.
Highlights of the Bill:
2 lakhs lives to be saved in first 5 years due
to reduction in road traffic accident deaths
4% GDP improvement on account of
increased efficiency and safety of road
transport sector
10 lac Jobs to be created with increase in
investment in the sector
The new Bill makes significant departures
from the 1988 Motor Vehicle Act as it
includes safety in construction, design,
maintenance and use of motor vehicles and
roads as a major component.
The Bill provides for more stringent penalties
to offenders. A graded penalty point system
would now act as a deterrent and improve

Current Affairs 19 October-2015

traffic condition whereas electronic detection


and centralized information of offences
would facilitate to identify repeat-offenders.
New proposed Agencies and systems:
The Bill proposes to introduce an
independent agency called the National Road
Safety Authority of India, which will be an
independent, legally empowered and
accountable expert lead agency. It shall be
accountable to the Parliament and Central
Government.
The new Bill provides for the establishment
of State Safety Authorities which shall act in
accordance with the directions issued by the
National Authority.
The Bill seeks to establish a unified driver
licensing system in India which will be
transparent. Such a system shall facilitate any
time anywhere licence application
mechanism in the country and mitigate
duplication of licences from various regional
transport offices.
According to the Provisions of the Bill there
will be a unified vehicle registration system
to enable electronic and online submission of
applications for registration at any registering
authority leading to real time interchange of
data relating to such an activity.
On the safety issues, the Bill envisages for
enforcement of modern safety technologies.
It also contains the provision for creation of a
motor vehicle accident fund for immediate
relief to the accident victim. It gives special
emphasis on safety of school children and
security of women.
The Bill also includes the setting up of a
Highway Traffic Regulation and Protection
Force (HTRPF).
Why are some against this Bill?
Due to some provisions in the proposed bill.
They say that the proposed fines are too high.
According to provisions of the Bill, the
Motor Vehicle Act 1988 will be scrapped and
State RTOs will close. Instead, a Central
authority will be created and private entities
will issue and renew licences. This move is
not being welcomed.
The provisions in the Bill are said to be
against the principles of jurisprudence.
Some state governments allege that the bill
encroaches upon the financial, legislative and
administrative powers of state governments.

Sources: The Hindu, pib.


Paper 2 Topic: Issues relating to poverty
and hunger.

World Bank estimates show fall in Indias


poverty rate
The World Bank has revised the global poverty line,
previously pegged at $1.25 a day to $1.90 a day
(approximately Rs. 130). This was stated in its latest
report Ending Extreme Poverty, Sharing Prosperity:
Progress and Policies.
The new poverty line has been arrived at
based on an average of the national poverty
lines of 15 poorest economies of the world.
The poverty lines were converted from local
currency into U.S. dollars using the new
2011 Purchasing Power Parity (PPP) data.
Notable facts:
The latest headline estimate for 2012 based
on the new data suggests that close to 900
million people (12.8% of the global
population) lived in extreme poverty.
Compared with 2011, this number represents
continued poverty reduction, as the
headcount estimate then, using 2011 PPP
data, was 987 million people (14.2% of
global population).
Indias poverty rate is one of the lowest
among those countries with the largest
number of poor. Indias poverty rate for
2011/12 is 21.2%.
The number of people living in extreme
poverty around the world is likely to fall to
under 10% of the global population in 2015.
The poverty rate in low-income countries
averages 43% in 2012, compared to 19% in
lower-middle-income countries. Yet lower
middle-income countries are home to about
half of the global poor, compared to a third
for low-income countries.
In South Asia, the poverty would fall to
13.5% in 2015 compared to 18.8% in 2012;
Sub-Saharan Africa poverty would decline to
35.2% in 2015 compared to 42.6% in 2012.
Tentative projections for global poverty in
2015 suggest that the global headcount may
have reached 700 million, leading to a
poverty rate of 9.6 %.
With the Sustainable Development Goals adopted in
September, seeking to end all forms of poverty
world over, the World Bank Group has set itself the
target of bringing down the number of people living

Current Affairs 20 October-2015

in extreme poverty to less than 3% of the world


population by 2030.
Sources: The Hindu.
Paper 2 Topic: Bilateral, regional and
global groupings and agreements.

U.S., 11 nations reach historic deal


Trade ministers from the Asia-Pacific region have
reached a deal on the Pacific trade pact that is
intended to cut trade barriers and establish common
standards for 12 countries. This is the largest trade
pact in 20 years.
About the Trans-Pacific Partnership:
The Trans-Pacific Partnership is headed by
the US and includes Australia, Brunei,
Canada, Chile, Japan, Malaysia, Mexico,
New Zealand, Peru, Singapore, and Vietnam.
The agreement covers 40% of the worlds
economy.
It would set new terms for trade and business
investment among the United States and 11
other Pacific Rim nations.
It would phase out thousands of import tariffs

as well as other barriers to international


trade. It also would establish uniform rules
on corporations intellectual property, open
the Internet even in communist Vietnam and
crack down on wildlife trafficking and
environmental abuses.
What the supporters say?
It would be a boon for all the nations
involved. It would unlock opportunities and
address vital 21st-century issues within the
global economy.
What the opponents say?
Opponents in the United States see the pact
as mostly a giveaway to business,
encouraging further export of manufacturing
jobs to low-wage nations while limiting
competition and encouraging higher prices
for pharmaceuticals and other high-value
products by spreading American standards
for patent protections to other countries.
A provision allowing multinational corporations to
challenge regulations and court rulings before
special tribunals is drawing intense opposition.

Current Affairs 21 October-2015

Why the US is interested in this pact:


The pact is a major component of President Obamas pivot to Asia. It is seen as a way to bind
Pacific trading partners closer to the United States while raising a challenge to Asias rising power,
China, which has pointedly been excluded from the deal, at least for now.
It is seen as a means to address a number of festering issues that have become stumbling blocks as
global trade has soared, including e-commerce, financial services and cross-border Internet
communications.
Sources: the Hindu, et.
Trivia:
revolution which played a big role in increasing
India has decided to set up a fast-track system for
milk production in the country.
German companies in the Department of Industrial
http://pib.nic.in/newsite/PrintRelease.aspx?
Policy and Promotion (DIPP), Ministry of
relid=111173
Commerce and Industry. The wing will become fully
operational by 2016. The exclusive special window
The following is only an informal proposal by India
for Germany will be the second since a similar wing at the international platform as of now (besides, even
was created for Japan after Prime Minister Narendra
if accepted, it would not be led as such by any one
Modis visit to Japan in September 2014.
country):
1. Question
At the special summit of UN, Mr. Modi also
With reference to the Flag Code of India, which
introduced an idea of the Blue Revolution, which
of the following statements is/are incorrect?
he elaborated as a special effort to preserve the
1. It contains a general description of the
oceans and ensure the sustenance and prosperity of
National Flag
island nations, particularly the smaller ones that face
2. Any Indian citizen can display the
the brunt of climate change.
National Flag at his or her residence
Keep yourself aware of any further developments on
3. The Flag can be used as a covering for a
Blue Revolution.
statue prior to its unveiling, provided the
3. Question
statue is placed on a pedestal
Consider the following statements:
Select the correct answer using codes given
1. His philosophy of Integral Humanism is a
below:
synthesis of the material and the spiritual
a) 1 and 2 only
2. He started a weekly Panchajanya and a
b) 2 and 3 only
daily Swadesh
c) 2 Only
3. He founded the Sanatan Dharma College
Which of the above statements about Pandit
d) 3 Only
Deendayal Upadhyaya are true?
Ans: d.
a) 1 and 2 only
Appendix IX, Flag Code of India Indian Polity by
b) 2 and 3 only
M Laxmikanth
2. Question
c) 1 and 3 Only
What is the Blue Revolution as envisaged by
d) All the above
the Government of India?
Ans: a.
a) Transformation of the fisheries sector
http://pibphoto.nic.in/documents/rlink/2015/sep/p20
with increased investment, better training
1592501.pdf
and development of infrastructure
4. Question
b) Transformation of Indian trade and
Which of the following statements associated
commerce through ports-led development
with Rani Gaidinliu are incorrect?
c) A special effort to be led by India to
1. She belonged to an ethnic Naga tribe
2. By her followers, she was considered to
protect and preserve the five oceans of the
be an incarnation of a goddess
world
3. She joined the Tharewukara Reformist
d) A special effort to rejuvenate the rivers
Religious Movement (TRRM) which
of India
steadfastly turned out to be a political
Ans: a.
movement against the British Raj
Union Minister for Agriculture Radhamohan Singh
4. TRRM recognizes supremacy of one God
has said that the government is all set to launch the
who is behind creation of nature be it air,
blue revolution on the lines of the white
Current Affairs 22 October-2015

water or earth
Select the correct answer using codes given
below:
a) 1 and 3 Only
b) 2, 3 and 4 Only
c) 3 and 4 Only
d) 1,3 and 4 Only
Ans: c.
There is no Tharewukara movement!
http://pib.nic.in/newsite/mbErel.aspx?relid=127060

07 October 2015
Paper 3 Topic: Effects of liberalization on the
economy, changes in industrial policy and
their effects on industrial growth.

e-commerce likely to be $80 billion


industry by 2020: report
According to a report released by Snapdeal and
KPMG, the e-commerce sector in India is projected
to cross $80 billion by 2020, and grow further to
$300 billion by 2030. This growth will mainly be
driven by growing adoption of smartphones and
increasing Internet penetration.
The report is titled, Impact of e-commerce
on SMEs in India.
What the report says?
Around 85% of small and medium
enterprises (SMEs), which adopt ecommerce, believe that it was a cost effective
medium to grow sales.
SMEs using Internet for business have seen
51% higher revenues and 49% more profit.
About 73% SMEs feel that e-commerce
enables them to understand their markets
better and 46% SMEs have seen increase in
business through listings on online
marketplaces.
e-commerce-enabled SMEs also reported 60-

80% reduction in distribution, marketing and


sales spend to get incremental business.
The report also points out patchy Internet
connectivity due to underdeveloped infrastructure, a
general lack awareness about the benefits that ecommerce offers and a lack of trust among
organisations considering to go online, are some of
the key challenges faced by the e commerce sector.
Challenges for SMEs:
Inadequate financing is one of the major
challenges for small and medium enterprises.
Approximately 41% of SMEs in India do not
have access to bank loans or other products
offered by financial institutions, with a
financing gap of more than Rs.2.93 trillion in
the SME sector.
This fast paced growth of the ecommerce industry in
India represents an unprecedented opportunity for
the sector.
Sources: the hindu.
Paper 3 Topic: indigenization of
technology.

INS Astradharini commissioned


The Indian Navy has commissioned the first
indigenously-designed and built torpedo launch and
recovery vessel INS Astradharini.
About the Vessel:
INS Astradhani is an advanced replacement
for Astravahini which was decommissioned
on July 17 this year.
It can operate at high sea states and has a
large deck area with Torpedo Launchers for
deploying and recovering various kinds of
torpedos during the trials. It has a length of
50 m.
It has modern power generation and
distribution, navigation and communication
systems.

Current Affairs 23 October-2015

The unique hull form of the ship demonstrates


the countrys ship design and shipbuilding
capabilities. The catamaran hull configuration
significantly reduces the power requirement of
the ship that is capable of attaining 15 knots.
The unique design of the vessel is a
collaborative effort of Naval Science and
Technological Laboratory (NSTL), IIT
Kharagpur and Shoft Shipyard.
INS Astradhani will be used to carry out the
technical trials of underwater weapons and
systems developed by NSTL.
It reflects the capability of our scientists and
manufacturing facilities and would go a long
way in the Make in India campaign.
sources: pib.
Paper 3 Topic: Effects of liberalization
on the economy, changes in industrial
policy and their effects on industrial
growth.

IMF cuts Indias growth forecast for FY16


to 7.3%
In its latest World Economic Outlook (WEO), the
IMF has forecast global growth of 3.1% this year
and 3.6% in 2016.
The IMF has forecast a lower global growth
this year citing modest pickup in advanced
economies and a slowdown in emerging
markets, primarily reflecting weakness in
some large developing nations and oilexporting countries.
What the WEO says?
The new forecasts mark down expected nearterm growth marginally. Moreover, downside
risks to the world economy appear more
pronounced than they did just a few months
ago. Hence, the WEO underlines the need for
policymakers to raise actual and potential
growth.
Growth in advanced economies is projected
to increase modestly this year and next.
This years pickup reflects primarily a
strengthening of the modest recovery in the
eurozone and a return to positive growth in
Japan, supported by declining oil prices,
accommodative monetary policy, and
improved financial conditions, and in some
cases, currency depreciation.
While growth prospects in emerging markets
and developing economies vary across
countries and regions, the outlook in 2015 is

generally weakening, with growth for these


economies as a group projected to decline
from 4.6% in 2014 to 4% in 2015.
The fifth straight year of slowing growth
reflects a combination of factors weaker
growth in oil exporters, a slowdown in China
with less reliance on commodity-intensive
investment, adjustment in the aftermath of
credit and investment booms, and a weaker
outlook for exporters of other commodities,
including in Latin America, following
declines in their export prices.
Some experts say that these forecasts reflect a world
economy that is at the intersection of at least three
powerful forces. First, Chinas economic
transformation away from export- and investmentled growth and manufacturing, in favour of a greater
focus on consumption and services; second, and
related, the fall in commodity prices; and third, the
impending increase in US interest rates, which can
have global repercussions and add to current
uncertainties.
Sources: BS.
Paper 3 Topic: Science and Technologydevelopments and their applications.

Kajita, McDonald win physics Nobel for


neutrino work
Takaaki Kajita of Japan and Arthur McDonald of
Canada have won the 2015 Nobel Prize in physics
for discovering the chameleon-like nature of
neutrinos, work that yielded the crucial insight that
the tiny particles have mass.
The two researchers had made key
contributions to experiments showing that
neutrinos change identities as they whiz
through the universe at nearly the speed of
light.
Kajita showed in 1998 that neutrinos
captured at the detector underwent a
metamorphosis in the atmosphere. Three
years later McDonald found that neutrinos
coming from the sun also switched identities.
The winners will split the 8 million Swedish
kronor (about $960,000) prize money. Each
winner also gets a diploma and a gold medal
at the prize ceremony on December 10.
What are neutrinos?
Neutrinos are miniscule particles created in nuclear
reactions, such as in the sun and the stars, or in
nuclear power plants. There are three kinds of
neutrinos.
Neutrinos interact with matter via the weak

Current Affairs 24 October-2015

force. The weakness of this force gives


neutrinos the property that matter is almost
trans- parent to them.
Since they rarely interact, these neutrinos
pass through the Sun, and even the Earth,
unhindered. There are many other natural
sources of neutrinos including exploding
stars (supernovae), relic neutrinos, natural
radioactivity, and cosmic ray interactions in
the atmosphere of the Earth.
The neutrino was proposed by Wolfgang
Pauli in 1930; but it took another 26 years for
it to be actually detected. In 1956 Reines and
Cowan found evidence of neutrino
interactions by monitoring a volume of
cadmium chloride with scintillating liquid
near to a nuclear reactor. Reines was jointly
awarded the Nobel Prize in Physics in 1995
in part for this revolutionary work.
sources: the hindu, wiki.
Paper 3 Topic: Investment models.

7,500 km of BOT highway projects at risk:


CRISIL
According to CRISIL Ratings, around 7,500 km of
highway projects in India 5,100 km under
construction and 2,400 km operational awarded
between FY10 and FY12 on a build, operate,
transfer (BOT) basis are at high risk.
CRISILs analysis shows around 50% of the
projects under construction are at high risk of
not being completed because of significant
cost overruns and weak wherewithal of
sponsors.
However, CRISIL Ratings considers the
recent government move to ensure 80%
right of way before a project is awarded to
be constructive.
How the situation can be improved?
The removal of restriction on exit clause can
allow developers to sell stakes in some
projects and raise about Rs 5,000 crore.
These funds can be used to turn around
stressed projects, meet existing commitments
and also as growth capital.
A change of promoter in projects sold could
also open up access to better refinancing
terms and further financial support.
Developers with operational projects can also
securitize receivables to either raise
additional debt to support fund commitments
or to improve project viability by realigning

debt repayment and reducing cost of


borrowing. Around Rs 15,000 crore of debt
can be refinanced through capital market
including through Infrastructure Debt Funds
(IDFs).
About Buildoperatetransfer (BOT):
It is a form of project financing, wherein a private
entity receives a concession from the private or
public sector to finance, design, construct, and
operate a facility stated in the concession contract.
BOT finds extensive application in
infrastructure projects and in publicprivate
partnership.
How it operates?
In the BOT framework a third party, delegates to a
private sector entity to design and build
infrastructure and to operate and maintain these
facilities for a certain period. During this period the
private party has the responsibility to raise the
finance for the project and is entitled to retain all
revenues generated by the project and is the owner
of the regarded facility. The facility will be then
transferred to the public administration at the end of
the concession agreement, without any remuneration
of the private entity involved.
However, in such projects the private entity bears a
substantial part of the risk. Some types of the most
common risks involved:
Political risk: especially in the developing
countries because of the possibility of
dramatic overnight political change.
Technical risk: construction difficulties, for
example unforeseen soil conditions,
breakdown of equipment.
Financing risk: foreign exchange rate risk
and interest rate fluctuation, market risk
(change in the price of raw materials),
income risk (over-optimistic cash-flow
forecasts), cost overrun risk.
sources: bs, wiki.
1. Question

Consider the following statements:


1. The CDM (Clean Development
Mechanism) has been established under
the Kyoto Protocol
2. Parties participating in the second
commitment period of the Kyoto Protocol
represent only 12 per cent of global
emissions
Which of the above statements are correct?
a) 1 Only
b) 2 Only
c) Both 1 and 2

Current Affairs 25 October-2015

d) Neither 1 or 2
Ans: c.
Pg 128, Economic Survey 2014-15 Volume 2
http://pib.nic.in/newsite/PrintRelease.aspx?
relid=128224
2. Question
If potatoes cost Rs.100 per kg in August 2014,
Rs.110 per kg in August 2015 and Rs.120 per kg
in August 2016, which two processes in these two
periods, in the context of rise and fall of prices,
are respectively taking place?
a) Inflation and Inflation
b) Inflation and Disinflation
c) Inflation and Deflation
d) Hyperinflation and Inflation
Ans: b.
Rate of increase from Aug 2014 to Aug 2015 = 10%
Rate of increase from Aug 2015 to Aug 2016 = less
than 10%
looking at the Consumer Price Index, India is
currently technically going through a phase of
disinflation.
http://www.thehindu.com/business/ready-reckonerwhat-isdeflation-and-is-it-bad/article7692832.ece
3. Question
With reference to commitments by different
countries towards emissions reduction, consider
the following statements:
1. India has already set a voluntary goal of
reducing the emissions intensity of its
GDP (excluding emissions from
agriculture) by 20-25 per cent by 2020 as
compared to the base year of 2005
2. The United States of America intends to
reduce by 2030 its emission of greenhouse
gases by 26 per cent to 28 per cent below
its 2005 level
3. China intends to peak its emissions of
carbon dioxide in 2030, if not earlier
Which of the above statements is/are correct?
a) 1 and 2 only
b) 2 and 3 only
c) 1 and 3 Only
d) 3 Only
Ans: c.
The United States of America intends to reduce by
2025 its emission of greenhouse gases by 26 per cent
to 28 per cent below its 2005 level.
Pg 122-123, Economic Survey 2014-15 Volume 2
http://www.thehindu.com/news/international/world/c
limate-change-us-china-unveil-ambitious-goals-tocut-pollution-levels/article6589412.ece

http://www.thehindu.com/news/national/india-toannounce-climate-commitments-on-gandhijayanti/article7692846.ece
4. Question
The Global Innovation Index, a leading
benchmarking tool for anyone seeking insight
into the state of innovation of a country, is copublished by?
1. World Bank
2. Cornell University
3. INSEAD (European Institute of Business
Administration)
4. Innovation International
5. The World Intellectual Property
Organization (WIPO, an agency of the
United Nations)
6. Organisation for Economic Cooperation
and Development
Select the correct answer using the codes given
below:
a) 1, 4 and 5 only
b) 2,3 and 5 Only
c) 1,2 and 6 Only
d) 4, 5 and 6 only
Ans: b.
https://www.globalinnovationindex.org/content/page
/GII-Home

http://www.thehindu.com/todays-paper/tpnational/techies-want-innovation-fewer-regulatoryhassles/article7693293.ece
5. Question
Consider the following statements:
1. The Mysore Wars refer to four military
confrontations between the Marathas and
rulers of Mysore
2. Tipu Sultans negotiations with the French
served the British a pretext to launch the
fourth Mysore War
Which of the above statements are incorrect?
a) 1 only
b) 2 only
c) Both 1 and 2
d) Neither 1 or 2
Ans: a.
http://www.britannica.com/topic/Mysore-Wars

http://www.britannica.com/biography/Tippu-Sultan

http://www.thehindu.com/news/national/karnataka/ti
pu-sultan-a-secular-internationalist-not-abigot/article7692879.ece
6. Question

Current Affairs 26 October-2015

Which of the following statements about


INTACH (Indian National Trust for Art and
Cultural Heritage) is/are true?
1. It is under the Ministry of Culture,
Government of India
2. Its mission is in line with the
fundamental duty of preserving the rich
heritage of our composite culture
Select the correct answer using the codes given
below:

a) 1 Only
b) 2 Only
c) Both 1 and 2
d) Neither 1 0r 2
Ans: b.
http://www.thehindu.com/news/national/karnataka/g
overnments-urged-to-safeguard-monuments-atgulbarga-fort/article7693558.ece

http://www.intach.org/about-mission.php

Insights into Editorials:


Internet Access at the Cost of Net neutrality?
08 October 2015

Access at the cost of Net neutrality?


Earlier this year, the social media giant, Facebook, formalised a partnership with Reliance Communications
that enabled Reliance to provide access to over 30 different websites, without any charge on mobile data
accruing to the ultimate user.
The platform, originally known as Internet.org, has now been rebranded as Free Basics.
However, its fundamental ethos remains unchanged.
What it does?
It allows Reliances subscribers to surf completely free of cost a bouquet of websites covered within
the scheme, which includes facebook.com.
How is it being viewed?
Facebook founder, Mark Zuckerberg, views this initiative as a philanthropic gesture, as part of a purported,
larger aim to bring access to the Internet to those people who find the costs of using generally available
mobile data prohibitive.
But, there are many critics who argue that Free Basics violates what has come to be known as the principle
of network (or Net) neutrality.
So, what is Net neutrality?
Net neutrality is an interpretive concept. The term was coined by Tim Wu an American lawyer and
presently a professor at the Columbia University. He views the notion of Net neutrality as signifying an
Internet that does not favour any one application over another. In other words, the idea is to ensure that
Internet service providers do not discriminate content by either charging a fee for acting as its carrier or by
incorporating any technical qualifications.
How is it managed in India?
There are no laws enforcing net neutrality in India. Although TRAI guidelines for the Unified Access Service
license promote net neutrality, it does not enforce it. The Information Technology Act 2000 also does not
prohibit companies from throttling their service in accordance with their business interests.
TRAIs recent draft consultation paper:
Recently, the Telecom Regulatory Authority of India (TRAI) released a draft consultation paper seeking the
publics views on whether the Internet needed regulation. Much of its attention was focussed on the
supposedly pernicious impact of applications such as WhatsApp and Viber, and very less on net neutrality.
TRAI says, In a multi-ethnic society there is a vital need to ensure that the social equilibrium is not
impacted adversely by communications that inflame passions, disturb law and order and lead to
sectarian disputes.
The basic questions raised by the above view are:
Should at least some Internet applications be amenable to a greater regulation?
Should they compensate the telecom service providers in addition to the data charges that the
consumers pay directly for the use of mobile Internet?
What if the government answers these questions in the affirmative?
Current Affairs 27 October-2015

If the government eventually answers these questions in the affirmative, the consequences could be drastic.
It could lead to a classification of Internet applications based on arbitrary grounds, by bringing some
of them, whom the government views as harmful to society in some manner or another, within its
regulatory net.
Through such a move, the state, contrary to helping establish principles of Net neutrality as a rule of
law, would be actively promoting an unequal Internet.
Why it is necessary to have a specific law mandating net neutrality?
In the absence of a specific law mandating a neutral Internet, telecom companies enjoy a virtual carte
blanche to discriminate between different applications.
Though these companies have not yet completely exploited this autonomy, they are certainly
proceeding towards such an exercise.
Also alarming is that mobile Internet service providers could, in the future, plausibly also control the
speeds at which different applications are delivered to consumers. This kind of discrimination tends
to breed an unequal playing field, and, if allowed to subsist, it could create a deep division in the
online world.
Airtel Zero case:
In April this year, Airtel announced Airtel Zero, an initiative that would allow applications to
purchase data from Airtel in exchange for the telecom company offering them to consumers free of
cost.
Airtel Zero was widely perceived as a violation of net neutrality which could potentially stifle
innovation and startup growth. There was also an allegation that it effectively tilts the balance in
favor of the bigger players.
By paying to be on Airtel Zero, companies could make sure that their users get free access to their
service, while smaller players are at a disadvantage. However, Airtel has said that zero rating does not
violate net neutrality as it lowers the cost of access and it is non-discriminatory.
To prevent such things, it appears necessary to have a specific mandating a neutral internet.
Views of telecom companies:
Telecom companies that wish to discriminate between applications argue that in the absence of an
Internet that has completely permeated all strata of society, an obligation to maintain neutrality is not
only unreasonable on the companies, but also unfair on the consumer.
They argue that initiatives like internet.org and Airtel zero bring, at least, some portions of the
Internet to people who otherwise have no means to access the web.
This gives rise to a clash of values: between access to the Internet (in a limited form) and the maintenance of
neutrality in an atmosphere that is inherently unequal. This makes tailoring a solution to the problem a
particularly arduous process.
What net neutrality proponents say?
Net neutrality proponents arent resistant to the idea of a greater penetration of the Internet. But, their
apprehensions lie in companies resorting to what they believe is an unethical means to achieving, at
least in theory, a laudable end.
According to them, negating Net neutrality, in a bid to purportedly achieve greater access to the
Internet in the immediate future, could prove profoundly injurious in the long run.
Conclusion:
It is therefore, absolutely necessary that any debate that on the issue ought to include the tension between the
two apparently conflicting values the importance of maintaining a neutral Internet and the need to ensure
a greater access to the web across the country. Facebooks CEO Zuckerberg argues that these two values are
not fundamentally opposed to each other, but can and must coexist. He is possibly correct at a
theoretical level.
Source: The Hindu.
Topic Covered (Paper 2):
Important aspects of governance, transparency and accountability. (UPSC Civil Services Mains Syllabus)

Current Affairs 28 October-2015

08 October 2015
Paper 3 Topic: infrastructure

Banks killed road sector: official


Road Secretary Vijay Chhibber recently told that
banks have killed the road sector. According to
him, banks caused a bubble, failed to do due
diligence and lent more than what was warranted to
developers, and this has led to the high NonPerforming Assets (NPA) arising out of the road
sector.
His allegations on banks:
There was a bubble because the banks gave
developers money even before the land was
made available.
They allowed funding to happen to these
projects which were not yet ready for
construction.
They agreed to costs and TPCs [total project
cost] much higher than what NHAI had
assessed.
Ultimately, developers received loans for
amounts far higher than the governments
project cost estimates.
He also calls it a scam. Why?
Banks have not done proper diligence before
releasing so much money. Here, developers do not
have any problem. They have taken their money out
of the projects, but the banks are the ones in trouble.
And, taxpayers are asked to re-capitalise the banks.
Thus, he calls it a scam.
NPAs:
In August 2015, Union Finance Minister Arun
Jaitley said the road sector was responsible for the
second highest amount of NPAs, after the steel
sector.
A recent Crisil report said almost half of the road
projects, being constructed under the build, operate,
transfer with a sanctioned debt of Rs. 45,900 crore,
are at high risk of not being completed.
Other allegations:
The developers are entitled to a higher TPC
due to delays, but they usually take
advantage of it. Even if there is a delay, at
best, there will be a 10% increase or a 20%
increase in overall cost. But, in a portfolio of
70 projects, most of the projects see more
than 70% increase.
NHAI is a professional road construction
company. It had assessed a project at Rs.
1,000 crore, but the developers said the total
cost was Rs. 2,000 crore.
Sources: The Hindu.

Paper 2 Topic: Issues relating to


development and management of Social
Sector/Services relating to Health,
Education, Human Resources.

Dengue alert could have helped save lives:


experts
Public health experts feel that effective risk
communication strategy by public health authorities
to warn and alert people in advance of impending
outbreak of diseases and protective measures to be
taken could have prevented avoidable deaths such as
those caused by dengue in different parts of the
country.
Risk communication is an essential public health
function to disseminate information about outbreaks,
epidemics and upcoming emergencies to people.
Practices in other countries:
Risk communication has evolved as a sophisticated
tool and deployed effectively in developed nations
for people to take protective measures during natural
disasters, disease outbreaks and food-borne illnesses,
among others.
In US
Here, colour-coded risks, developed by the
US Federal Emergency Management Agency
(FEMA) with red indicating the highest
threat, are used. Recently, after hurricane
Katrina claimed over 1,200 lives and
damaged property estimated at nearly $ 108
billion, FEMA disseminated information
through cooperation and collaboration with
State and local public health authorities.
Similarly, US-based Centers for Disease
Control and Prevention (CDC) have
dedicated teams of epidemiologists to scour
their passive surveillance system to identify
any disease outbreaks. The CDCs Crisis and
Emergency Risk Communication (CERC),
which was formed after the 9/11 attacks and
anthrax threats, has dedicated personnel who
draw lessons from public health emergencies
to improve and strengthen communication.
In UK
In the United Kingdom, the Cabinet provides
specific guidelines on communicating risks
to their population.
In India:
Although the National Vector Borne Disease Control
Programme in India has a long-term action plan
document to prevent and control dengue and
chikungunya with the States expected to prepare a
contingency plan, very little communication has

Current Affairs 29 October-2015

been in the public domain. Despite being inundated


with information, the public health sector lacks the
wherewithal to communicate appropriately.
Need for India:
With large population in India susceptible to
infectious disease outbreaks, it is important
that any information about potential
outbreaks or diseases be communicated
appropriately and repeatedly to the public.
Year after year, dengue cases are seen
popping up in India exactly for the same
reasonslack of awareness about daytime
mosquito bites, lack of knowledge about
signs and symptoms and lack of resources on
seeking help in a timely manner. Hence,
Information about dengue and measures for
protection should be issued at the start of the
monsoon season with repeated reminders.
Sources: the hindu.
Paper 2 Topic: Government policies and
interventions for development in various
sectors and issues arising out of their
design and implementation.

Govt. cant punish us for lack of


education
Women candidates affected by Haryanas new poll
law imposing minimum educational qualifications to
contest panchayat elections recently told the
Supreme Court that people do not choose to be
illiterate.
They say it is the States failure to have not
provided them with education and hence,
they should not be punished for the States
failure.
According to them, India is a country where
primary and secondary education is hardly
available.
Background:
This was the maiden hearing following the
Haryana governments decision to freeze its
panchayat elections scheduled in October
after the Supreme Court refused to stop
questioning the constitutionality of imposing
minimum educational qualifications on
candidates aspiring to be part of grassroots
democracy.
Many candidates have been affected by the
Haryana Panchayati Raj (Amendment) Act,
2015.
The court had stayed the new amendments in
the States panchayat poll law, seeing it as

prima facie a move affecting poor illiterate


people from contesting elections.
The State government had gone ahead with
the changes despite the Punjab and Haryana
High Court staying the relevant ordinance in
August 2015.
About the Haryana Panchayati Raj (Amendment)
Act, 2015:
According to the amendments, general
category candidates require a minimum
qualification of Class X pass, men contesting
in the Scheduled Caste category and women
in the general category need to be Class VIII
pass, while women in the Scheduled Caste
category need to be Class V pass to be
eligible.
The amendments also require that candidates
should not have any dues in co-operative
banks, electricity bills should be paid up and
there should be a functional toilet at home.
The petition also says that 83% of Dalit women and
71% women in general and 56% males would be
excluded from contesting the panchayat polls by this
law, affecting fundamental rights of the candidates.
Sources: the hindu.
Paper 2 Topic: Important International
institutions, agencies and fora, their
structure, mandate.

Hoesung Lee to head U.N. climate panel


Hoesung Lee, 69, a Korean professor of economics
of climate change, energy and sustainable
development, has been elected as head of the Intergovernmental Panel for Climate Change. He
succeeds R.K. Pachauri of India.
About Hoesung lee:

Lee, until now one of the vice-chairs of the

Current Affairs 30 October-2015

IPCC, served as executive member of the


Korean Academy of Environmental Sciences;
a member of the Asia Development Bank
Presidents advisory board; a council member
of the Global Green Growth Institute; and an
editorial board member of the U.K.-based
Climate Policy.
He was the founding president of the Korea
Energy Economic Institute and the former
president of the International Association for
Energy Economics. He has been serving the
IPCC in various capacities, including as
Working Group III Co-Chair since its Second
Assessment Report of 1992.
About IPCC:
The Intergovernmental Panel on Climate
Change (IPCC) is a scientific
intergovernmental body under the auspices of
the United Nations, set up at the request of
member governments.
It was first established in 1988 by two United
Nations organizations, the World
Meteorological Organization (WMO) and the
United Nations Environment Programme
(UNEP), and later endorsed by the United
Nations General Assembly.
Membership of the IPCC is open to all
members of the WMO and UNEP.
The IPCC produces reports that support the
United Nations Framework Convention on
Climate Change (UNFCCC), which is the
main international treaty on climate change.
The IPCC does not carry out its own original

research, nor does it do the work of


monitoring climate or related phenomena
itself. The IPCC bases its assessment on the
published literature, which includes peerreviewed and non-peer-reviewed sources.
Thousands of scientists and other experts
contribute, on a voluntary basis, to writing
and reviewing reports, which are then
reviewed by governments.
IPCC reports contain a Summary for
Policymakers, which is subject to line-byline approval by delegates from all
participating governments.
The IPCC provides an internationally
accepted authority on climate change,
producing reports which have the agreement
of leading climate scientists and the
consensus of participating governments.
The 2007 Nobel Peace Prize was shared, in
two equal parts, between the IPCC and Al
Gore.
sources: the hindu, wiki.

Lindahl, of the Francis Crick Institute in


London, was honoured for his discoveries on
base excision repair the cellular mechanism
that repairs damaged DNA during the cell
cycle.
Modrich, of the Howard Hughes Medical
Institute and Duke University School of
Medicine, was recognised for showing how

cells correct errors that occur when DNA is


replicated during cell division.
Sancar, of the University of North Carolina,
Chapel Hill, was cited for mapping the
mechanism cells use to repair ultraviolet
damage to DNA.
Their systematic work has made a decisive
contribution to the understanding of how the living

Paper 3 Topic: Science and Technologydevelopments and their applications.

Chemistry Nobel for mapping how cells


repair damaged DNA
Tomas Lindahl, Paul L. Modrich and Aziz Sancar
have jointly won the 2015 Nobel Prize in Chemistry
for having mapped and explained how the cell
repairs its DNA and safeguards its genetic
information.
About the awardees:

Current Affairs 31 October-2015

cell functions, as well as providing knowledge about


the molecular causes of several hereditary diseases
and about mechanisms behind both cancer
development and aging.
sources: the hindu.
Paper 2 Topic: Effect of policies and
politics of developed and developing

The waterways of Xijiang River that flows


through Nanning city lead to the Pearl River
and the South China Sea.
Nanning would enable a cargo ship of 2,000
tonnes to head for the bustling commercial
cities of Guangzhou, Hong Kong and Macao.
By 2020, Nanning ports capacity is expected
to rise to 22.83 million tonnes.
The Guangxi province, of which Nanning is
the capital, is also the gateway to a large
landlocked space.
Its prized geographic location is making the
city the fulcrum of Chinas access to the most
dynamic zones of Southeast Asia.
MSR:
The MSR(maritime silk road) is part of a string of
Silk Road initiatives that the Chinese are
undertaking that includes the Bangladesh-ChinaIndia-Myanmar (BCIM) corridor, which aspire to
establish integral economic linkages between South
and Southeast Asia.
The Silk Road project is an initiative by
China to resurrect the ancient maritime Silk

countries on Indias interests.

Nanning emerges as a pillar of the


Maritime Silk Road
China has made Nanning one of the focal points of
the proposed Maritime Silk Road, leveraging the
southern citys natural connectivity linkages with
Southeast Asia and growth hubs of Guangzhou,
Hong Kong and Macao.

Road. It is perceived to be an attempt by


China to ameliorate relations with South and
Southeast Asia.
The new initiative is a pet project of
President Xi Jinping for connecting Asia with
Europe along a land corridor, with China as
its hub.
Under the new Silk Route, the Chinese want to
open up the transportation channel from the
Pacific to the Baltic Sea, from which would
radiate rail and road routes, which would also
connect with East Asia, West Asia, and South
Asia.
The Silk Road strategys ambitious vision
aligns with Beijings goals much more
closely than the Trans-Pacific Partnership
(TPP), which is a reflection of the U.S.
international trade model writ large.
The Silk Road strategy aims to facilitate
large-scale infrastructure construction,
energy sale and transport, and relocation of
manufacturing industries.
This initiative aspires to deepen linkages

Current Affairs 32 October-2015

between China and its neighbours via trade,


investment, energy, infrastructure, and
internationalization of Chinas currency, the

1. Question

Which of the following statements about


Sustainable Development Goals (SDGs) is/are
incorrect?
1. It is a set of 17 goals which will cover a
period of 17 years
2. These are applicable to all members of
the United Nations and not only the

renminbi.
sources: the hindu.

developing
Select the correct answer using the codes given
below:
a) 1 Only
b) 2 Only
c) Both 1 and 2
d) Neither 1 or 2
Ans: a.

Current Affairs 33 October-2015

2. Question

Which of the following statements about the


Razakars is/are incorrect?
1. They resisted the integration
of Hyderabad State into the Dominion of
India
2. They were once led by Qasim Razvi
3. This regiment captured areas around
Lahore in the Indo-Pakistani War of 1965
Select the correct answer using the codes given
below.
a) 1 Only
b) 1 and 3 Only
c) 3 Only
d) 2 and 3 Only
Ans: c.
http://pib.nic.in/newsite/PrintRelease.aspx?
relid=127001

https://en.wikipedia.org/wiki/Razakars_(Hyderabad)
3. Question
Consider the following list of states:
1. Travancore
2. Jodhpur
3. Junagadh
4. Hyderabad
5. Bhopal
Which of the above states acceded to the Indian
Union before Independence?
a) 1, 2 and 3 only
b) 3, 4 and 5 only
c) 1, 2 and 5 Only
d) 2, 3 and 5 Only
Ans: c.
Junagadh, Hyderabad and Kashmir were the only
states which acceded to the Indian Union after
Independence.
India after Gandhi, Chapter 3: Apples in the basket

http://pib.nic.in/newsite/PrintRelease.aspx?
relid=127001

https://en.wikipedia.org/wiki/Political_integration_o
f_India#The_accession_process
4. Question
Consider the following statements about stupas:
1. They are unique to Buddhism
2. The stupa at Sanchi was built during the
time of Ashoka
3. Royal patronage was the sole form of
patronage for their construction in ancient
India
Which of the above statements is/are incorrect?

a) 1 only
b) 1 and 2 Only
c) 3 Only
d) 1 and 3 only
Ans: d.
Stupa, vihara and chaitya are part of Buddhist and
Jaina monastic complexes but the largest number
belongs to the Buddhist religion. One of the best
examples of the structure of a stupa in the third
century BCE is at Bairat in Rajasthan. It is a very
grand stupa having a circular mound with a
circumambulatory path. The great stupa at Sanchi
(which will be discussed later) was built with bricks
during the time of Ashoka and later it was covered
with stone and many new additions were made.
Subsequently many such stupas were constructed
which shows the popularity of Buddhism.
From the second century BCE onwards, we get
many inscriptional evidences mentioning donors
and, at times, their profession. The pattern of
patronage has been a very collective one and there
are very few examples of royal patronage. Patrons
range from lay devotees to gahapatis and kings.
Donations by the guilds are also mentioned at
several sites.
Page 21, Fine Art (NCERT Class XI)

http://www.thehindu.com/news/international/ashokastupa-restored-in-china/article7663044.ece
5. Question
Consider the following statements:
1. The Chakmas are also known as the
Daingnet people
2. Hajongs have been notified as a
Scheduled Tribe (ST) in some Indian
states
3. Chakmas have also been notified as a ST
in some Indian states
Which of the above statements are correct?
a) 1 and 2 only
b) 2 and 3 only
c) 1 and 3 Only
d) All the above
Ans: d.
http://tribal.nic.in/content/list%20of%20scheduled
%20tribes%20in%20India.aspx

http://www.northeasttoday.in/sc-grant-citizenshiprights-to-chakmas-hajongs-in-arunachal/
6. Question
Which of the following life-threatening diseases
does the Pentavalent vaccine provide
immunization from?

Current Affairs 34 October-2015

1. DPT (Diptheria, Pertusis and Tetanus)


2. DPT (Diptheria, Polio and Tetanus)
3. Hepatitis B
4. Haemophilus Influenza Type B
5. Measles
Select the correct answer using the codes given
below:
a) 1 and 4 Only
b) 3 and 4 Only

c) 1, 2 and 4 Only
d) 1,3 and 4 Only
Ans: d.
http://www.searo.who.int/india/topics/routine_immu
nization/Pentavalent_vaccine_Guide_for_HWs_with
_answers_to_FAQs.pdf

http://www.thehindu.com/news/national/kerala/m
alappuram-children-under-observation-fordiphtheria/article7663971.ece

Insights Into Editorial: Climate Goals on Target


09 October 2015
There is a criticism that Indias use of coal for electricity generation is projected to double by 2030. But, if
seen in the perspective of International Energy Agency, India will use less coal for electricity generation
than the US even in 2040.
Indias efforts towards combating climate change:
India is the third largest economy in terms of PPP. In the recent times, it is offering concrete deliverables.
In 2009, it had promised an emissions intensity reduction of 20-25% by 2020, from 2005 levels. It
has achieved an emissions intensity reduction of 18.6%. India now aims for 33 to 35%
India has decided to have 40% of the total installed power capacity in 2030 based on non-fossil
fuel-based Currently, renewable energy, nuclear energy and hydropower together contribute 30% of
the overall installed capacity. With power production expected to triple, this will amount to 320 GW
of non-fossil fuel capacity.
India is also seeking investments of U.S. $100 billion over seven years to boost the domestic solar
energy capacity by 33 times to 1,00,000 megawatts by 2022.
Nuclear capacity is also expected to increase more than ten times to 63GW and hydro power is
expected to double to 84 GW, with land availability being the key concern.
Additional carbon sinks of 2.5 to 3 billion tonnes of carbon dioxide equivalent will be created by
2030. An afforestation fund has also been set up. It is meant to encourage the setting up of projects on
forest land.
The government has also planned to develop a 1,40,000-km tree-line along both sides of the national
highways.
Governmental efforts:
Prime Minister Modi has made climate change an integral part of national transformation. This geopolitical
shift is also providing India the confidence to shape the new rules. The prime minister has also called for
countries to take into account the levels of development of various countries and allow them the
developmental space so that they can also aspire to become middle and developed countries.
Indias INDCs:
The Intended Nationally Determined Contributions state that Indias objective in Paris in December 2015 is
to establish an effective, cooperative and equitable global architecture.
Three key elements of this framework are:
Promoting sustainable production processes and sustainable lifestyles across the globe.
Creation of a regime where facilitative technology transfer replaces an exploitative market-driven
mechanism.
A common understanding of universal progress.
Comparative analysis:
It is being said that the lifestyle in India is already more austere than it is in China.
Chinas industrial production is eight times, consumption of primary energy five times, metals eleven
times, GDP four times and per capita emissions two times higher.
By 2030, per capita energy use and emissions in China and the U.S. are expected to around 10-12
Current Affairs 35 October-2015

tonnes of carbon dioxide per capita whereas Indias per capita emissions will be just one-quarter of
this level.
Indias strategy:
The information technology revolution spearheaded by India is the first global transformation not
based on increasing use of energy.
India has also taken the initiative to host a meeting of 107 sunshine countries before the Paris
summit, to forge a common platform on sharing research and looking for common financial
solutions.
India has also proposed to set up a global virtual centre for clean energy research and
development and fund collaborative projects.
Experts say that India should integrate its Smart Cities campaign into a plan for low carbon development of
cities.
Problem with the current methodology:
The current emphasis on emission reductions focuses on symptoms rather than causes and solutions. Faced
with global ecological limits, focus has to shift from environmental risk management to economic
growth within ecological limits.
Anthropogenic impact:
It has been estimated that currently three basic human needs housing, food, mobility directly
account for 80% of resource use, 40% of energy demand and 36% of carbon dioxide emissions.
How can the situation be improved?
Experts say that nearly two-fifth of the cumulative emission reductions required by 2050 could come
from efficiency improvements.
Key systems such as the transport, energy, housing and food systems should be transformed.
Focus should be on long term remedies.
Periodic reviews of national contributions should also be undertaken.

09 October 2015
Paper 2 Topic: Role of civil services in a
democracy.

Indian Skill Development Service


The Union cabinet recently gave its nod for the
formations of Indian Skill Development Service
(ISDS).
Details:
Indian Skill Development Service (ISDS)
will be a Group A service of the technical
cadre of the Ministry of Skill Development &
Entrepreneurship (MSD&E).
ISDS was earlier called the Directorate
General of Employment & Training which
was under the aegis of Ministry of Labour
Employment.
Why was it created?
To improve skill development administration
in the country.
To enhance the capacity and efficiency of the
organization.
Selection:
The cadre will be selected through the Union Public

Source: The Hindu


Topic: Environmental Pollution (Paper 3)
Service Commission (UPSC) and have the same
ranks and profile as other Group A services such as
the Indian Revenue Service, Indian Audit and
Accounts Service and Indian Railway Service.
Their roles:
The new cadre of officers will run the skill
development and entrepreneurship ministry,
help implement various skill and
apprenticeship schemes conducted by the
ministry as well as those by the Directorate
General of Training (DGT).
The new officers will help formulate skill
policies, devise a road map for improving the
apprenticeship system, revamping ITIs and
also help revamp course work for various
schemes.
Was it necessary?
India aims to skill some 500 million people by 2022
in both organized and unorganized sectors to
improve the efficiency of the work force, provide
job-ready human resources to industries and
ultimately improve the competitiveness of a young
demography considered crucial for the economic
growth of India. To achieve these objectives, it is
necessary to have such a dedicated service.

Current Affairs 36 October-2015

sources: pib.
Paper 2 Topic: Government policies and
interventions for development in various
sectors.

Neeranchal programme gets Cabinet nod


The World Bank-assisted National Watershed
Management Project or Neeranchal has received
the approval for implementation by the Union
Cabinet.
Neeranchal:
Neeranchal is designed to further strengthen and
provide technical assistance to the Watershed
Component of PMKSY, in particular and all
components of PMKSY, in general and to enhance
its delivery capacity.
Aim: The project aims to fulfil the watershed
component of the Pradhan Mantri Krishi Sinchai
Yojana (PMSKY) to reduce surface runoff of
rainwater, increase groundwater levels and better
water availability in rain-fed areas.
The cost of the project is estimated at Rs.
2,142.30 crore of which the Centre will be
pitching in with Rs. 889 crore while Rs. 182
crore will be provided by the respective State
Governments. The remaining 50% of the
project cost will be financed by a World
Bank loan.
It will be implemented across nine States
Andhra Pradesh, Telangana, Madhya
Pradesh, Maharashtra, Gujarat, Odisha,
Chhattisgarh, Jharkhand and Rajasthan.
Neeranchal is primarily designed to address the
following concerns:
bring about institutional changes in
watershed and rainfed agricultural
management practices in India,
build systems that ensure watershed
programmes and rainfed irrigation
management practices are better focussed,
and more coordinated, and have quantifiable
results,
devise strategies for the sustainability
of improved watershed. management
practices in programme areas, even after the
withdrawal of project support,
through the watershed plus approach,
support improved equity, livelihoods, and
incomes through forward linkages, on a
platform of inclusiveness and local
participation.
About the Pradhan Mantri Krishi Sinchai Yojana
(PMSKY):

It is a central scheme that aims at providing


irrigation facilities to every village in the country by
converging ongoing irrigation schemes implemented
by various ministries.
Under the project, a dynamic annual fund
allocation methodology, which mandates
states to allot more funds to irrigation sectors
for becoming eligible to access funds under
this scheme, is being considered.
The Scheme intends to focus on end-to-end
solution in irrigation supply chain by
implementing the new programme in a
project mode with decentralised state-level
planning and execution.
The programme architecture of PMKSY aims
at a decentralized State level planning and
execution structure, in order to allow States
to draw up a District Irrigation Plan (DIP)
and a State Irrigation Plan (SIP).
Implementation:
The programme will be supervised and
monitored at the national level by an InterMinisterial National Steering Committee
(NSC) under the Chairmanship of the Prime
Minister with Union Ministers of all
concerned Ministries.
A National Executive Committee (NEC) will
be constituted under the Chairmanship of the
Vice Chairman, NITI Aayog to oversee
programme implementation, allocation of
resources, inter ministerial coordination,
monitoring and performance assessment,
addressing administrative issues etc.
At the state level the scheme will be
administered by a State Level Sanctioning
Committee (SLSC) to be Chaired by the
Chief Secretary of the respective States. The
committee will have all authority to sanction
the project and also monitor the progress of
the scheme.
At the district level there will be a district
level implementation committee for ensuring
last mile coordination at the field level.
The state agriculture department would be
the nodal agency for implementation of
PMKSY projects.
Eligibility and funds:
A state will become eligible to access
PMKSY funds only if it has prepared the
district irrigation plans and state irrigation
plans and sustained an increasing expenditure
trend in irrigation sector in state plan.

Current Affairs 37 October-2015

The programme has an outlay of Rs. 50,000


crore over a period of five years (2015-16 to
2019-20). The allocation for the current
financial year is Rs. 5300 crore.
PMKSY funds would be given to states as
75% grant by the central government and the
remaining 25% share is to be borne by the
state government. For northeastern region
and hilly states, the funding pattern would be
90:10.
Sources: PIB.
Paper 3 Topic: Awareness in the fields
of IT, Space.

technology which has been developed by


Raytheon.
India is the fourth country to offer space-based
satellite navigation services to the aviation sector.
sources: the hindu, pib.
topic: General awareness

Journalist wins literature Nobel


Svetlana Alexievich, a Belarusian journalist and
prose writer, has won the 2015 Nobel Prize in
Literature for her polyphonic writings, courage and a
monument to suffering.
About Svetlana alexievich:

AAI readying sops to push GAGAN


Civil Aviation Ministry is planning to aggressively
promote GAGAN across all transportation sectors,
such as Railways, maritime and road transport.
Airport Authority of India is also considering
incentives and programmes to accelerate
GAGANs use among commercial airlines
and general aviation aircraft.
About GAGAN:

GAGAN was developed by the Indian Space


Research Organisation (ISRO) and the
Airports Authority of India (AAI) at a cost of
Rs. 774 crore, over 15 years.
GAGAN will provide augmentation service
for the GPS over the country, the Bay of
Bengal, South East Asia and Middle East and
up to Africa.
Some of its benefits are improved efficiency,
direct routes, increased fuel savings,
approach with vertical guidance at runways,
significant cost savings because of the
withdrawal of ground aids and reduced
workload of flight crew and air traffic
controllers.
Gagan works by augmenting and relaying
data from GPS satellites with the help of two
augmentation satellites and 15 earth-based
reference stations.
The system utilises the satellite-based wide
area augmentation system (SBAS)

Alexievich, 67, is the 14th woman to win the


literature prize.
Alexievichs works often blend literature and
journalism. She is best known for giving
voice to women and men who lived through
World War II, the Soviet occupation of
Afghanistan that lasted from 1979 to 1989,
and the Chernobyl nuclear disaster of 1986.
In the United States, Alexievich is best
known for the oral history Voices From
Chernobyl.
The Nobel in literature, one of the most prestigious
prizes in the literary world, is given in recognition of
a writers entire body of work rather than a single
title.
sources: the hindu.
Paper 3 Topic: Science and Technologydevelopments and their applications and
effects in everyday life.

IT-based malaria control initiative from


October 17
In a first-of-its-kind initiative, the Mangaluru City
Corporation in Karnataka will officially unveil its
IT-based initiative to track and control malaria cases
on October 17.
Details:
The initiative would be through software

Current Affairs 38 October-2015

developed under a public-private partnership.


The software has a web-based system for
clinics and laboratories, a mobile-based
system for field workers or multi-purpose
health workers, and another web-based
system for the city corporation.
The software connects the field workers
(who visit households and report sources of
malaria and malaria cases), clinics and
laboratories (which are supposed to report to
the civic body when a malaria case is
diagnosed) and the officials in city
corporation.
The software has been developed by
following the guidelines of the National
Vector Borne Disease Control Programme
(NVBDCP).
Clinics, laboratories and hospitals would
have to compulsorily feed data on malaria
cases reported following blood tests. Multipurpose health workers would have access to
this data and they, in turn, will visit houses
and areas from where the cases have been
reported. They will initiate steps for
controlling the disease both among patients
and in breeding places. A file would be
opened for each case and it would be
monitored by senior officials till each file is
closed. In addition, the workers would
conduct field visits on their own.
The software would now give city planners,
including officials and elected representatives, a
birds eye view of instances of outbreak of malaria
and follow-up action on the ground.
Present situation:
At present, the reports from field workers and clinics

and laboratories reach the civic body a week or a


month after the case is diagnosed. No purpose would
be served if there is a case of malaria reported after
30 days. The software will help bridge this gap and
also in reporting malaria within 48 hours.
sources: the hindu.
1. Question

The Aegean Sea lies in-between


a) Italy and Croatia
b) Italy and Greece
c) Turkey and Greece
d) Turkey and Egypt
Ans: c.
2. Question

Consider the following statements about the


Indian Evidence Act:
1. It was framed during the time of British
Raj
2. The same law exists today, with an
amendment having been passed only once
due to the rapid increase in ownership of
mobile phones
Which of the above statements are true?
a) 1 Only
b) 2 Only
c) Both 1 and 2
d) Neither 1 or 2
Ans: a.
http://www.thehindu.com/news/national/evidenceact-likely-to-be-amended/article7665770.ece
3. Question
Lake Chad is surrounded by which of the
following countries?
1. Chad
2. Nigeria
3. Niger
4. Cameroon
Select the correct answer
a) 1 only
b) 1 and 2 only
c) 1, 2 and 3 Only
d) 1, 2, 3 and 4
Ans: d.
http://www.bbc.com/news/world-africa-34289474

https://en.wikipedia.org/wiki/Lake_Chad
4. Question
Arrange the following military units in order of
increasing number of military men among them:
1. Company
2. Division
3. Battalion
4. Brigade

Current Affairs 39 October-2015

Select the correct answer using the codes given


below:
a) 1-2-3-4
b) 1-3-4-2
c) 3-2-1-4
d) 3-1-2-4
Ans: b.
With respect to the 1965 Indo-Pak War, the
armed forces had begun a command restructuring,
force expansion and doctrinal shift process which
had given its rank and file new confidence. Indian
commanders were new to handling brigade and
division-sized forces in battles. Indian commanders
thus far had experience as battalion commanders in
1948 in Jammu and Kashmir and in 1962
http://www.thehindu.com/opinion/lead/1965-a-warwith-no-winners/article7665654.ece

https://en.wikipedia.org/wiki/Military_organization#
Armies
5. Question
Which of the following schemes are administered
by the Ministry of Textiles?
1. North East Region Textile Promotional
Scheme
2. West Himalayan Wool Development
Scheme
3. Technology Upgradation Fund Scheme

Select the correct answer using the codes given


below:
a) 1 and 2 only
b) 2 and 3 only
c) 1 and 3 Only
d) All the above
Ans: c.
http://www.thehindu.com/business/Industry/concern
-over-fall-in-textile-exports/article7665650.ece

http://texmin.nic.in/policy/policy_scheme.htm
6. Question
Consider the following statements about pensions in
India:
1. The Government of India treats it as
salary, and hence is taxed
2. Dearness allowance on basic pension is
given to mitigate the impact of inflation,
and is thus not taxed
Which of the above statements is/are correct?
a) 1 Only
b) 2 Only
c) Both 1 and 2
d) Neither 1 or 2
Ans: a.
http://www.thehindu.com/business/Economy/a-ratecut-will-add-to-the-misery/article7695086.ece

Insights Into Editorials: Two Sides of Trans-Pacific Partnership pact


10 October 2015
The Trans-Pacific Partnership pact reached recently between the United States and 11 Pacific Rim nations
including Canada and Japan, has raised both hopes and concerns.
What is Trans-Pacific Partnership pact?
It is a trade pact that is intended to cut trade barriers and establish common standards for 12 countries.
What it does?
It would set new terms for trade and business investment among the United States and 11 other
Pacific Rim nations.
It would phase out thousands of import tariffs as well as other barriers to international trade.
It also would establish uniform rules on corporations intellectual property, open the Internet even in
communist Vietnam and crack down on wildlife trafficking and environmental abuses.
Its significance:
This is the largest trade pact in 20 years.
The agreement covers 40% of the worlds economy.
It is seen as a means to address a number of festering issues that have become stumbling blocks as
global trade has soared, including e-commerce, financial services and cross-border Internet
communications.
What its supporters say?
The pact would boost growth in the U.S. as well as the Asian economies.
It would be a boon for all the nations involved.
It would unlock opportunities and address vital 21st-century issues within the global economy.

Current Affairs 40 October-2015

What its opponents say?


Current Affairs 41 October-2015

Critics in the US say it would only help American companies send jobs abroad. Some people call it a
trade disaster.
Critics in other countries say it would benefit large corporations, particularly American big pharma,
with the common people at the receiving end.

Some people say it would reduce access to generic medicines in developing countries.
Internet freedom campaigners see it as a big threat.
Why is the US interested in this deal?
It is seen as a way to bind Pacific trading partners closer to the United States while raising a
challenge to Asias rising power, China.
Traditionally, the U.S. has tried to isolate its enemies and integrate allies with its own worldview.
With china it couldnt do either.
China is now the worlds second largest economy, which has invested trillions of dollars in U.S.
treasury bonds. Hence, isolating such an economy is next to impossible.
Now, with China emerging as an economic powerhouse with new institutions such as the Asian
Infrastructure Investment Bank in place, the U.S. is trying to form a grand alliance that would shore
up its influence in Asia.
Economists such as Joseph Stiglitz have pointed out that the TPP would hardly meet either its declared
Current Affairs 42 October-2015

commercial goals or its undeclared strategic ambitions, and could turn counterproductive.
agriculture sector would increasingly reap the
benefits of space technology in the coming years
Paper 3 Topic: conservation.
through various upcoming projects, including the
Indo-US NISAR (NASA-ISRO Synthetic Aperture
West Bengal to get Indias first dolphin
Radar) satellite.
reserve
About NISAR:
Indias first community reserve to protect the
The Nasa-Isro Synthetic Aperture Radar
endangered Gangetic river dolphins will come up
(NISAR) mission is a joint project between
in West Bengal. This decision was taken at the
NASA and ISRO to co-develop and launch a
recently held State Wildlife Board meeting in WB.
dual frequency synthetic aperture radar
The reserve will be set up in the Hooghly
satellite.
river.
The satellite will be the first radar imaging
The methodology to develop the community
satellite to use dual frequency and it is
reserve is being chalked out by a separate
planned to be used for remote sensing to
committee. The committee will take a
observe and understand natural processes of
decision based on inputs from all
the Earth.
stakeholders since its a community reserve.

It is slated to be launched in 2020-21.


About Gangetic Dolphin:
NISAR would provide information about a
place more frequently than older satellites
orbiting the Earth at present.
Among the objectives of NISAR are
estimation of soil moisture, agriculture and
forest biomass.
It is also designed to observe and take
measurements of some of the planets most
The Ganges River dolphin, or susu, inhabits
complex processes, including ecosystem
the Ganges-Brahmaputra-Meghna and
disturbances, ice-sheet collapse, and natural
Karnaphuli-Sangu river systems of Nepal,
hazards such as earthquakes, tsunamis,
India, and Bangladesh. It is a freshwater
volcanoes and landslides.
Once found in thousands, there are fewer
sources: the hindu, wiki, isro.
than 2,000 Gangetic dolphins left in the
country in the entire distribution range along
Paper 2 Topic: Bilateral, regional and
the Ganga and Brahamaputra river system.
global groupings and agreements
It was declared as the National Aquatic
involving India and/or affecting Indias
Animal in 2010.
interests.
One of the main threats to the species is loss
BRICS to establish liberal visa regime
of habitat due in large part to the creation of
among member countries
dams and irrigation projects. It is also
The BRICS nations have decided to allow free
threatened by removal of river water and
movement of skilled professionals among member
siltation arising from deforestation, pollution
countries by setting up a liberalised visa regime.
and entanglement in fisheries nets.
This joint declaration was issued at the first-ever
This species is also referred to as the blind
ministerial meeting on migration held recently in
dolphin.
Sochi, Russia.
It has been classified as endangered by the Other outcomes of the meeting:
IUCN.
The BRICS countries have joined hands to
sources: the hindu, wiki.
combat and prevent organised human
trafficking and migrant smuggling, and have
Paper 3 Topic: Agriculture, space.
promised to strengthen dialogue and
Farm sector sure to gain from space tech
cooperation among the member countries.
Indian Space Research Organisation Chairman A.S.
They have also resolved to combat and
Kiran Kumar has indicated that the countrys
prevent transnational organised crime.

10 October 2015

Current Affairs 43 October-2015

The nations have affirmed their interest in


exchanging of views and sharing of
experience on migration issues.
They have accepted the inter-relationship
between transnational migration and
development; and the need to deal with the
opportunities and challenges that migration
presents and take advantage of its positive
impacts.
The meeting was also attended by
representatives of Shanghai Cooperation
Council, the Commonwealth of Independent
States and other international bodies working
on migration.
sources: the hindu.
Paper 2 Topic: Bilateral, regional and
global groupings and agreements
involving India and/or affecting Indias
interests.

International Code of Conduct against Ballistic


Missile Proliferation (ICOC), also known as the
Hague Code of Conduct, which calls for restraint
and care in the proliferation of ballistic missile
systems capable of delivering weapons of mass
destruction, and has 119 members, thus working
parallel to the MTCR with less specific restrictions
but with a greater membership.
sources: the hindu, wiki.
Paper 1 Topic: World History and
General awareness.

Tunisian mediators win Nobel Peace Prize


The 2015 Nobel Peace Prize has been awarded to
Tunisias National Dialogue Quartet. It has been
selected for its decisive contribution to the building
of a pluralistic democracy in Tunisia in the wake of
the Jasmine Revolution of 2011.
The quartet comprises four organisations:

No decision at MTCR meeting


The meeting of the Missile Technology Control
Regime (MCTR) concluded recently in the
Netherlands. But, the decision on Indias
membership is still pending.
India has already said that its inclusion would
further strengthen global non- proliferation
objectives.
India wants to gain entry into MCTR
favouring non-proliferation of unmanned
delivery systems capable of delivering
weapons of mass destruction.
India has an application under submission
since June 2015 to be a member of the
Missile Technology Control Regime
(MTCR)..
While countries like the United States and
Sweden have been backing Indias bid, other
nations like Italy and China are not too
enthusiastic about granting it membership.
About MCTR: quick facts
The Missile Technology Control Regime
(MTCR) is an informal and voluntary
partnership between 34 countries to prevent
the proliferation of missile and unmanned
aerial vehicle technology capable of carrying
a 500 kg payload for at least 300 km.
It was established in April 1987 by Canada,
France, Germany, Italy, Japan, Great Britain,
and the United States.
China is not a member of the MTCR.
In 2002, the MTCR was supplemented by the

1. the Tunisian General Labour Union.


2. the Tunisian Confederation of Industry, Trade
and Handicrafts.
3. the Tunisian Human Rights League.
4. and the Tunisian Order of Lawyers.
But the Norwegian Nobel Committee emphasized
that the prize is awarded to this quartet, not to the
four individual organizations as such.
About National Dialogue Quartet:
The quartet has played a key role in
mediating between the different parties in the
countrys post-Arab Spring government.
It was created in 2013, two years after the
revolution, when security in the country was
threatened following the assassination of two
key politicians and deadly clashes between
Islamists and secular parts of society.
The Arab Spring originated in Tunisia in
2010 and 2011, but it quickly spread to other
countries in North African and the Middle

Current Affairs 44 October-2015

East. In many of these countries, the struggle


for democracy and human rights has come to
a standstill or suffered setbacks. Tunisia,
however, has seen a democratic transition
based on vibrant civil society, with demands
for respect of basic human rights.
Tunisias uprising was the first and most
successful of the Arab Spring.
sources: the hindu, bbc, wiki.
Paper 2 Topic: Bilateral, regional and
global groupings and agreements
involving India and/or affecting Indias
interests.

Revision of Piracy High Risk Area (HRA)

Piracy off the Coast of Somalia (CGPCS) has


announced the revision of the limits of the piracy
High Risk Area (HRA).
With this, Indias west coast has been
excluded from piracy High Risk Area (HRA).
The decision will come into effect from
December 1.
Background:
Consequent to the spread of piracy to the
East Arabian Sea, the international shipping
industry had extended the eastern limit of
piracy HRA in June 2010 to 78oE longitude,
thereby including the west coast of India
within its ambit.
Why was india concerned about this?

European Union Chair of the Contact Group of

This extension had led to security concerns on


account of the presence of private security
personnel onboard merchant vessels transiting
the piracy HRA, and the presence of floating
armouries off the Indian coast.
The shipping industry also incurred
additional costs for insurance and
implementation of various recommendations
for transit through the piracy HRA.
The extended HRA also came near the Indian
coastline up to as close as about 35 nautical
miles from the baseline. This was an
unwarranted encroachment into Indias EEZ
(Exclusive Economic Zone).

What made this revision necessary?


Affirmative action and increased surveillance
contributed towards the decline of piracy
incidents in the East Arabian Sea. The last
reported piratical activity in the East Arabian
Sea was in March, 2012.
In addition to deployment of Indian Naval
ships in the Gulf of Aden since October 2008
for anti-piracy patrols, robust action by the
Indian Navy and Indian Coast Guard led to
the arrest of 120 pirates from four pirate
mother-ships between January-March 2011.
The absence of piracy in Indian maritime
zones led to India seeking a review with

Current Affairs 45 October-2015

support from many other countries.


This revision of the HRA boundary back to its
original state should thus greatly reduce the
insurance costs of Indian shipping companies. In
total, this could save the industry $25 million.
sources: the hindu, pib.
1. Question

Consider the following statements:


Assertion (A): Wild fluctuations and alleged
irregularities have remained untamed in the
commodities markets
Reasons (R): The Forward Markets Commission
was the regulator of commodities markets until
recently
Select the correct answer using the codes given
below:
a) A and R both are true, and R is the
correct explanation for A
b) A and R both are true, and R is the
NOT the correct explanation for A
c) A is correct, R is incorrect
d) A and R both are incorrect
Ans: b.
FMC has been regulating commodities markets
since 1953, but lack of powers has led to wild
fluctuations and alleged irregularities remaining
untamed in this market segment.
2. Question
Which among the following statements best
describe the New World Order?
a) As a conspiracy theory, it refers to the
emergence of a totalitarian world
government
b) It refers to a backdoor-agreement to be
struck by the permanent members of the
United Nations Security Council with the
G4
c) It refers to a proposed idea of an
alternative to the United Nations, by the
G4+1
d) It is the blueprint created by Western
nations involved in the wars/battles of
West Asia since early 2000s
Ans: a.
https://en.wikipedia.org/wiki/New_World_Order_(c
onspiracy_theory)

http://www.thehindu.com/news/international/911was-manipulated-so-that-west-could-blame-osamacorbyn/article7695008.ece
3. Question
Consider the following statements about the
Green Highways Policy which was recently

launched:
1. 1% of the total project cost of all
highways projects will be kept aside for
the highway plantation and its
maintenance
2. Its implementation can help bridge the
gap between notified forest cover and
National Forest Policy-envisaged forest
cover
3. This policy will generate employment
opportunities for people from rural areas
Which of the above statements is/are incorrect?
a) 1 and 2 only
b) 1 and 3 only
c) 3 Only
d) None of the above
Ans: d.
http://pib.nic.in/newsite/PrintRelease.aspx?
relid=128298
4. Question
Consider the following statements:
Assertion (A): India has been relatively resilient
in coping with the sluggish global demand
(which owes a great extent to the weakness in the
Eurozone)
Reason (R): There has been significant market
diversification in Indias trade in recent years
Select the correct answer using the codes given
below:
a) A and R both are true, and R is the
correct explanation for A
b) A and R both are true, and R is the
NOT the correct explanation for A
c) A is correct, R is incorrect
d) A and R both are incorrect
Ans: a.
Page 15, Volume 2, Economic Survey 2014-15
http://www.thehindu.com/business/south-asiaimportant-for-india-geopolitically-but-noteconomically-cea/article7702915.ece
5. Question
Which of the following reasons is/are incorrect
and/or cannot be considered as valid with respect
to having a trade bloc in South Asia, similar to
the North Atlantic Free Trade Agreement
(NAFTA)?
1. Levels of development among countries
in the two regions are different
2. The USA, a democratic nation, is the
anchor economy for NAFTA, but China,
the anchor economy for South Asia, is a
single-party state
3. Unlike NAFTA, which is a partnership

Current Affairs 46 October-2015

only between Bermuda, Canada, Mexico,


USA and Greenland, a similar trade
partnership in South Asia would include a
larger number of members
Select the correct answer using the codes given
below:
a) 1 and 2 only
b) 2 and 3 only
c) 1 and 3 Only
d) All the above
Ans: b
Who said China is the anchor economy of SouthAsia? J
NAFTA is an agreement signed by Canada, Mexico,
and the United States, creating a trilateral rulesbased trade bloc in North America.
http://www.thehindu.com/business/south-asiaimportant-for-india-geopolitically-but-noteconomically-cea/article7702915.ece
6. Question
Which among the following best describes
Accommodative Monetary Policy?
a) When a central bank takes into

consideration growth as well as inflation


trends while deciding whether to expand
or contract money supply in an economy
b) When a central bank attempts to
expand the overall money supply to boost
the economy, when growth is slowing (as
measured by GDP)
c) When a central bank pays heed to
demands of the Finance Ministry of the
Government
d) When a central bank and the Finance
Ministry of the Government together
decide on repurchase-agreement rates
Ans: b.
http://www.investopedia.com/terms/a/accomodative
monetarypolicy.asp
While the RBIs stance will continue to be
accommodative, Dr. Rajan said, the focus of
monetary action for the near term will shift to
working with the Government to ensure that
impediments to banks passing on the bulk of the
cumulative 125 basis points cut in the policy rate are
removed.

Insights into Editorials: Limited Finance Limits Democracy


12 October 2015
Indias campaign finance rules are frequently violated. It is now a well known fact that candidates in Indian
elections grossly overshoot spending limits imposed by the Election Commission (EC). Such violations
create deeper problems for representation and democracy at large. The major Indian political parties agree
that spending limits in Indian elections, and the countrys campaign finance rules in general, are
unreasonable and have called for a radical rethinking of campaign finance rules.
What has the Election Commission been doing?
To keep it in line with ground realities, particularly inflation, the EC has been increasing the spending limit
from time to time. In the 2014 Lok Sabha elections, each candidate was allowed to spend a maximum of
Rs.70 lakh in his or her constituency, representing an increase of Rs.30 lakh over the 2009 limit. However,
such moves have not reduced the number of spending violations.
Challenges being faced by the political parties:
Current limit of Rs.70 lakh implies a total spending of just Rs.3-4 per individual in a constituency,
which is a lot less than what the EC itself spends on conducting elections.
In terms of the time for campaigning, the period per constituency currently stands at just a few
weeks, with restrictions on the resources a candidate can deploy, such as the number of vehicles that
can be used. Many of these constituencies are rural, with about 10-15 lakh individuals spread across
thousands of villages. Covering these villages poses a significant challenge for candidates.
What should be done?
To reduce the number of violations, the spending limits should be closer to the actual amount candidates
need to campaign effectively, and that requires an exponential increase in the spending limits.
Who prescribes the campaign finance rules and why do we need it?
Election commission of India prescribes such rules. The authority of the election commission comes
from the Representation of People Act, which states, the total of the said expenditure shall not
exceed such amount as may be prescribed.
Current Affairs 47 October-2015

The primary objective behind the ECs campaign finance rules is to level the playing field.
How such restrictive campaign rules infuse corruption into day-to-day politics?
Due to the official limits, candidates rely almost completely on unaccounted cash from undisclosed donors,
which essentially renders all the other transparency initiatives of the EC redundant. Once in office, the
candidates must find ways to repay their debts to these donors, and often do so by favouring them through
policy changes or resource allocation. Thus the restrictive campaign finance rules infuse corruption into dayto-day politics.
According to The Economist:
Money power has proven to be the more powerful by far. The EC sets limits on both fundraising and
expenditures, but they are laughably ineffective. Political parties and candidates must break the rules in order
to stand a chance of winning. This drives them into the arms of the criminal underworld, especially at the
local level: that is where they find the men who have ready access to the black money that escapes the
official banking sector, and the networks to disperse it.
How such rules are leading to lower levels of representation?
The low limits on campaign finance have a large impact on the very essence of representation. For voters to
make an informed choice, it is imperative that candidates and parties get their message across to each voter.
For voters to make the right choices, they need to understand and respond to the candidates policy positions
and sometimes interact with the candidates themselves. With the current rules, a law-abiding candidate
would not have the resources nor the time to make that happen. This implies lower levels of representation
and consequently greater arbitrariness in voting decisions, both of which are harmful to democratic
accountability and democracy at large. Politicians then turn to middlemen to mobilise votes with the all too
obvious negative consequences.
Conclusion:
Electoral corruption in India is a product of the institutions and systems that we have put in place. The limits
on election spending, along with the other restrictive campaign finance rules of the EC, perpetuates a tightlyguarded socialist mindset among many Indian policymakers, which often makes them wary of individual
affluence. By relaxing these rules, the Election Commission will be able to not only increase compliance,
transparency and representation in Indian elections, but also help align Indias politics with its new
economics.
For more Points, Please refer this Document.
(http://www.observerindia.com/cms/export/orfonline/modules/orfseminarseries/attachments/ss_8_1441
780285133.pdf)
places, walls of government and educational
institutions and also the thoughtless erection
Paper 2 Topic: Government policies and
of huge arches as part of campaigning, be it
interventions for development in various
during an election or for propaganda for
sectors and issues arising out of their design
commercial purposes.
and implementation.
The Bill seeks to impose curbs on sticking
posters and pictures on signboards,
Anti-Defacement Bill hanging fire
inconveniencing the public and also tourists.
Anti-Defacement Bill, mooted by the Kerala State
It also mandates prior clearance of the local
Election Commission, has not yet been passed by the
Kerala state government.
government concerned for publicity at public
About the Anti-Defacement Bill:
places.
The draft Anti-Defacement Bill was initiated by the
As per the draft, if a representative of a
state election commission in 2013. It was modelled
candidate or an institution tends to violate the
on the laws that were already in force in West
provisions of the Bill, the onus for the breach
Bengal, Haryana and Goa to check defacement by
of law would be on the beneficiary and not
indiscreetly putting up posters, bills and painting
on the representative who executes it.
graffiti in government buildings, heritage structures
The Bill also ensured due protection for
and other public places during campaigning and also
enforcing officers so that no one could
for commercial purposes.
initiate legal action against them for acting
The bill was armed with provisions to
against those violating the law.
prevent despoiling of heritage zones, public
The election commission had furnished a number of

12 October 2015

Current Affairs 48 October-2015

suggestions to add teeth to the Bill and make it an


effective tool to check such practices. But the draft
reportedly got entangled in bureaucratic wrangles
and continues to remain in cold storage.
sources: the hindu.
Paper 2 Topic: Government policies and
interventions for development in various
sectors and issues arising out of their
design and implementation.

Lok Adalats held Across the Country


Lok Adalats were recently held across the country
for cases relating to Traffic, Petty matters, Municipal
Matters, etc. that are at pre-litigation stage and also
those pending in courts for their settlement.
The aim was to reduce pendency as well as
prevent additional litigation swathing the
courts, by resolving litigation, wherever
settlements are possible.
What is Lok Adalat?
Lok Adalat is a system of alternative dispute
resolution developed in India. It roughly means
Peoples court.
Background:
The idea of Lok Adalat was mainly
advocated by Justice P.N. Bhagwati, a former
Chief Justice of India.
The first Lok Adalat was held on March 14,
1982 in Gujarat.
How they operate?
Lok Adalat is a non-adversarial system, whereby
mock courts (called Lok Adalats) are held by the
State Authority, District Authority, Supreme Court
Legal Services Committee, High Court Legal
Services Committee, or Taluk Legal Services
Committee.
They are held periodically for exercising
such jurisdiction as they determine.
These are usually presided over by retired
judges, social activists, or other members of
the legal profession.
Their Ambit:
The Lok Adalats can deal with all Civil
Cases, Matrimonial Disputes, Land Disputes,
Partition/Property Disputes, Labour Disputes
etc., and compoundable criminal Cases.
How are these different from regular courts?
These are less expensive and relations
between litigants do not get strained.
The focus in Lok Adalats is on compromise.
When no compromise is reached, the matter
goes back to the court. However, if a

compromise is reached, an award is made


and is binding on the parties.
The disputing parties plead their case
themselves in Lok Adalats. No advocate or
pleader is allowed, even witnesses are not
examined.
No court fee is levied. Speedy justice is given
to the people of all classes of society.
Award has same effect as of a Civil Court
decree.
It was the LEGAL SERVICES AUTHORITY ACT
1987, which gave statutory status to Lok Adalat.
sources: pib.
Paper 3 Topic: Indian Economy and
issues relating to planning, mobilization
of resources, growth, development and
employment.

Vodafone wins transfer pricing tax dispute


case
The Bombay High Court has ruled in favour of
Vodafone in one of a series of tax cases involving
the British telecom company in India. This decision
is being seen as positive for several other firms
fighting similar disputes.
The court backed Vodafones efforts to
oppose a move by tax authorities to add Rs
8,500 crore ($1.3 billion) to the taxable
income of a unit, Vodafone India Services
Pvt Ltd, which provided call centre services
to some group companies.
Background:
The case dates back to financial year 2007-8
involving the sale of Vodafone India Services
Private Ltd., the call centre business of
Vodafone, to Hutchison, and the tax
authorities demanded capital gain tax for this
transaction. The Income Tax department had
demanded that Rs.8,500 crore be added to the
companys taxable income.
Vodafone acquired the telecom business of
Hutchison in India to enter the Indian market.
In 2013, the Income Tax Department issued a
tax demand of Rs. 3,700 crore to Vodafone
India.
However, Vodafone argued that the sale of
the call centre business was between two
domestic companies and the transfer pricing
officer had no jurisdiction over the deal. It
argued that the Income Tax Department had
no jurisdiction in this case because the
transaction was not an international one and

Current Affairs 49 October-2015

did not attract any tax.


What is transfer pricing?
Transfer pricing is the practice of setting prices (for
goods and services sold) in transactions between
group companies (divisions of a big company) based
in different countries, on an arms length (fair value)
basis.
Investopedia definition:
The price at which divisions of a company transact
with each other. Transactions may include the trade
of supplies or labor between departments. Transfer
prices are used when individual entities of a larger
multi-entity firm are treated and measured as
separately run entities. (For more: Source)
What is arms length?
A transaction is generally described as being on an
arms length basis when a buyer and a seller act
independently and have no relationship with each
other. The concept is used to ensure both parties in
the deal are acting in their own interest and are not
subject to any pressure from the other party.
sources: the hindu, wiki.
1. Question

Consider the following statements about the


Institute for Defence Studies and Analyses
(IDSA):
1. It is an autonomous body, a think-tank on
strategic affairs
2. It is funded by the Indian Ministry of
Defense
3. It was established in 1965
Which of the above statements are correct?
a) 1 and 2 Only
b) 2 and 3 Only
c) 1 and 3 Only
d) All are correct
Ans: d.
http://www.thehindu.com/news/national/jayantprasad-is-idsa-directorgeneral/article7703445.ece

http://www.idsa.in/
2. Question
Which of the following statements about the
Astrosat are correct?
1. Indian researchers will no longer have to
rely on other space agencies for X-ray
data
2. As in the case of Chandrayaan-1 and
Mangalyaan, the Astrosat telescope will
have no commercial or societal
implications
3. Only one among five of its payloads has
been made by ISRO, the rest comes from

scientific institutions based in India


Select the correct answer using the codes given
below:
a) 1 and 2 Only
b) 3 Only
c) 1 and 3 Only
d) All of the above
Ans: c.
As in the case of Chandrayaan-1 and Mangalyaan,
the Astrosat telescope will have no immediate
commercial or societal implications. The only reason
such programmes exist and are funded by the
Government is because of the potential for positive
change they have in the future.
3. Question
With reference to the Comptroller and Auditor
General (CAG) of India, which of the following
statements is/are incorrect?
1. Its reports are submitted to the head of the
executive who then lays them before both
Houses of the Parliament
2. Its reports must be tabled by the executive
in the legislature in a time-bound manner
3. The CAG is responsible only to the
Parliament
Select the correct answer using the codes given
below:
a) 1 only
b) 1 and 2 only
c) 2 Only
d) 2 and 3 Only
Ans: b.
Page 17.14, 45.2-45.3 Indian Polity by M
Laxmikanth 4th Edition
CAG submits its reports to the President. The
President is the head of state but not of the
executive.

The CAGs reports on quite a few occasions were


not being presented to the legislatures concerned on
time, as they were deemed inconvenient by the
government of the day, aborting the scope for timely
legislative scrutiny and follow-up remedial action,
including a fixing of responsibility on the
functionaries concerned. Thus, there was a clear
undermining of the constitutional system, impeding
the accountability of the executive to the legislature
by not allowing the CAG to perform his or her
constitutionally mandated role and serve Parliament
adequately in the area of financial oversight.
http://www.thehindu.com/opinion/lead/empowernot-weaken-the-cag/article7703141.ece

Current Affairs 50 October-2015

4. Question
Consider the following statements:
Assertion (A): The Public Accounts Committee
(PAC) consists of members only from the Lok
Sabha
Reason (R): The function of the PAC is to
examine annual audit reports of the CAG which
seek to ensure financial accountability of the
executive to the Lok Sabha
Select the correct answer using the codes given
below:
a) A and R both are true, and R is the
correct explanation for A
b) A and R both are true, and R is the
NOT the correct explanation for A
c) A is correct, R is incorrect
d) A and R both are incorrect
Ans: d.
Annual audit reports of the CAG seek to ensure
financial accountability of the executive to the
Parliament (both houses, not only the Lok Sabha).
Page 23.3 Role of the CAG Indian Polity by M
Laxmikanth 4th Edition
http://www.thehindu.com/opinion/lead/empowernot-weaken-the-cag/article7703141.ece
5. Question
Consider the following statements:
Assertion (A): The immediate past, present and
next G20 Chair constitute a troika
Reason (R): The G20 does not have any
permanent secretariat or management and
administrative structure
Which of the above statements is/are true?
a) A and R both are true, and R is the
correct explanation for A
b) A and R both are true, and R is the
NOT the correct explanation for A

c) A is correct, R is incorrect
d) A and R both are incorrect
Ans: a.
The G20 operates as a forum and not as an
organisation. Therefore, it does not have any
permanent secretariat or management and
administrative structure. One of the G20 countries is
selected to hold the Chair in rotation, also known as
G20 presidency.
The presidency establishes a temporary secretariat
for the duration it holds the Chair. The secretariat
coordinates all work and organises G20 meetings.
The immediate past, present and next Chair
constitute a troika and ensure continuity in the G20
work.
6. Question
With reference to the G20 Sherpa, consider the
following statements:
1. He is the personal representative of a head
of state or government at an international
summit
2. They are generally quite influential
3. They generally lack the authority to
independently make final decisions
4. The name is derived from the Sherpa
people, a Nepalese ethnic group, who
serve as guides and porters in
the Himalayas
Which of the above statements is/are incorrect?
a) 1 and 2 Only
b) 2 and 3 Only
c) 1 and 3 Only
d) All the above
Ans: d.
http://g20watch.edu.au/what-sherpa
India recently announced the appointment of NITI
Aayog Vice-Chairman Arvind Panagariya as its G20
Sherpa.

Insights Into Editorial: India not obliged to arrest President Bashir


13 October 2015
India not obliged to arrest President Bashir
The president of Sudan, Omar al-Bashir, will shortly be visiting India to attend the Third India-Africa
Forum Summit. India has invited al-Bashir, along with 54 heads of state, for the Summit. This summit is
expected to be the largest gathering of world leaders in India since the Non-Aligned Summit in New Delhi in
1983.
However, the presence of Sudanese president Omar al-Bashir may not be appreciated by some countries.
Why?
The International Criminal Court (ICC) has issued two separate arrest warrants against president Omar alBashir and his arrival may represent a serious challenge to the authority of the International Criminal Court
(ICC).
Why were the warrants issued against him?
The first warrant was issued in March, 2009, when the President was charged with war crimes over the
Current Affairs 51 October-2015

conflict in Darfur, sudan. Second warrant was issued in July, 2010, and the ICC charged Mr. Bashir with
three counts of genocide in Darfur, accusing him of trying to wipe out three non-Arab ethnic groups in the
region.
What now?
It is being said that the office of the prosecutor of the ICC has called upon India to contribute to the
important goal of ending impunity for the worlds worst crimes by arresting and surrendering President
Bashir to the ICC.
Is India under any international legal obligation to do so?
Since India is not a signatory to the Rome Statute that established the ICC, it is not under any legal
obligation to comply with a request made by the Court. However, the situation is complicated by virtue of
the Security Council referral. Taking a serious note of what happened in Sudan, and given that Sudan was
not a party to the Rome Statute establishing the ICC, the United Nations Security Council, acting under
Chapter VII of the United Nations Charter, had referred the situation in the country to the ICC vide
Resolution 1593.
Since the resolution was adopted under Chapter VII, it is binding on all member states, including India. If
it creates legal obligations, it would prevail over Indias other international law obligations.
However, the wording of the resolution falls short of creating a legal obligation on non-member states
and only urges non-member states to co-operate.
If India arrests President Bashir, it also amounts to the violation of international law, since heads of state are
accorded full immunity from arrest in a foreign state under both treaty and customary international law.
Therefore, India is not under any legal obligation to arrest and surrender President Bashir to the ICC and
may not be permitted to arrest President Bashir on Indian soil.
Conclusion:
Nevertheless, given Indias position in the international legal order and its push for a permanent seat on the
UNSC, it may have been prudent on the part of India to have avoided this controversy altogether by not
extending the invitation to host President Bashir in the first place.
What happened in Sudan? Brief background:
President Bashir first came to power in a coup in 1989 when Sudan was in the midst of a civil war between
north and south. He then allegedly exploited these differences in order to destroy those ethnic groups, which
presented a threat to his power. This resulted in extended clashes between the governments armed forces
and rebel groups, which reportedly lasted from 2003 to 2008, a period that witnessed serious war atrocities
such as pillaging and rape. It is believed that as a result of the ensuing armed conflict, over 2,00,000 civilians
were killed and over 2.4 million people were forced to flee their homes.

13 October 2015
Paper 3 Topic: conservation.

Taking cue from Centre, State bans a drug


to save vultures
The kerala state government has withdrawn
Ketoprofen, a non steroid anti- inflammatory drug
(NSAID) used extensively for veterinary purposes,
to save the vulture population in three districts of the
state.
The State government had included
Ketoprofen based on an effort to identify an
alternative to the banned drug Diclofenac.
The Centre had banned Diclofenac multi-vial
doses after wildlife biologists proved that
presence of the drug in the carcasses of the
cattle caused the vulture population to
dwindle drastically.
How vultures are affected by these drugs?

Vultures act as scavengers, preying on dead


animals. Diclofenac in carcasses lead to slow
death of vultures.
Ketoprofen, which is seen as an alternative,
causes the same effect on the vulture
population.
sources: the hindu.
Paper 2 Topic: Government policies and
interventions for development in various
sectors and issues arising out of their
design and implementation.

Tightened DNA profiling Bill ready


Union Minister for Science and Technology Harsh
Vardhan recently said that apprehensions expressed
about misuse of the Human DNA Profiling Bill were
addressed by the Central government and the
legislation is ready for introduction in the next
session of Parliament.
The Bill is in the advanced stage of

Current Affairs 52 October-2015

becoming a law and it would be put up for


consideration by the Cabinet.
Highlights of the Human DNA Profiling Bill:
The bill proposes to allow collection of
samples from private parts of human body
for DNA profiling and data preservation with
the approval of a regulatory body.
It suggests that a National DNA Profiling
Board and a National DNA Bank be set up in
Hyderabad, with every state having a
regional DNA data bank. The DNA Data
Bank would maintain records of samples
found at crime scenes, or from suspects,
offenders, missing persons, volunteers, etc.
The bill also makes it clear that no DNA
Laboratory shall undertake DNA profiling
without the prior approval of the DNA
Board.
If a foreign country requests DNA profiling,
the DNA Bank will coordinate through CBI

Paper 1 Topic: Post-independence


consolidation and reorganization within
the country.

or a concerned department.
The bill mandates that the DNA profiles or
samples be kept confidential, and they should
be used only for establishing identity of a
person and nothing else.

Government investigation agencies and


judiciary, among others, can seek information
from Data Banks. For unauthorized use of
data, a stringent punishment is provided.
What is DNA profiling?
It is a forensic technique used to
identify individuals by characteristics
of their DNA. A DNA profile is a small
set of DNA variations that is very
likely to be different in all unrelated
individuals, thereby being as unique to
individuals as are fingerprints.
sources: the hindu, wiki.

Centre seeks report on HC order on


Article 370
The Ministry of Home Affairs has sought a report
from the Jammu and Kashmir government on the
State High Courts order that Article 370, which

Current Affairs 53 October-2015

grants special status to the State, is beyond


amendment, repeal or abrogation.
Background:
The HC had recently ruled that Article 370
has assumed place of permanence in the
Constitution and the feature is beyond
amendment, repeal or abrogation.
The court had also said that Article 35A gives
protection to existing laws in force in the
State.
It said, Article 370 though titled as
Temporary Provision and included in Para
XXI titled Temporary, Transitional and
Special Provisions, has assumed place of
permanence in the Constitution.
What is Article 370?
Article 370 of the Indian Constitution is a
temporary provision which grants special
autonomous status to Jammu & Kashmir.
Under Part XXI of the Constitution of India,
which deals with Temporary, Transitional
and Special provisions, the state of Jammu
& Kashmir has been accorded special status
under Article 370.
All the provisions of the Constitution which
are applicable to other states are not
applicable to J&K.
Important provisions under the article:
According to this article, except for defence,
foreign affairs, finance and communications,
Parliament needs the state governments
concurrence for applying all other laws. Thus
the states residents live under a separate set
of laws, including those related to
citizenship, ownership of property, and
fundamental rights, as compared to other
Indians.
Indian citizens from other states cannot
purchase land or property in Jammu &
Kashmir.
Under Article 370, the Centre has no power
to declare financial emergency under Article
360 in the state. It can declare emergency in
the state only in case of war or external
aggression. The Union government can
therefore not declare emergency on grounds
of internal disturbance or imminent danger
unless it is made at the request or with the
concurrence of the state government.
Under Article 370 the Indian Parliament
cannot increase or reduce the borders of the
state.
sources: the hindu, wiki.

Paper 2 Topic: Statutory, regulatory and various


quasi-judicial bodies.

ICHR to conduct research on princely


States
The Indian Council of Historical Research (ICHR)
will be conducting a research to unravel the history
of the many princely States.
ICHR claims that these princely states were
the guardians of Indian life and culture, but
disappeared unceremoniously and unwept
after India abolished royal entitlements.
The project has been approved in principle
by the ICHR, with an aim to shine light on
the many accomplishments of these rulers
that have not been sufficiently documented.
Research into the lives and times of these
rulers is also expected to bring out littleknown facets about them.
About the Indian Council of Historical Research:
The Indian Council of Historical Research (ICHR) is
plainly a Fund-disbursing Agency that disburses
funds to Indian and Foreign Scholars on specific
applications for Fellowships and Grants.
The source of the Funds, at the disposal of
the ICHR is Grants-in- Aid received from the
Department of Higher Education in the
Ministry of Human Resource Development,
Grants-in- Aid from various Indian States,
donations and from the proceeds of revenues
from the sale of Publications.
The council does not carry any historical
research of its own by its staff as was initially
envisaged.
It was established in 1972 by an
Administrative Order of the then Ministry of
Education of Government of India.
It is an autonomous organization.
sources: the hindu, ichr.
Topic: General awareness.
British economist Angus Deaton wins
Nobel for studies on income, poverty
Professor Angus Deaton, a British
economist, has won the Nobel Memorial
Prize in Economic Science for
improving the accuracy of basic
economic gauges, including measures of
income, poverty and consumption.
Deaton, 69, of Princeton, is best known for his
insight that economic averages such as measures of
national income could be misleading, because

Current Affairs 54 October-2015

they conceal important variations among


individuals.
Deaton was born in Scotland, and is a British
and U.S. citizen.
He was an early contributor to one of the
defining trends in modern economics: the
emphasis on building models of broad
economic trends from the ground up, using
data about individual behaviour rather than
relying on averages. He then used this
carefully constructed data to address large
questions about wealth, health and wellbeing.

lakh for a plan period of five to ten years


years along with APOs of Rs. 11,195.32 lakh
for this financial year.
The total forest and non-forest area taken up
in the four states under GIM during the total
plan period will be 1, 08,335 hectares, out of
which 81,939 ha will be improving the
density of existing forests and 16, 396 ha will
be new areas.
Green India Mission:
It is one of the eight Missions outlined under
National Action Plan on Climate Change
(NAPCC).
It acknowledges the influence forests have on
environmental amelioration through climate
change mitigation, food security, water
security, biodiversity conservation and
livelihood security of forest-dependent
communities.
It hinges on decentralized participatory
approach involving grass root level
organizations and community in planning,
decision making, implementation and
monitoring.
It lays emphasis on landscape approach and
convergence with complementary schemes
and programmes for better coordination in
developing forests and their fringe areas in a
holistic and sustainable manner.
sources: pib.
1. Question

He is the one who has brought very careful


statistical analysis of household choices into
development economics.
sources: the hindu.
Paper 3 Topic: conservation.

Green India Mission Plans of Four States


Approved
National Mission for a Green India (GIM) falling
under the Environment Ministry has approved
annual plans for Kerala, Mizoram, Manipur
and Jharkhand.
The National Executive Council (NEC),
which met recently, approved the Perspective
Plans (PP) and Annual Plan of Operations
(APOs) of Mizoram, Manipur, Jharkhand and
Kerala.
NEC approved the plans of all four states
with a total financial outlay of Rs. 90,202.68

Which of the following statements about


PRAGATI is/are correct?
1. It stands for Pro-Active-Governance
aimed at Transformation of India
2. It brings on one stage the Secretaries of
Government of India and the Chief
Secretaries of the States
Select the correct answer using codes given
below:
a) 1 Only
b) 2 Only
c) Both 1 and 2
d) Neither 1 or 2
Ans: b.
http://pib.nic.in/newsite/PrintRelease.aspx?
relid=128347

http://pib.nic.in/newsite/PrintRelease.aspx?
relid=117685
2. Question
Global Competitiveness Report is released by
the

Current Affairs 55 October-2015

a) World Bank
b) World Economic Forum
c) Organisation for Economic
Cooperation and Development
d) World Trade Organisation
Ans: b.
http://www.thehindu.com/business/Economy/indiaclimbs-16-places-in-wefs-globalcompetitivenessindex/article7708043.ece
3. Question
Operation Muskaan and Operation Smile were
campaigns recently launched by the
a) Ministry of Women and Child
Development
b) Ministry of Home Affairs
c) Ministry of Health and Family Welfare
d) Ministry of Minority Affairs
Ans: b.
http://pib.nic.in/newsite/PrintRelease.aspx?
relid=128316
4. Question
Consider the following statements:
1. As of 2013, insurance penetration (both,
life and non-life) stood at 3.9% in India
2. The Pradhan Mantri Jan-Dhan Yojana
provided a life insurance cover, to those
who met the criteria, during the initial
period following its launch
Which of the above statements are incorrect?

a) 1 Only
b) 2 Only
c) Both 1 and 2
d) Neither 1 or 2
Ans: d.
Page 45, 48: Economic Survey 2014-15, Volume 2
http://www.thehindu.com/business/Industry/lifeinsurance-penetration-should-be-through-banksreport/article7708040.ece
This article states, As compared to global
average penetration and density level of 3.4 per cent
and $368, Indias insurance penetration and density
levels are much lower at 2.6 per cent and $44.
However, it is safe for us to assume that the
penetration of life insurance in India is actually
3.1%, as mentioned in the survey.
5. Question
The Caracal has been classified by the
International Union for Conservation of Nature as
a) Critically Endangered
b) Endangered
c) Vulnerable
d) Least concern
Ans: d.
http://www.iucnredlist.org/details/3847/0

http://www.thehindu.com/opinion/op-ed/lost-in-aforest-of-bad-ideas/article7707959.ece
The article in the Hindu mentions caracal to be
endangered but IUCN lists it as least concern

Insights into Editorial: It is Time to Render Sedition (Section 124A of IPC)


Unconstitutional
14 October 2015
Some state governments in India have been using Sedition as a tool to silence or discipline critics.
What is sedition?
Sedition is an offence incorporated into the Indian Penal Code (IPC) in 1870. Section 124A of the IPC
defines sedition and says:
whoever by words either spoken or written or by signs or by visible representation or otherwise
brings or attempts to bring into hatred or contempt, the government established by law; or
whoever by the above means excites or attempts to excite disaffection towards the government
established by law, has committed the offence of sedition.

According to the law, disaffection includes disloyalty and all feelings of enmity. However,
Current Affairs 56 October-2015

disapprobation of the measures or administrative action of the government to obtain their alteration by
lawful means is not an offence.
The law also says that there should be no attempt to excite hatred or contempt or disaffection. In other
words, even if the impugned speech or article or cartoon seeks to obtain the alteration of the wrong
governmental decisions, if they excite hatred, contempt or disaffection towards the government, the author of
the speech and others are liable to be charged with sedition and punished.
How is it being used by the governments?
A number of litigations in 1950s and 1960s, and the amendments made to Article 19 of the
Constitution have widened the scope of sedition.
In recent years, sedition has been frequently invocated to deal with free speech and expression. Some
recent cases are indicative of a high level of intolerance being displayed by governments towards the
basic freedom enjoyed by citizens.
Brief history:
Originally, it was enacted to silence the Indian people by the colonial rulers. After independence, it was
retained by the democratic government in free India. Sedition was not a part of the original Indian Penal
Code (IPC) enacted in 1860. It was introduced in 1870.
Difficulty in its interpretation:
Pre independence era:
In the pre-Independence era, a number of landmark cases on sedition were decided by the Federal
Court as well as the Privy Council. These two high judicial bodies had taken diametrically opposite
positions on the meaning and scope of sedition as a penal offence.
Views of the federal court:
The Federal Court in Niharendu Dutt Majumdar Vs. King Emperor (1942) FCR 48 , held that public
disorder or the reasonable anticipation or likelihood of public disorder is the gist of the offence. The court
was of the view that sedition implies resistance or lawlessness in some form. However, the federal court held
that if there is no incitement to violence, there is no sedition.
Views of the privy council:
The Privy Council was of the view that acts like incitement to violence and insurrection are
immaterial while deciding the culpability of a person charged with sedition. It said that since the IPC
defines the offence of sedition, unlike the English Law, which doesnt define it, one needs to go by
that definition only.
In King Emperor V. Sadashiv Narayan Bhalerao (1947 ) case, the Privy Council held that the Federal
Courts statement of law in the Niharendu Majumdar case was wrong. The Privy Council overruled
the decision of the Federal Court and held that excitement of feelings of enmity to the government is
sufficient to make one guilty under Section 124A of the Code.
Queen Empress Vs. Bal Gangadhar Tilak (1897) was the first case wherein the law on sedition under
Section 124A in the IPC was explained. Strachey J. stated the law in the following terms;
The offence consists in exciting or attempting to excite in others certain bad feelings towards the
government. It is not the exciting or attempting to excite mutiny or rebellion or any sort of actual
disturbance, great or small. Whether any disturbance or outbreak was caused by these articles is
absolutely immaterial.
Post independence era:
The Constitution bench of the Supreme Court explained the amplitude of sedition for the first time in 1962 in
the case of Kedarnath Vs. State of Bihar ( 1962 ). The court adopted the view of the Federal Court of India
that the gist of the offence of sedition is incitement to violence or the tendency or the intention to create
public disorder. So, as per the Constitution Bench of the Supreme Court, a person can be charged with
sedition only if there is incitement to violence in his speech or writing or an intention to create
disorder.
Current scenario:
Despite the fact that sedition was a colonial law meant to suppress the voice of Indian people, the law
enforcement agencies today have been using it against artists, public men, intellectuals criticising the
governments.
It appears that the government and its agencies are following the law enunciated by the Privy Council
Current Affairs 57 October-2015

and not by the Supreme Court in Kedarnath. It is also being said that the governments in free India
continue to use it for the very purpose for which the colonial government used it.
Conclusion:
Democracy has no meaning without freedoms and sedition as interpreted and applied by the police and
governements is a negation of it. Hence, before the law loses its potency, the Supreme Court, being the
protector of the fundamental rights of the citizens has to step in and evaluate the law and may declare
Section 124A unconstitutional if necessary.
become operational by mid-November this year.
About Cyberdome:
Paper 3 Topic: Awareness in the fields of IT.
Cyberdome will be a hi-tech centre for cyber
security. The project is worth Rs.2-crore. The project
Cyberdome to become operational next
is being established on the public-private partnership
month
model with the technical support offered by IT
Cyberdome, the hi-tech centre for cybersecurity
companies.
being set up by the Kerala Police, is expected to
Unique features of the project:

14 October 2015

As many as 500 ethical hackers and


cybersecurity experts would be involved in the
project
It would have centres for social media
awareness, protection of children on the
Internet, Internet monitoring and ICT
(Information and Communication
Technology) in service delivery.
It would also host an Anti-Cyber Terror Cell
and a cybersecurity training unit.
It would be equipped with an automated
crime intelligence-gathering unit and a unit
for anti-piracy on the Internet.
It will have its server hosted at the State Data
Centre. Software companies will provide
technical support on a voluntary basis,

develop software for the purpose, and supply


technical manpower.
The station will be manned by police officers
with IT-related qualifications. The Additional
Director General of Police (Crimes) will be
in charge of the project.
Cyberdome would be open to new models of
partnership to find solutions to emerging
threats and challenges.
Important functions:
It would work on prevention of hacking and
defacement of websites and child
pornography.
It would have facilities to analyse Skype and
VOIP (Voice over Inernet Protocol) calls and
deal with money laundering on the Internet.

Current Affairs 58 October-2015

Policing the Dark Net would be another


mandate of the Cyberdome.
Cyberdome would synergise its operations
with the Computer Emergency Response
Team-Kerala.
sources: the hindu.
Paper 2 Topic: Important aspects of
governance, transparency and
accountability.

PMs photo on ads: SC notice to Centre


The Supreme Court has asked the Centre to respond
to a plea by an NGO challenging the publishing of
Prime Ministers photograph in government
advertisements.
Background:
An NGO had filed a PIL seeking a judicial review of
the supreme courts verdict exempting the Prime
Minister from a ban on politicians photographs,
including Chief Ministers, on government
advertisements.
Why, according to NGO, even the PM should not be
exempted from the ban?
The NGO contends that the exemption from
the ban gives the PM and the political party
he belongs to, mileage over the political
opponents.
Such advertisements have the potential to
create partisan politics, favouring party in
power and putting Opposition in bad light.
The exemption given to the Prime Minister
undermines the letter and spirit of the courts
guidelines and defeated the very purpose
involved in disseminating information to the
citizens about government schemes, policies,
welfare programmes and achievements.
Supreme Courts order:
In May 2015, the Supreme Court issued
guidelines for the publication of government
advertisements and held that publication of
photographs of politicians and government
functionaries, including Chief Ministers,
defeated the public interest behind
advertising welfare schemes and encouraged
personality cults.
However, the court exempted the President,
the Prime Minister and the Chief Justice of
India from this ban, leaving it to their
discretion to decide whether they wanted
their pictures published in the advertisements
or not.
Sources: the hindu.

Paper 1 Topic: Salient features of Indian


society.

Uniform Civil Code must for national


integration
Union Law Minister D V Sadananda Gowda has
asserted that a common code is necessary for
national integration. However, the issue requires
discussions with all stakeholders, including various
personal law boards, to evolve consensus.
The Supreme Court had recently asked the
central government to come to a conclusion
regarding framing and implementing the
Uniform Civil Code.
Even the high courts of Kerala and
Karnataka, in their judgments dealing with
some marriage laws, had already said that a
common code was needed.
What does Article 44 say?
Article 44 of the Constitution says that there should
be a Uniform Civil Code. According to this article,
The State shall endeavour to secure for the citizens
a uniform civil code throughout the territory of
India. Since, the Directive Principles are only
guidelines, it is not mandatory to use them.
What is uniform civil code?
Uniform civil Code is a proposal to have a generic
set of governing laws for every citizen without
taking into consideration the religion.
Current situation:
Currently, there are personal laws based on the
scriptures and customs of each major religious
community. They are separate from the public law
and are applied on issues like Marriage
Divorce
Inheritance
Adoption and maintenance
Arguments against the implementation of a Uniform
civil code:
India being a secular country guarantees its
minorities the right to follow their own religion,
culture and customs under Article 29 and 30. But
implementing a Uniform Code will hamper Indias
secularism.
sources: the Hindu, India today.
Paper 3 Topic: Inclusive growth and
issues arising from it.

Jan Dhan deposits exceed Rs 25k cr


Deposits in accounts opened under the Pradhan
Mantri Jan Dhan Yojana (PMJDY) recently
exceeded Rs 25,000 crore. And, PMJDY accounts

Current Affairs 59 October-2015

with balances now exceed 60%, with zero-balance


accounts constituting less than 40%. This ratio was
76% to 24% in September 2014.
PRADHAN MANTRI JAN DHAN YOJANA:
The PMJDY was conceived as a national mission on
financial inclusion with the objective of covering all
households in the country with banking facilities and
having a bank account for each household.
It is a scheme for comprehensive financial
inclusion.
Benefits under PMJDY Scheme:
Interest on deposit.
Accidental insurance cover of Rs.1.00 lac
Accounts can be opened with zero balance.
No minimum balance required.
Life insurance cover of Rs.30,000/ Easy Transfer of money across India
Beneficiaries of Government Schemes will
get Direct Benefit Transfer in these accounts.
After satisfactory operation of the account
for 6 months, an overdraft facility will be
permitted
Access to Pension, insurance products.
Accidental Insurance Cover, RuPay Debit
Card must be used at least once in 45 days.
Overdraft facility up to Rs.5000/- is available
in only one account per household,
preferably lady of the household.
Sources: the Hindu.
1. Question

The Mid-day Meal Scheme is administered by


the
a) Ministry of Human Resource
Development
b) Ministry of Health and Family Welfare
c) Ministry of Women and Child
Development
d) Ministry of Consumer Affairs, Food
and Public Distribution
Ans: a.
http://mdm.nic.in/

http://timesofindia.indiatimes.com/india/20-yearsafter-its-launch-Mid-Day-Meals-become-a-childsright/articleshow/49190327.cms
2. Question
The Manual of Departmental Security
Instructions, 1994 was issued by
a) A Union Cabinet resolution
b) The Ministry of Home Affairs
c) The Ministry of Law and Justice
d) The Ministry of Defense

Ans: b.
The Ministry of Home Affairs had issued the
Manual of Departmental Security Instructions,
1994 which provides for a comprehensive set of
instructions regarding classification of documents,
the procedure for dealing with the same as also the
responsibility of Government servants. These
instructions are already available with all the
Ministries/Departments. These instructions are also
reiterated from time to time to all
Ministries/Department by the Ministry of Home
Affairs.
http://pib.nic.in/newsite/PrintRelease.aspx?
relid=117184

http://www.thehindu.com/opinion/editorial/thearchives-beckon/article7712680.ece
3. Question
Consider the followings statements about the
Official Secrets Act:
1. Classification of files is done under its
provisions
2. Any breach of the Manual of
Departmental Security Instructions could
attract its provisions
Which of the above statements are correct?
a) 1 Only
b) 2 Only
c) Both 1 and 2
d) Neither 1 or 2
Ans: b.
The classification of files is not done under the
provisions of the Official Secrets Act. The
classification or declassification of files is done by
each Ministry/ Department of the Government as per
their requirement in terms of the Manual of
Departmental Security Instructions, 1994. These
instructions are reviewed by the Ministry of Home
Affairs from time to time and reiterated to all the
Ministries/Departments for compliance.
http://www.thehindu.com/opinion/editorial/thearchives-beckon/article7712680.ece
4. Question
Which of the following sections of the Right to
Information Act, 2005 provides the means to
government departments to decline information
sought by an individual?
a) Section 6
b) Section 7
c) Section 8
d) Section 9
Ans: c.
http://www.thehindu.com/opinion/editorial/the-

Current Affairs 60 October-2015

archives-beckon/article7712680.ece
5. Question
Consider the following statements about ASCI
(Advertising Standards Council of India):
1. It comes under the Ministry of Consumer
Affairs, Food and Public Distribution
2. Any consumer can lodge complaints (with
ASCI) on what they may consider to be
misleading advertisements
Which of the above statements are incorrect?
a) 1 only
b) 2 Only

c) Both 1 and 2
d) Neither 1 or 2
Ans: a.
ASCI is a voluntary, self-regulatory body. It has
partnered with the Department of Consumer Affairs
to tackle misleading advertisements.
http://www.ascionline.org/

http://www.thehindu.com/business/Industry/ascifinds-airtel-4g-ad-misleading-teleco-to-seekreview/article7712821.ece

Insights into Editorial: The Perils of e-fixation


15 October 2015
The Organization for Economic Cooperation and Development (OECD), through its recent study, has
generated sufficient evidence to say that computers in schools do not necessarily contribute to higher
achievement levels by children.
How the study was carried out?
The study used two kinds of data from different countries. One set of data consisted of childrens
scores in the Programme for International Student Assessment (PISA) tests of reading and
mathematics. The other set of data is about the availability and use of computers and Internet in
schools and homes.
These two kinds of data have been analyzed to find out whether computer-based teaching in the
classroom improves childrens ability to read purposefully, and their performance in science and
mathematics.
The study also compared different OECD countries in terms of the average daily time spent by
children in computer-assisted learning in the classroom.
Findings of the study:
The findings of the study have received worldwide attention. The study shows that;
In countries where computers and the Internet are used frequently in the classroom, students have not
consistently achieved high scores in digital reading, mathematics or science.
On the other hand, countries like South Korea, Japan and Singapore, where classroom use of
computers and the Internet is relatively limited or minimal, students have achieved consistently high
scores over the recent years.
First of all, why do we need computer knowledge?
It is an accepted fact that young people need to be competent in the use of computers and Internet in order to
do well in the job market.
What these studies indicate?
These revelations have raised some basic issues and questions about the policies followed in major European
countries with the OECDs inspiration and guidance. The finding also causes a dilemma.
How, even with limited accessibility to Internet and computers, East Asian countries are performing well?
Noted child psychologists have always advised caution in the use of new digital tools for teaching
children. The OECD study suggests that East Asian countries are exercising this kind of caution.
These countries also exercise much focused policies.
And also, the policy environment across East Asia is a lot more positive as far as teachers are
concerned.
Indian scenario:
Like all Western countries, India has invested a vast amount of public funds for the supply of
computers to schools. Since the 1990s, State governments have spiritedly promoted the use of
computers in teaching. In private schools too, the idea that computers enhance childrens academic
achievement has been assumed to be true.
Is it true that the need of teachers is more in a computer-assisted lesson than in a conventional lesson?
Current Affairs 61 October-2015

Teachers matter even more in computer-equipped classrooms. The digital environment requires a
greater engagement between the teacher and students over any subject matter. This necessity arises
out of the nature of tools involved in the new information and communication technologies (ICTs).
Both the speed at which the technologies respond and the quantum of information they provide need
dexterous negotiation by the user.
Students using a computer with Internet need to develop considerable experience and skills of
mindful reading to be able to spot important points and trace how they have been arrived at. When a
variety of sources are available, students need to know how to distinguish reliable sources from the
rest. Skills of this kind require painstaking guidance by a competent teacher.
Hence, the engagement expected between the teacher and the student would be higher in a computer-assisted
lesson than in a conventional lesson.
How computer based education in India is treated?
In India, the computer is treated as a device that can make the teacher dispensable, even disposable. Millions
of rupees have been spent by the governments on equipping schools with computers and millions more have
been saved by reducing expenditure on teacher recruitment, emoluments, and training.
Where the real problem lies?
When it comes to preparing children and teachers for the problems and demands that the digital
environment poses for education, Indias policies lack vision. Purchase and supply of technology to
schools appear to be the only challenges the government worries about. At what age children need
introduction to ICT and how their progress is to be guided have received less attention.
The new-age administrators in India believe that the Internet can address all pedagogic needs. They
do not understand curriculum policies or examination reforms.
In India, it is increasingly becoming difficult to explain State policies. In many regions of India, lack
of focus and volatility characterize both routine and reform activities in education.
In almost all States in India, equipping schools with computers is treated as a reliable short cut to
higher quality.
Conclusion:
Psychologists started to decipher the process of learning in childhood more than a hundred years ago. Jerome
Bruner, arguably the most respected living psychologist today, wrote in an article a few years ago that a
century of research on learning tells that we know very little about it. That means we need to be modest in
our hopes, and substantially worried about the tumult our children are facing in a world that looks radically
different from the one their parents know and live in. The OECD study compels us to revisit conventional
assumption and the policies based on it. The study asks us to reflect on both the potential and limitations of
the new tools now available for learning and teaching.
from FAO earlier this year.
About FAO:
Paper 2 Topic: Important International
The Food and Agriculture Organization is an
institutions, agencies and fora, their structure,
agency of the United Nations that leads
mandate.
international efforts to defeat hunger.
It acts as a neutral forum where all nations
Indian appointed to key FAO post
Kundhavi Kadiresan has been appointed as the
meet as equals to negotiate agreements and
Assistant Director-General of the UNs Food and
debate policy.
Agriculture Organization and Regional
it is also a source of knowledge and
Representative for Asia and the Pacific.
information, and helps developing countries
Kundhavi is an economist who has spent
and countries in transition modernize and
most of her professional career with the
improve agriculture, forestry and fisheries
World Bank Group.
practices, ensuring good nutrition and food
Kadiresan will lead the prioritisation of
security for all.

There are a total of 197 members comprising


FAOs work across the region in close
consultation with technical specialists of the
194 member nations, 1 member organization
organisation.
(European Union) and 2 associate members
She replaces Hiroyuki Konuma who retired
(Faroe Islands and Tokelau).

15 October 2015

Current Affairs 62 October-2015

sources: the hindu, wiki.


Paper 3 Topic: Effects of liberalization
on the economy, changes in industrial
policy and their effects on industrial
growth.

India now most attractive investment


destination: EY
India has been named the most attractive country
for investment in a survey of more than 500 global
investors published by accounting firm EY (Ernst &
Young).
What else the survey says?
According to the survey, the second most
favoured investment destination is China and
is followed by Southeast Asia and Brazil.
32% of the 505 executives questioned said
India was their favoured market for
investment, with China second on 15% of the
vote. About 62% said they were looking at
manufacturing, both to serve the Indian and
global markets from India.
Perception about Indias macroeconomic
stability is up to 76% in 2015 in comparison
to 70% in.
Perception about political and social stability
is up from 59% in 2014 to 74% in 2015.
For relaxation in FDI policy the score
improved from 60% in 2014 to 68% in 2015.
For governments efforts to ease doing
business the score has improved from 57% in
2014 to 67% in 2015.
Compared to the 2014 survey, the number of
respondents, who believe that India would be
among the worlds leading top three
destinations for manufacturing by 2020, had
increased from 24% to 35%, while those who

believed India would evolve as a regional


and global hub for operations was up from
9% to 21%.
Among specific reforms expected to drive
growth, 89% of the investors polled said that
investment in infrastructure projects and the
100 Smart Cities project would be
significant.
Financial inclusion, including Digital India
and the Governments proposal to reduce the
rate of corporate tax from 30 %to 25%, were
considered significant by 83% of the
respondents.
Implementation of Goods and Services Tax
(GST) and legislation on land acquisition
were also mentioned by investors as
important for attracting FDI.
Investors rated Indias domestic market and
availability of labour among the most
attractive features for doing business.
Indias notorious red tape, complex investment rules
and poor infrastructure have long posed huge
obstacles for companies, with the World Bank
ranking it 142 out of 189 countries in its Ease of
Doing Business index.
sources: the hindu.
Paper 2 Topic: India and its
neighborhood- relations.

Cabinet approves revised cost of Kaladan


project with Myanmar
The Union Cabinet has given its approval to the
revised cost estimate of Rs 2,904 crore for the
Kaladan Multi Modal Transit Transport project in
Myanmar.
About the project:
The Kaladan project connects Sittwe Port in
Myanmar to the India-Myanmar border.

Current Affairs 63 October-2015

The project was jointly initiated by India and


Myanmar to create a multi-modal platform for
cargo shipments from the eastern ports to
Myanmar and to the North-eastern parts of the
country through Myanmar.
It is expected to open up sea routes and
promote economic development in the Northeastern states, and also add value to the
economic, commercial and strategic ties
between India and Myanmar.
This project will reduce distance from
Kolkata to Sittwe by approximately 1328 km
and will reduce the need to transport good
through the narrow Siliguri corridor, also
known as Chickens Neck.
The project is likely to be completed by
2016. The project is being piloted and funded
by the Ministry of External Affairs (India).
sources: the hindu, wiki.
Paper 2 Topic: Statutory, regulatory and
various quasi-judicial bodies

FSSAI to notify junk food guidelines for


schools
The countrys apex food regulator, FSSAI, has put
up draft guidelines on its website intended to
monitor the consumption of junk food by school
children.
Important guidelines:
Under the draft guidelines, which were first
submitted to the court last year, food high in
fat, salt or sugar will not be sold within 50
metres of a schools premises.
This includes an array of food & beverages
commonly consumed by school children
including chips, ready-to-eat noodles, pizzas,

burgers, sugar-sweetened carbonated and


non-carbonated drinks, potato fries
(commonly called French fries) and
confectionery items.
The draft guidelines have also suggested
creation of a canteen policy and school health
education programmes to make students and
parents aware about the ill-effects of
unhealthy food habits.
Implications:
The guidelines if notified could bring about a
paradigm shift in how food is consumed in schools
in India.
Background:
The move comes after the Delhi High Court
had given the Food Safety & Standards
Authority of India three months in a hearing
to do so.
The judgement was in response to a public
interest suit moved in 2010 by a nongovernment organisation.
The NGO had raised the issue of easy
availability of junk food and carbonated
drinks to children and had sought a ban on
these food items in schools.
About FSSAI:
The Food Safety and Standards Authority of India
(FSSAI) has been established under Food Safety and
Standards Act, 2006 which consolidates various acts
& orders that have hitherto handled food related
issues in various Ministries and Departments.
It was created for laying down science based
standards for articles of food and to regulate
their manufacture, storage, distribution, sale
and import to ensure availability of safe and
wholesome food for human consumption.

Current Affairs 64 October-2015

Ministry of Health & Family Welfare,


Government of India is the Administrative
Ministry for the implementation of FSSAI.
The Chairperson and Chief Executive Officer
of Food Safety and Standards Authority of
India (FSSAI) are appointed by Government
of India.
The Chairperson is in the rank of Secretary to
Government of India.
Important functions performed by the authority:
Framing of Regulations to lay down the
Standards and guidelines in relation to
articles of food and specifying appropriate
system of enforcing various standards thus
notified.
Laying down mechanisms and guidelines for
accreditation of certification bodies engaged
in certification of food safety management
system for food businesses.
Laying down procedure and guidelines for
accreditation of laboratories and notification
of the accredited laboratories.
To provide scientific advice and technical
support to Central Government and State
Governments in the matters of framing the
policy and rules in areas which have a direct
or indirect bearing of food safety and
nutrition .
Collect and collate data regarding food
consumption, incidence and prevalence of
biological risk, contaminants in food,
residues of various, contaminants in foods
products, identification of emerging risks and
introduction of rapid alert system.
Creating an information network across the
country so that the public, consumers,
Panchayats etc receive rapid, reliable and
objective information about food safety and
issues of concern.
Provide training programmes for persons
who are involved or intend to get involved in
food businesses.
sources: bs.
Prelims: International Current events

Jamaican novelist Marlon James wins


Man Booker prize
Marlon James, the Jamaican novelist, has won the
prestigious Man Booker Prize for Fiction for A
Brief History of Seven Killings.
He is the first Jamaican author to win the
prize in the British awards 47 years. Its also
the first for his publisher, Oneworld

Publications.
The novel is fictional retelling of the 1976
attempted murder of Bob Marley.

About Man Booker Prize:


The Man Booker Prize for Fiction is a
literary prize awarded each year for the best
original novel, written in the English
language, and published in the UK.
The winner of the Man Booker Prize is
generally assured of international renown and
success; therefore, the prize is of great
significance for the book trade.
The prize was historically limited to authors
from the U.K., the Commonwealth, Ireland
and Zimbabwe; this is only the second year
that it has been open to anyone writing in the
English language and published in the U.K.
The prize is the worlds most important
literary award and has the power to transform
the fortunes of authors and publishers. It
carries a cash prize of 50000 pounds.
sources: the hindu, wiki.
1. Question

With reference to Dengue, which of the following


statements are incorrect?
1. A drop in platelet count is proof that the
person has dengue fever
2. Recovery from infection by one serotype
provides lifelong immunity against other
serotypes
Select the correct answer using the codes given
below:
a) 1 only
b) 2 only
c) Both
d) None
Ans: b.
In a number of viral infections the platelet count
drops but that in itself is not an indication of an
emergency and the majority of dengue cases can be
managed at home with the standard treatment being
adequate hydration through ingestion of fluids.
There are 4 distinct, but closely related, serotypes of
the virus that cause dengue (DEN-1, DEN-2, DEN-3
and DEN-4). Recovery from infection by one
provides lifelong immunity against that particular

Current Affairs 65 October-2015

serotype. However, cross-immunity to the other


serotypes after recovery is only partial and
temporary. Subsequent infections by other
serotypes increase the risk of developing severe
dengue.
2. Question
Consider the following statements associated
with Indias INDC (Intended Nationally
Determined Contribution):
1. It contains sector-specific targets
2. Currently, the installed power capacity
from non-fossil fuel-based energy sources
is around 30%
Which of the above statements are incorrect?
a) 1 only
b) 2 Only
c) Both
d) None
Ans: a.
http://www.business-standard.com/article/economypolicy/india-announces-new-climate-change-targets115100200124_1.html
3. Question
Consider the following statements about Rabies
infection:
1. It is caused only by dogs (usually due to
dog bites)
2. In rare cases, rabies can be spread when
infected saliva gets into an open wound or
the mucous membranes, such as the
mouth or eyes
Which of the above statements are correct?
a) 1 Only
b) 2 Only
c) Both
d) None
Ans: b.
http://www.mayoclinic.org/diseasesconditions/rabies/basics/causes/con-20019900

http://www.thehindu.com/news/national/kerala/need
ed-better-urban-planning-not-mass-culling-ofdogs/article7720743.ece
4. Question
Which among the following list are security
features, which facilitate the detection of genuine
notes vis--vis forgeries, present on Indian banknotes?
1. Virtual Image
2. Intaglio Printing
3. Check Mark
4. Micro-lettering
5. See through Register

Select the correct answer using the codes given


below:
a) 1, 2 and 4 only
b) 3 and 5 only
c) 2, 4 and 5 only
d) All of the above
Ans: c.
https://www.rbi.org.in/currency/Security
%20Features.html

http://www.thehindu.com/news/national/nia-teamfor-bangladesh-to-probe-fake-currencycase/article7720759.ece
5. Question
With reference to the Iron Curtain, which of the
following statements is/are correct?
1. The Berlin wall formed a part of it
2. The curtain fell around the time of
disintegration of the Soviet Union
Select the correct answer using the codes given
below:
a) 1 Only
b) 2 Only
c) Both
d) None
Ans: c.
http://www.thehindu.com/news/national/cold-war-isback-peace-activist/article7720749.ece

https://en.wikipedia.org/wiki/Iron_Curtain
6. Question
Consider the following list of countries:
1. India
2. Germany
3. Russia
4. Afghanistan
5. Nepal
6. United Kingdom
7. Myanmar
The proposed INSPA (International Agency for
Solar Policy & Application) will comprise which
of the above list of countries?
a) 1, 2, 3 and 6 only
b) 1, 4, 5 and 7 only
c) 1 and 7 only
d) All of the above
Ans: c.
India has also decided to anchor a global solar
alliance, INSPA (International Agency for Solar
Policy & Application), of all countries located in
between Tropic of Cancer and Tropic of
Capricorn.

Current Affairs 66 October-2015

Stay updated on this there are chances that the


membership criteria may change later on.

http://pib.nic.in/newsite/PrintRelease.aspx?
relid=128403

Insights into Editorial: Tying up Innovation in


Legal Knots
16 October 2015
Paper 3 Topic: Issues relating to intellectual property rights.
The Patent Office in August this year released a new set of Guidelines for the Examination of Computer
Related Inventions. However, these new set of guidelines were not widely welcomed. It is being said that
these new rules completely change the extant interpretation of the Patents Act, 1970 and the legislative intent
behind certain provisions of the Act.
Quick look at Section 3(k) of the patents act and its significance:
Section 3(k) of the Patents Act was introduced in 2002, following a Joint Parliamentary Committee (JPC)
report on various amendments to the Act proposed initially in 1999.
This provision was originally intended as a blanket ban on patenting of all mathematical/business
methods, computer programmes and algorithms.
However, following the recommendations of the JPC, the exclusion on software was narrowed by the
insertion of the words per se to qualify computer programme.
What did per se mean?
By using per se, the JPC did not mean that the software be granted patent protection. It was only
narrowing the exclusion made by Section 3(k) to permit patents for those innovations comprising
both hardware and software, where the entire device was worthy of patent protection and not
merely the software portion
The JPC was clear in its intent to only permit protections for those inventions where the novelty
being protected did not reside exclusively in the software portion of the invention.
The final provision in the Patents Act, 1970 therefore specified that a patent could not be granted for a
mathematical or business method or a computer programme per se or algorithms, while limiting the
scope of patenting of software to certain specific circumstances.
What the new guidelines say?
The new Guidelines provide that if a claim specifies an apparatus in connection with or a technical
process for carrying out a business method or demonstrates a practical application for a mathematical
method, it can be considered an invention (i.e. could be the subject of patenting if it meets the other
tests of patentability). However, this violates the letter and spirit of Section 3(k) of the Patents Act.
The Guidelines lay down that a patent should not be denied if a claim directed primarily at software
also establishes industrial applicability of the invention.
The Guidelines also permit patents to be granted where a claim shows novel software with known
hardware that goes beyond the normal interaction with that hardware and that affects a change in
functionality of the hardware.
Criticism:
The new Guidelines go directly against the previous statutory provision.
Laws of nature and abstract ideas are considered the basic tools of scientific and technological
work. However, the new rules are against these globally accepted principles of patenting law, which
recognise that abstract ideas, mathematical models and the like all occur naturally/are found in nature
and as such are not invented, but merely discovered.
There is also an allegation that by making these changes, the Patent Office has given in to the
demands of the pro-patenting lobby, thereby putting innovation in India at serious risk.
Many leaders from the Indian IT industry have stated that software patents are a scourge to the
Indian software industry.
Current problems:
Current Affairs 67 October-2015

In so far as software patents are concerned, it is being said that the new Guidelines have
misinterpreted the earlier comments of the JPC and have also ignored certain critical changes made
by Parliament to the Patents (Amendment) Ordinance of 2004, when finally enacted as the Patents
(Amendment) Act of 2005.
In the 2005 amendment to the Patent Act, Parliament specifically rejected proposed amendments to
Section 3(k) that would have the effect of further narrowing the exception created by the section
(thereby increasing the scope of patenting software).
Parliament also specifically rejected proposals to permit software to be patented when industrial or
technical application was demonstrated or when in a combination with hardware.
Why software patents should not be encouraged?
Software patents restrict technological progress and encourage monopolisation.
They also massively enhance costs through the creation of patent thickets and through the diversion
of funds from productive R&D towards litigation and discovery/licenses.
They lead to stunting of technical progress and innovation.
The U.S. Supreme Court in Alice Corporation v. CLS Bank has also reaffirmed that, monopolisation of
those tools through the grant of a patent might tend to impede innovation more than it would tend to
promote it, thereby thwarting the primary object of intellectual property law.
India should also learn from countries like New Zealand and Germany which have already moved to
reconfigure their patent regimes to abolish software patents.

16 October 2015
Paper 2 Topic: Government policies and
interventions for development in various
sectors and issues arising out of their design
and implementation.

Govt. cannot insist on Aadhaar: SC


The Supreme Court has ruled that the Aadhaar
Card may be used to avail government schemes
including employment guarantee, pension and bank
accounts.
Now, the Aadhaar card can be used for
availing the benefits of Mahatma Gandhi
National Rural Employment Gurantee Act,
Jan-Dhan Yojana, provident funds and
pension schemes.
Previously, as per the courts order, the card
could have been used only voluntarily for
availing subsidies under the public
distribution system and purchasing
kerosene and cooking gas.
This ruling is being seen as a relief to the
government which has been trying to
persuade the Supreme Court to extend
Aadhaars uses after a slew of blows to the
unique identity or UID programme
following privacy petitions.
The aadhar scheme was challenged in court
over privacy concerns since it uses
biometric data like fingerprint and iris
scans.
About AADHAAR:
Aadhaar is a 12 digit individual identification

number which will serve as a proof of identity and


address, anywhere in India.
Aadhaar-platform is aimed at providing
social security benefits / subsidies based on
eligibility through direct benefit transfer.
It helps bring transparency and eliminate
corruption, leakage and inefficiency.
It was conceived as an initiative that would
provide identification for each resident
across the country and would be used
primarily as the basis for efficient delivery
of welfare services.
It is also also expected to act as a tool for
effective monitoring of various programs
and schemes of the Government.
Who assigns the number?
The Unique Identification Authority of India
(UIDAI) assigns Unique Identification Number
Aadhaar to residents of India on voluntary basis.
Issues:
Aadhaar lacks legal or statutory authority as
of now.
The AADHAAR number is not recognized
as a legal proof of residence due to issues
with the data protection.
Indias Intelligence Bureau claims anyone
with an Aadhaar number can introduce
others without any documentation to get the
identity number, which makes it vulnerable
to terrorism and other issues.
Sources: the hindu, wiki.

Current Affairs 68 October-2015

Paper 2 Topic: Statutory, regulatory


and various quasi-judicial bodies.

Call drops: TRAI plans to impose fines


The Telecom Regulatory Authority of India (TRAI)
has raised the penalty on telecom operators to up to
Rs. 2 lakh for poor mobile service quality,
including call drops.
TRAI said that the penalty amount would
be as high as Rs. 2 lakh if the operators are
unable to meet the benchmark set for the
quality of service in two or more
subsequent quarters.
The penalty will also be imposed if call
drops in a quarter average more than 2% of
the total traffic in a telecom circle.
As per existing norms, there is a penalty
provision of up to Rs 50,000 for the first
violation and Rs 1 lakh for subsequent
failures in case of network related quality
parameters. However, with regard to
consumer-related issues the penalty was
capped at Rs 50,000 for each violation.
These new measures act as a sufficient
deterrent against prolonged non-compliance
and may further improve the quality of
service in a time-bound manner.
What is a Call Drop?
There is no standard definition of a dropped
call. In telecommunications, it referes to the
telephone calls which, due to technical
reasons, were cut off before the speaking
parties had finished their conversation and
before one of them had hung up (dropped
calls).
There are many reasons why a call drops,
including network infrastructure, spectrum
allocation, traffic, as well as the handsets
that consumers use.
About TRAI:
It is the independent regulator of the
telecommunications business in India.
It was established in 1997 by an Act of
Parliament to regulate telecom services and
tariffs in India.
In January 2000, TRAI was amended to
establish the Telecom Disputes Settlement
Appellate Tribunal (TDSAT) to take over
the adjudicatory functions of the TRAI.
The TDSAT was set up to resolve any
dispute between a licencor and a licensee,
between two or more service providers,

between a service provider and a group of


consumers. In addition, any direction, TRAI
orders or decisions can be challenged by
appealing to TDSAT.
sources: the hindu, trai.
Paper 2 Topic: Government policies
and interventions for development in
various sectors and issues arising out of
their design and implementation.

Dr.Abdul Kalams birth place


Rameswaram declared as AMRUT town
Rameswaram town in Tamil Naidu has been
included in Atal Mission for Rejuvenation and
Urban Transformatin (AMRUT) for providing
basic urban infrastructure. This was announced on
the occasion of 84th birth anniversary of former
President Dr.APJ Abdul Kalam.
The Ministry of Urban Development has so
far identified 497 cities and towns under
Atal Mission under which basic
infrastructure is to be developed in 500
cities and towns.
Rameswaram is the 498th to be included in
the Mission. With the addition of
Rameswaram, Tamil Nadu would have 32
cities and towns under Atal Mission.
AMRUT:
AMRUT is the new avatar of the Jawaharlal Nehru
National Urban Renewal Mission (JNNURM).
AMRUT adopts a project approach to
ensure basic infrastructure services relating
to water supply, sewerage, storm-water
drains, transportation and development of
green spaces and parks with special
provision for meeting the needs of children.
Under this mission, 10% of the budget
allocation will be given to states and union
territories as incentive based on the
achievement of reforms during the previous
year.
AMRUT will be implemented in 500
locations with a population of one lakh and
above.
It would cover some cities situated on stems
of main rivers, a few state capitals and
important cities located in hilly areas,
islands and tourist areas.
Under this mission, states get the flexibility
of designing schemes based on the needs of
identified cities and in their execution and
monitoring.

Current Affairs 69 October-2015

States will only submit state annual action


Plans to the centre for broad concurrence
based on which funds will be released. But,
in a significant departure from JNNURM,
the central government will not appraise
individual projects.
Central assistance will be to the extent of
50% of project cost for cities and towns
with a population of up to 10 lakhs and onethird of the project cost for those with a
population of above 10 lakhs.
Under the mission, states will transfer funds
to urban local bodies within 7 days of
transfer by central government and no
diversion of funds to be made failing which
penal interest would be charged besides
taking other adverse action by the centre.
sources: pib.
Paper 3 Topic: Infrastructure,
railways.

Alternate Train Accommodation Scheme


VIKALP
The Rail Ministry has announced a new scheme,
called VIKALP, that would allow wait-listed
passengers of a train to opt for confirmed
accommodation in alternate trains.
The Alternate Train Accommodation
Scheme (ATAS), also called VIKALP, will
come into effect beginning 1st November
on a pilot basis for six months on DelhiLucknow and Delhi-Jammu routes for
tickets booked online.
The scheme has been launched with a view
to provide confirmed accommodation to
waitlisted passengers and also to ensure
optimal utilisation of available
accommodation
In this scheme, waitlisted passengers of a
train can opt for confirmed accommodation
in alternate trains.
The ministry has also clarified that opting for
VIKALP does not mean confirmed berth will be
provided to passengers in alternate train as it is
subject to train and berth availability. Also, once
confirmed in alternate train, cancellation charges
will apply as per the berth and train status in
alternate train. As per the scheme, boarding and
terminating station might change to nearby cluster
stations.
sources: pib.

Paper 1 Topic: Art and culture.

Buddhist monk comes to Sannati looking


for Ashokas tomb
Buddhist monk Bhante Tissavro, who heads the
Bodh Gaya-based Budh Avsesh Bachao Abhiyaan
(Save Buddhist Relics Campaign), has landed in
Kalaburagi district in his search for the tomb of
Emperor Ashoka, one of the greatest propagators of
Buddhism in India and elsewhere.
The monk is on a visit to one of the
important Buddhist sites in India at Sannati
in Kalaburgi district in Karnataka.
The monk said that his research over the
years and visit to several Buddhist centres
in India and abroad had made him suspect
that Ashoka, who had visited Sannati during
his second sojourn to south India, had died
there. He says Ashokas tomb could be
somewhere among the mounds in the
Sannati site.
Historians are silent on whether Ashoka
returned to north India after his second visit
to the south, giving credence to the belief that
Ashoka may have died at Sannati.
Sannati:
Sannati is located on the banks of the
Bhima in Chittapur in Karnataka.
It is a well-known Buddhist site.
The only available sculpture of Emperor
Ashoka, in a limestone relief, along with his
consort, was found here. It was sculpted on
the orders of an unnamed grandson of
Ashoka.
Excavation by the Archaeological Survey of
India is on at this site.
A stupa and many Buddhist relics and
sculptures in limestone and sandstone have
already been found here.
The Karnataka State government has
formed Sannati Development Authority to

Current Affairs 70 October-2015

develop the region.


sources: the hindu.

b) 2,3 and 4 Only


c) 1,2 and 4 Only
d) All the above
Ans: d.
http://pib.nic.in/newsite/PrintRelease.aspx?
relid=128406

1. Question

Consider the following statements about


Mdecins Sans Frontires (MSF):
1. It is an independent French medical
humanitarian organisation
2. It delivers emergency aid to people only
in areas affected by armed conflict
Which of the above statements is/are correct?
a) 1 only
b) 2 only
c) Both
d) None
Ans: d.
http://www.msf.org/about-msf

http://www.thehindu.com/todays-paper/tpinternational/msf-withdraws-operations-inkunduz/article7724336.ece

2. Question

Which of the following Government initiatives


(act/schemes/campaigns) are associated with
pest management?
1. Grow Safe Food campaign
2. Monitoring of Pesticide Residues at
National Level scheme
3. Strengthening and Modernization of
Pest Management Approach in India
scheme
4. Insecticide Act, 1968
Select the correct answer using the codes given
below:
a) 1,2 and 3 only

3. Question

Curcumin is
a) Another name for Cumin (Jeera),
which is beneficial against a wide set of
ailments
b) A new spice discovered in Munnar
(Kerala), and is regarded as a member of
the Cumin (Jeera) family
c) A component of turmeric, which is
beneficial against a wide set of ailments
d) A locally known and used spice in
Munnar (Kerala), recently discovered to
be a cure for dengue fever
Ans: c.
http://www.thehindu.com/sci-tech/health/medicineand-research/curcumin-a-wonder-drug-inwaiting/article7719513.ece

4. Question

Consider the following statements:


Assertion (A): There are numerous thin copper
strands inside an electric wire instead of a single
thick copper strand
Reason (R): Alternating current (AC) has a
tendency to flow along the surface of any
conductor rather than uniformly across the
cross-section of a conductor
Which of the above statements is/are true?
a) A and R both are true, and R is the
correct explanation for A
b) A and R both are true, and R is the
NOT the correct explanation for A
c) A is correct, R is incorrect
d) A and R both are incorrect
Ans: a.
http://www.thehindu.com/sci-tech/science/whyare-there-numerous-thin-copper-strands-inside-anelectric-wire-instead-of-a-single-copper-strandthat-is-as-thick/article7719517.ece
5 Question
Consider the following statements about the
Swiss challenge method:
1. The Supreme Court of India has
approved it as a way of awarding
contracts
2. This method can be applied also for
sectors that are not covered under the

Current Affairs 71 October-2015

Public-Private-Partnership (PPP)
framework
3. The original proposer has the right of
first refusal
Which of the above statements is/are incorrect?
a) 1 Only
b) 1 and 2 Only
c) 2 and 3 Only
d) All are correct
Ans: d.
http://www.businessstandard.com/article/economy-policy/indianislands-to-be-developed-under-swiss-challengemodel-115100500050_1.html

http://www.livemint.com/Politics/HOCSnmCWarO
4hpYglBsHBP/What-is-the-Swiss-ChallengeMethod.html
6. Question
Which of the following statements about Indias

new gas pricing policy is/are correct?


1. The domestic gas price is the weighted
average price of four global benchmarks,
one of which is the US-based Henry hub
2. The gas pricing formula does not apply
to those of us who use LPG cylinders
Select the correct answer using the codes given
below:
a) 1 only
b) 2 only
c) Both 1 and 2
d) Neither 1 or 2
Ans: c.
http://www.thehindu.com/opinion/editorial/gaspricing-and-some-issues/article7723290.ece

http://www.thehindubusinessline.com/opinion/colu
mns/all-you-wanted-to-know-about-gas-pricingformula/article7074268.ece

Insights into Editorial: An assertion of primacy


17 October 2015
It is widely accepted fact that the present system of judicial appointments that was put in place in 1993
is deeply unsatisfactory and there is a need for significant legislative reform. However, a five-judge
supreme court Bench recently struck down the Constitution (99th Amendment) Act, 2014, by which
the government intended to establish a National Judicial Appointments Commission to select members
of the higher judiciary.
Why was it struck down?
The composition of the NJAC, especially the inclusion in it of the Union Law Minister and two
eminent persons was not widely appreciated. According to the court, the inclusion of the Law
Minister in the body impinged on the doctrine of separation of powers.
The NJAC act gave any two members a veto over all decisions, raising the question whether the
executive representatives could overrule the judicial members.
The court also felt that this new institutional mechanism to appoint judges impinged on the
independence of the judiciary, a basic feature of the Constitution.
The clauses provided in the amendment are also inadequate to preserve the primacy of the judiciary.
And even the Attorney General, representing the government of India, could not convince the court
that the amendment, along with the NJAC Act, was aimed at restoring the system of checks and
balances which, according to the government, was lost after the Supreme Court created the collegium
scheme of appointments.
How is this decision being viewed?
To some it may appear that striking down a Constitution amendment passed unanimously in both
Houses of Parliament and ratified by 20 State Assemblies amounts to negating the peoples will.
However, it should be noted that the judiciary remains the sole authority to decide whether a law violates the
basic structure of the Constitution.
What is the NJAC?
The National Judicial Appointments Commission (NJAC) is a constitutional body proposed to replace the
present Collegium system of appointing judges.
Composition of the NJAC:
Current Affairs 72 October-2015

National Judicial Appointments Commission (NJAC) is a six-member panel headed by the Chief Justice of
India, and includes two senior-most Supreme Court judges, the Union Minister of Law and Justice and two
eminent persons nominated by a committee comprising the Prime Minister, the CJI and the Leader of the
Opposition.
What is the Collegium system?
The Collegium system is one where the Chief Justice of India and a forum of four senior-most judges of the
Supreme Court recommend appointments and transfers of judges. However, it has no place in the Indian
Constitution. The system was evolved through Supreme Court judgments in the Three Judges Cases
(October 28, 1998).
Why is Collegium system being criticized?
The Central government has criticized it saying it has created an imperium in imperio (empire within
an empire) within the Supreme Court.
The Supreme Court Bar Association has blamed it for creating a give-and-take culture, creating a
rift between the haves and have-nots. While politicians and actors get instant relief from courts, the
common man struggles for years for justice.
The collegium system also lacked transparency and was also susceptible to abuse.

17 October 2015
Paper 3 Topic: Indigenization of technology
and developing new technology.

Indias subsonic Nirbhay missile fails


again
Nirbhay, the subsonic cruise missile developed by
the Defence Research and Development
Organisation (DRDO), has failed again.
It was recently test fired in Odisha. Although
the take off was successful amid repeated
disruptions of countdown, the missile missed
the target 11 minutes after it was test fired.

The missile had a range of 750-1,000 km. But


it nosedived after covering 128 km in the Bay
of Bengal.
The missiles first test on March 12, 2013
had also failed as it too fell after 20 minutes

of flight. The second test on October 17,


2014 was also not up to the mark as it could
not maintain a low height.
About Nirbhay:
Nirbhay is an all-weather low-cost longrange cruise missile with stealth and high
accuracy. The missile has a range of more
than 1000 km. It weighs about one tonne and
has a length of 6 metres.
Its relatively slow flight speed allows it to
navigate its way precisely to the target.
The Nirbhay cruise missile is an Indian
version of the American Tomahawk.
The missile is capable of being launched
from multiple platforms on land, sea and air.
In particular, Nirbhay is being adapted for
the Indo/Russian Su-30MKI. The missile is
capable of carrying nuclear warheads.
The missile is also capable of flying at
different altitudes ranging from 500 m to 4
km above the ground and can also fly at low
altitudes to avoid detection by enemy radar.
A key hurdle to developing a long-range cruise
missile like the Nirbhay is the Missile Technology
Control Regime (MTCR), which forbids signatory
countries from assisting or providing technology to
any other country developing a cruise missile with a
range of 300 km or more.
sources: the hindu, wiki.
Paper 2 Topic: Bilateral, regional and
global groupings and agreements
involving India and/or affecting Indias
interests.

Current Affairs 73 October-2015

Pakistan, Russia sign gas pipeline pact


Pakistan and Russia have signed an agreement to
build a gas pipeline stretching hundreds of
kilometres from Karachi on the Arabian Sea to the
eastern city of Lahore.
Details:
The project would be built by Russian
company RT Global Resources a part of
Russian state corporation Rostec.
It will be a 1,100-kilometre (680-mile)
pipeline, with a capacity of 12.4 billion cubic
metres (438 billion cubic feet) per year.
It will connect liquefied natural gas (LNG)
terminals in Karachi with those in Lahore.
Russia will invest about $2 billion in the
pipeline and its first phase is expected to be
completed by December 2017.
The pipeline will be operated by the
manufacturer for 25 years before being
transferred to the Pakistani government.
sources: the hindu.
Paper 2 Topic: Government policies
and interventions for development in
various sectors and issues arising out of
their design and implementation.

SC Bench strikes down NJAC Act as


unconstitutional and void
The Supreme Court has rejected the National
Judicial Appointments Commission (NJAC) Act and
the 99th Constitutional Amendment which sought to
give politicians and civil society a final say in the
appointment of judges to the highest courts.
It held that the collegium system, as it
existed before the NJAC, would again
become operative.
The court declared that the judiciary cannot
risk being caught in a web of indebtedness
towards the government.
It is after 35 years that a constitutional
amendment has been quashed by the top
court.
Observations made by the court:
The amendment to constitute the NJAC
sought to trample upon the primacy of the
judiciary and the role of the CJI, who would
be reduced to an individual figure from an
institutional head in the panel that has the
Law Minister and two eminent persons, as its
other members.
The sensitivity of selecting judges is so
enormous, and the consequences of making

inappropriate appointments so dangerous,


that if those involved in the process of
selection and appointment of judges to the
higher judiciary, make wrongful selections, it
may well lead the nation into a chaos of
sorts.
The appointment of judges, coupled with
primacy of judiciary and the CJI, was part of
the basic structure of the Constitution and
that the parliament had no power to tinker
with this structural distribution.
The Union government had previously argued that
NJAC represented the will of the people. However,
rejecting the Centres argument, court noted that
the will of the people is the Constitution while the
Parliament represents the will of the majority at a
given point of time which is subordinate to the
Constitution.
NJAC:
NJAC was a proposed body responsible for
the appointment and transfer of judges to the
higher judiciary in India. It sought to replace
the collegium system of appointing the
judges of Supreme Court and 24 High
Courts.
The 99th Constitutional Amendment Act and
the NJAC Act had proposed that
appointments be done by a six-member body,
headed by the Chief Justice of India, and
including two seniormost SC judges, the
Union Law Minister and two eminent
persons. These two would be selected by a
panel including the Prime Minister, the CJI
and the leader of the largest Opposition party
in the Lok Sabha.
sources: the hindu.
1. Question

Which of the following statements associated


with Minorities (or Minority) in India are
incorrect?
1. Their rights have been spelt out in the
Constitution
2. The Constitution does not define the term
3. Directive Principles of State Policy
includes provisions that have significant
implications for them
Select the correct answer using the codes given
below:
a) 1 and 2 only
b) 2 only
c) 3 Only
d) All are correct
Ans: d.

Current Affairs 74 October-2015

http://www.thehindu.com/news/national/minoritiespanel-sends-notice-on-dadrilynching/article7723354.ece

http://ncm.nic.in/Constitutional_provisions.html

2. Question

Which of the following statements about the


National Commission for Minorities is/are
correct?
1. It is a statutory body
2. Only 5 communities, viz. Muslims,
Christians, Sikhs, Buddhists, Zoroastrians
(Parsis) have been notified as minority
communities
Select the correct answer using the codes given
below:
a) 1 only
b) 2 Only
c) Both
d) None
Ans: a.
http://www.thehindu.com/news/national/minoritiespanel-sends-notice-on-dadrilynching/article7723354.ece
The Union Government set up the National
Commission for Minorities (NCM) under the
National Commission for Minorities Act, 1992. Six
religious communities, viz; Muslims, Christians,
Sikhs, Buddhists, Zoroastrians (Parsis) and Jains
have been notified as minority communities by the
Union Government.

3. Question

Which of the following are components of the


National Crop Insurance Programme?
1. Modified National Agricultural Insurance
Scheme
2. Weather Based Crop Insurance Scheme
3. Coconut Palm Insurance Scheme
4. National Agricultural Insurance Scheme
Select the correct answer using the codes given
below:
a) 1,2 and 3 Only
b) 2,3 and 4 Only
c) 1,2 and 3 Only
d) All are correct
Ans: a.
A new central sector scheme, National Crop
Insurance Programme (NCIP) was introduced by
merging Modified National Agricultural Insurance
Scheme (MNAIS), Weather Based Crop Insurance
Scheme (WBCIS) and Coconut Palm Insurance
Scheme (CPIS) throughout the country from Rabi
2013-14.

http://pib.nic.in/newsite/PrintRelease.aspx?
relid=124296

http://www.thehindu.com/todays-paper/tpnational/drones-to-help-gauge-cropdamage/article7727876.ece

4. Question

Consider the following statements about Crop


Cutting Experiments associated with Crop
Insurance:
1. It is a sample survey approach, which
provides an estimate of crop yield in an
area (it can be the area of a state or the
entire country)
2. Adoption of this method has resulted in
delayed payment of claims to farmers
Which of the above statements are incorrect?
a) 1 Only
b) 2 Only
c) Both 1 and 2
d) Neither 1 or 2
Ans: d.
http://www.thehindu.com/todays-paper/tpnational/drones-to-help-gauge-cropdamage/article7727876.ece

This definitely does not have to be read, but


statement 1 has been framed using information from
this link:
http://www.iasri.res.in/ebook/TEFCPI_sampling/DE
MONSTRATION%20OF%20CROP%20CUTTING
%20EXPERIMENT%20ON%20A%20FARMER
%E2%80%99S%20FIELD.pdf

5. Question

GIAN or Global Initiative for Academics


Network is an initiative to
a) Attract the best foreign academics to
Indian Universities of Excellence
b) Provide top-performing students from
Indian Universities the opportunity to
interact with academics from the USA
c) Promote student-exchange programmes
between Indian and German Universities
d) Promote teachers from Indian
Universities to develop pedagogical
methods like their western counterparts
Ans: a.
The Union Cabinet has approved a program titled
Global Initiative of Academic Networks (GIAN) in
Higher Education aimed at tapping the talent pool of
scientists and entrepreneurs, internationally (and
not only western or a specific country) to
encourage their engagement with the institutes of

Current Affairs 75 October-2015

Higher Education in India so as to augment the


countrys existing academic resources, accelerate the
pace of quality reform, and elevate Indias scientific
and technological capacity to global excellence.

http://www.thehindu.com/news/national/german-tobe-taught-again-in-centralschools/article7727600.ece

http://mhrd.gov.in/sites/upload_files/mhrd/files/PRE
SS_RELEASE_31-12-2014.pdf

6. Question

Which of the following statements associated


with subsidies in India are true?
1. There are subsidized services as well
2. Minimum Support Prices (for crops) can
hurt poor households
3. Cross-subsidisation in the railways has

contributed to increased traffic congestion


and emissions
Select the correct answer using the codes given
below:
a) 1 and 3 only
b) 1 and 2 only
c) 2 and 3 Only
d) All the above
Ans: d.
Statement 3 confirms that statement 1 is true. You
can eliminate option b at least. For understanding
how the other two statements are correct (statement
2 and the assertion in statement 3), refer to page 55:
Economic Survey 2014-15, Volume 1

http://www.thehindu.com/business/subsidies-slideto-16-per-cent-of-gdp/article7727208.ece

Insights into Editorial: Judging the Judgemaker


19 October 2015
Judicial appointments are vital in the running of a constitutional democracy. For long, the executive
and legislature have been seeking to have a say in the process of selecting judges.
History:
Pre 1993
Till 1993, judges were appointed by the executive in consultation with the judiciary. This was not always
transparent. Sometimes, governments indulged in rank favouritism. The judicial voice was often neither
dominant nor decisive. This situation prompted Ram Jethmalani to famously remark, There are two kinds
of judges, those who know the law and those who know the law minister.
Post 1993
In 1993, a quiet declaration of judicial independence occurred. Justice J.S. Vermas judgment in the
Supreme Court Advocates on Record case, gave the Chief Justice and senior judges of the
Supreme Court and the High Courts the power of making almost binding recommendations, for
future appointments of judges in the constitutional courts.
According to this judgement, whenever a vacancy arose, it would be filled by someone pre-approved
by the judges and the executive could only demur in the appointment if cogent grounds existed. If,
despite executive demur, the judges insisted on the appointment, the executive would have to confirm
it. Thus, the collegium system was established.
In 1998, on a presidential reference, the Supreme Court defined the collegium. Under the Collegium
system, the Chief Justice of India would consult the four senior most judges of the Supreme Court for
Supreme Court appointments and two senior-most judges for high court appointments.
Arguments against the collegium system:
It is seen as a closed-door affair without a formal and transparent system. Judges, hopeful of going
higher, have to please the members of the collegium.
This system overlooks several talented junior judges and advocates.
Sometimes, collegiums gets stymied, when old rivalries between its members see each others
favourites getting vetoed.
Sometimes collegium meetings become examples of bargaining within the collective, and consensus
emerging from a division of the spoils.
However, the collegium ensures that judges are not beholden to any politician.
Current Affairs 76 October-2015

What does the Constitution actually prescribe?


Article 124 deals with the appointment of Supreme Court judges. It says the appointment should be
made by the President after consultation with such judges of the High Courts and the Supreme Court
as the President may deem necessary. The CJI is to be consulted in all appointments, except his or her
own.
Article 217 deals with the appointment of High Court judges. It says a judge should be appointed by
the President after consultation with the CJI and the Governor of the state. The Chief Justice of the
High Court concerned too should be consulted.
NJAC:
Parliament of India amended the Constitution and brought about the 99th Amendment to provide for the
NJAC after the new government came to power. Subsequent ratification of 20 States was also obtained and it
seemed that the collegium was history.
However, soon petitions were filed challenging the constitutional amendment. The Supreme Court also
struck down the 99th Amendment recently.
Why it was struck down?
The court concluded that the NJAC did not provide an adequate representation, to the judicial
component and clauses (a) and (b) of Article 124A(1) are insufficient to preserve the primacy of the
judiciary in the matter of selection and appointment of Judges.
The court also held that Article 124A(1) is ultra vires the provisions of the Constitution, because of
the inclusion of the Union Minister in charge of Law and Justice as an ex officio Member of the
NJAC.
The clause also impinged upon the principles of independence of the judiciary, as well as,
separation of powers.
The clause which provided for the inclusion of two eminent persons as Members of the NJAC was
held ultra vires the provisions of the Constitution, for a variety of reasons.
The supreme court has reinstated the collegium as the clearinghouse of all judicial appointments to the
constitutional courts. It has now opted to take the path to reform, rather than change to an altogether new
road created by Parliament.
When the Supreme Court banned paid
donation, the government was mandated,
Paper 2 Topic: Government policies and
under the National Blood Policy (2002), to
interventions for development in various
phase out replacement donation in five
sectors and issues arising out of their design
years.
and implementation.
What has been done to address the issue?
Since very few people donate blood
Blood banks can borrow from one
regularly, hospitals ask family members of
another
patients to donate blood so as to preThe Union Health Ministry has decided to allow
emptively replace the blood withdrawn from
blood banks to borrow blood units from one another
the blood bank.
in case of a shortage.

The National and State Blood Transfusion


The Ministry took this decision as part of
Councils were also established in the 1990s
reforms to the handling of blood units in the
to promote voluntary donations.
country.
According to the Central Drug Standard Control
This decision is expected to sort out the
Organisation (CDSCO), India has 2,760 licensed
issue of shortage.
blood banks. A 2012 World Health Organisation
Earlier, donated blood could not be transferred from
(WHO) report said nine million of 12 million blood
one bank to another.
units needed annually in India were collected
Background:
through voluntary donation.
A shortage of blood units has been a
National Blood Policy:
recurring problem since the Supreme Court,
The policy aims to ensure easily accessible and
in 1998, banned paid donation.
adequate supply of safe and quality blood and blood
The government has been encouraging
components collected / procured from a voluntary
voluntary donation ever since.
non-remunerated regular blood donor in well

19 October 2015

Current Affairs 77 October-2015

equipped premises, which is free from transfusion


transmitted infections, and is stored and transported
under optimum conditions.
Transfusion under supervision of trained
personnel for all who need it irrespective of
their economic or social status through
comprehensive, efficient and a total quality
management approach will be ensured under
the policy.
Important objectives of the policy:
To reiterate firmly the Govt. commitment to
provide safe and adequate quantity of blood,
blood components and blood products.
To make available adequate resources to
develop and reorganise the blood transfusion
services in the entire country.
To make latest technology available for
operating the blood transfusion services and
ensure its functioning in an updated manner.
To launch extensive awareness programmes
for donor information, education,
motivation, recruitment and retention in
order to ensure adequate availability of safe
blood.
To encourage appropriate clinical use of
blood and blood products.
To strengthen the manpower through human
resource development.
sources: the hindu, pib.
Paper 3 Topic: Infrastructure.

Godavari to be declared national


waterway
The Centre has agreed to include the stretch of the
river Godavari from Nasik in Maharashtra to
Rajahmundry in Andhra Pradesh in the revised
National Waterways Bill, 2015.
Background:
The Telangana Government had also
recently requested the centre to include the
stretch of the Godavari from Kaleshwaram
to Bhadrachalam as part of the National
Waterway-4 from Bhadrachalam to
Rajahmundry.
The National Waterways Bill was tabled in
the parliament in May for declaring an
additional 101 inland waterways as national
waterways.
The bill was examined by the Parliamentary
Standing Committee on Transport, Tourism
and Culture and had recommended that a
revised bill be tabled during the next session

of Parliament by including the stretch of


Godavari from Nasik to Rajahmundry as
National Waterway.
Inland Waterways Authority of India (IWAI) is the
statutory authority in charge of the waterways in
India.
sources: the hindu, wiki.
Paper 2 Topic: Statutory, regulatory
and various quasi-judicial bodies.

Pension fund regulator seeks fiscal


support from Centre
The Pension Fund Regulatory and Development
Authority (PFRDA) has asked the central
government to support its ongoing move to expand
the subscriber base.
PFRDA recently achieved a new milestone
in assets under management (AUM) and
subscriber base. The AUM of its national
pension system (NPS) has crossed Rs.1trillion-mark in the first week of October at
around Rs.1,10,000 crore, while the NPS
subscriber base also crossed 1-crore-mark in
the first week of October.
The efforts of PFRDA are now targeted at
expanding the subscriber base further,
particularly in the corporate and the private
citizens segments.
It is also working towards further improving
the infrastructure for minimising the
response time. As part of this measure,
PFRDA is more closely co-ordinating with
the Centre and States whose share accounts
for over 90% of the total AUM.
PFRDA:
The Pension Fund Regulatory and Development
Authority (PFRDA) is a pension regulatory
authority which was established in 2003.
It is authorized by Ministry of Finance,
Department of Financial Services.
It promotes old age income security by
establishing, developing and regulating
pension funds and protects the interests of
subscribers to schemes of pension funds and
related matters.
It is also responsible for appointment of
various intermediate agencies such as
Central Record Keeping Agency (CRA),
Pension Fund Managers, Custodian, NPS
Trustee Bank, etc.
sources: the hindu, wiki.
1. Question

Consider the following statements about vertical

Current Affairs 78 October-2015

farming:
1. Only climbers and creepers can be grown
in this type of farming
2. Crop yield is greater when compared to
yield through conventional farming over
the same area
Which of the above statements are incorrect?
a) 1 only
b) 2 only
c) Both 1 and 2
d) Neither 1 or 2
Ans: a.
Wheat, maize and rice these things that provide
the bulk of our calories- will be very difficult to
grow on a vertical farm because you need to
accumulate a massive biomass for those crops you
might expect typically anything between 5 and 12
tonnes per hectare of grain from something like
wheat, but to do that you have to accumulate
upwards of 20 tonnes per hectare of dry weight of
plant. So its the weight of the plant which
determines whether or not it can be cultivated in
vertical farming. The crops that are likely to be
grown are high-value nutritious crops like
tomatoes, lettuces, green crops.

2. Question

Which of the following statements about


neutrinos is/are incorrect?
1. Trillions of neutrinos pass through our
body every second
2. They are created in nuclear reactions
3. They never interact with particles of other
matter
Select the correct answer using codes given
below:
a) 1 only
b) 2 and 3 Only
c) 1 and 3 Only
d) 3 Only
Ans: d.
Neutrinos are the second most bountiful particles
after photons, which carry light, with trillions of
them streaming through our bodies every second,
but their true nature has been poorly understood.
They rarely interact with particles of other matter.
http://www.bbc.com/news/science-environment34443695

3. Question

Neutrino Oscillations is
a) The movement of neutrinos in a backand-forth motion, a pendulum-like
manner through space while travelling
b) The flipping of neutrinos from one

type to another during flight


c) The flipping of a neutrino from
electron to proton and/or vice versa in
any atom
d) None of the above
Ans: b.
http://www.bbc.com/news/science-environment34443695

4. Question

The Agricultural and Processed Food Products


Export Development Authority (APEDA) is
under the
a) Ministry of Consumer Affairs, Food
and Public Distribution
b) Ministry of Agriculture
c) Ministry of Food Processing Industries
d) None of the above
Ans: d.
APEDA is under the Ministry of Commerce and
Industry.
http://www.thehindu.com/news/national/govt-tostep-up-vigil-at-ports-to-check-export-of-cowmeat/article7731341.ece

5. Question

Which of the following statements about


barefoot technicians, associated with the
Ministry of Rural Development, is/are correct?
1. To become one, any person above the age
of 21 is eligible for training
2. He/She can selected from the local SC/ST
MGNREGA worker households only
Select the correct answer using codes given
below:
a) 1 only
b) 2 Only
c) Both
d) None
Ans: b.
Only an educated individual is eligible to become
a barefoot technician, not any person.
http://pib.nic.in/newsite/PrintRelease.aspx?
relid=128470

6. Question

The Pravasi Bharatiya Divas is celebrated every


year on January 9th to mark
a) The date of return of Mahatma Gandhi
from South Africa for good
b) The date of departure of Mahatma
Gandhi to South Africa from India
c) The date when Independent India first
received a Non-Resident Indians
remittance

Current Affairs 79 October-2015

d) The date of return of indentured


labourers from the Caribbean in 1948
Ans: a.

http://pib.nic.in/newsite/PrintRelease.aspx?
relid=128468

http://www.pbd-india.com/about.html

Insights into Editorial: Aadhaar and the Right


to Privacy
October 20, 2015
Recently, the Supreme Court of India refused simultaneous applications by multiple agencies
demanding relief from the Supreme Courts interim order limiting the use of Aadhaar pending the
Courts final decision. The court has referred these government applications to a constitutional bench.
With this, on one side the court has assured Indians that a decision on their fundamental rights will not
be long delayed and on the other side it has increased pressure on the government.
The court has asked the government to address the most basic questions in a democracy governed by the
law: what are the privacy rights of its citizens; and are they protected equally, with the same justice for the
rich and the poor alike?
What has been the government saying?
The Government of India (GoI) has repeatedly been saying that it is the governments position that
Indian citizens have no constitutional right of privacy.
It argues that the poor, whose welfare is at stake in the continuance of subsidy payments and other
benefits, must be prepared to surrender their right of privacy, if any, in order to continue receiving
benefits.
How these arguments are being seen?
These arguments were sharply rejected by the supreme court, which recognises that the poor have the
same rights as the rich in any democratic society.
The governments stand may give rise to the doubt whether it is truly committed to protecting its
citizens from violations of their privacy by the unauthorised use of information provided by them.
Why there is a need to protect citizen information?
Identifying citizens for providing various services, maintaining security and crime-related
surveillance and performing governance functions, all involve the collection of information. In recent
years, owing to technological developments and emerging administrative challenges, several national
programmes and schemes are being implemented through information technology platforms, using
computerised data collected from citizens.
With more and more transactions being done over the Internet, such information is vulnerable to theft
and misuse. Therefore, it is imperative that any system of data collection should factor in privacy
risks and include procedures and systems to protect citizen information.
What should the government do?
Instead of arguing that privacy is not a fundamental right, it should assure the court that it has the
technology and systems to protect the data collected.
It should assure the citizens of India that it will do everything possible to prevent unauthorised
disclosure of or access to such data.
It should recognise all dimensions of the right to privacy and address concerns about data safety,
protection from unauthorised interception, surveillance, use of personal identifiers and bodily
privacy.
The data controller should be made accountable for the collection, processing and use to which data
are put.
Conclusion:
The governments most basic obligation is to protect its citizens rights both their right to sustenance and
their right to the privacy that enables freedom equally. It should recognise both the need for Aadhaar and
the need for stringent rules concerning access to and security of citizens biometric data, in order to preserve
Current Affairs 80 October-2015

their privacy. The Constitution does not specify right to privacy as a fundamental right, but the law on the
subject has evolved considerably in India, and privacy is now seen as an ingredient of personal liberty.
considered far more harmful in India. It has
long-term effects, but show signs of
Paper 2 Topic: Government policies and
discomfort among sensitive people
interventions for development in various
whenever consumed.
sectors and issues arising out of their design
Typical MSG complaints include: Burning
and implementation.
sensations of the mouth, head and neck
Headaches Weakness of the arms or legs
Karnataka, Gujarat lift ban on Maggi
Upset stomach Hives or other allergic-type
The governments of Gujarat and Karnataka have
reactions with the skin.
lifted the ban on Nestle Indias Maggi noodles,

Scientists have also discovered that the


becoming the first two states to do so.
Karnatakas move would allow Nestle India
compound can destroy Retina and parts of
the Brain. It can also lead to nervous
to manufacture the product at its plant in the
disorders and radical hormone fluctuations.
state. The companys other Maggi

Many studies have also shown that it is


manufacturing plants are in Goa, Punjab,
Himachal Pradesh and Uttarakhand.
particularly harmful for pregnant women and
These developments also come at a time
nursing mothers as infants and very young
children are susceptible to brain damage and
when the countrys apex food regulator
underdevelopment.
FSSAI is unlikely to challenge courtsources: the hindu, wiki.
directed retests that Maggi instant noodles
Paper 2 Topic: Government policies
cleared last week.
and interventions for development in
Gujarats food and drug control agency
various sectors and issues arising out of
removed the ban following the Bombay
their design and implementation.
High Courts verdict removing the ban
Kerala notifies stringent norms for
across the country.
Background:
transportation of livestock
The popular brand of Nestle India was
The Kerala state government has notified stringent
banned on June 5 following a directive of
rules for livestock transportation. These rules are
the Food Safety and Standards Authority of
aimed at curbing cruelty to animals.
India (FSSAI), the Central food watchdog.
These rules have been notified by the
Some Maggi noodles samples were
Ministry of Road Transport and Highways
reportedly found to contain higher-thanby amending the Central Motor Vehicles
permissible levels of monosodium glutamate
Rules 1989 and by inserting a separate rule,
(MSG).
125 E, in the principal rules.
Maggi noodles has been at the centre of
Details:
controversy since laboratory tests ordered by
Uttar Pradesh food inspectors on a batch of
maggi allegedly found eight times as much
lead as permissible. Several states had
banned the product since then.
About Monosodium glutamate (MSG):
It is one of the most common, naturally
occurring non-essential amino acid, which is
found in tomatoes, Parmesan cheese,
potatoes, mushrooms, and other vegetables
and fruits.
In the food industry, it is used as a taste
enhancer that increases the meaty, savoury
flavor of food.
According to these rules, from January 1,
Although the U.S Food and Drug
2016, the motor vehicles carrying animals
Administration recognises MSG as safe, it is
should have permanent partitions so that the

20 October 2015

Current Affairs 81 October-2015

animals being transported are carried


individually in each partition.
For cows and buffaloes the partition should
be 2 sq.m., while for horses and mares it
should be 2.25 sq. m. The partition size
should be 0.3 sq. m. for sheep and goats, 0.6
sq. m. for pigs, and 40 sq. cm. for poultry.
The specifications of the Bureau of Indian
Standards will have to be followed by the
transporter or consigner of livestock.
The motor vehicles meant for carrying
animals will not be permitted to carry any
other goods. The Regional Transport
Officers have been asked to issue special
licences for such vehicles.
At present, the transportation of animals falls under
the Prevention of Cruelty to Animals Act, 1960 that
came into force in 1978 and was amended in 2001
and 2009. The implementation of the rules had
come in for criticism as it was a non-cognizable
offence and the penalty was Rs. 50.
Implications of these new rules:
It would end the horrendous nightmare being
faced by animals while being transported by
road.
These rules will improve health of the
animals and drastically improve the
livestock transportation in the country.
However, the cost of transportation of livestock will
go up as the transporters will be able to ferry only
fewer animals in a vehicle.
sources: the hindu.
Paper 3 Topic: Conservation;
Environmental pollution

Developed worlds climate change targets


less than fair: Report
A recently released report by an NGO suggests that
almost all developing countries, including India and
China, have taken on more than their fair share of
the burden through their intended nationally
determined contributions (INDCs).
However, the report suggests that the
developed countries have taken on less than
their fair share of the burden.
The report assesses whether countries have
taken a fair burden to keep the global
temperature rise below two degrees Celsius
by the end of the century in relation to their
historical responsibilities for causing the
problem and their current capacities.
Important observations made by the report:

The US has committed to only a fifth of its


fair share and the European Union (EU) just
over a fifth in the fight against climate
change through its new targets.
The US compared to a fair share of
reduction by 12,943 million tonnes of carbon
dioxide equivalent (MTCO2e) only intended
to reduce its emissions by 2,089 MTCO2e
by 2030.
The EU had pledged to reduce its emissions
by 1,587 MTCO2e instead of a fair burden
of 7,036 MTCO2e.
China should have reduced 3,371 MTCO2e
of emissions but has offered to reduce 4,888
MTCO2e by 2030.
India could have fairly taken a burden of
reducing emissions by 54 MTCO2e but has
committed to reducing it by 280 MTCO2e.
The report took 1850 as base year to calculate the
historical emissions and to measure the reductions
in absolute terms against the base year.
What is the Burden, according to the report?
According to the report, the burden is the target the
countries have formally given to the United Nations
(UN), to be undertaken under the global Paris
climate change agreement. The target on
reducing emissions, providing finance and
technology is meant to be achieved between
2020 and 2030.
What are INDCs?
These are individual country commitments which
are expected to indicate through their form and
strength what shape any 2015 agreement might
take.
Countries across the globe have committed
to create a new international climate
agreement by the conclusion of the U.N.
Framework Convention on Climate Change
(UNFCCC) Conference of the Parties
(COP21) in Paris in December 2015.
In preparation, countries have agreed to
publicly outline what post-2020 climate
actions they intend to take under a new
international agreement, known as their
Intended Nationally Determined
Contributions (INDCs).
The INDCs combine the top-down system of
a United Nations climate agreement with
bottom-up system-in elements through
which countries put forward their
agreements in the context of their own
national circumstances, capabilities and
priorities, within the ambition to reduce

Current Affairs 82 October-2015

global greenhouse gas emissions enough to


keep global temperature rise to 2 degrees
Celsius.
The INDCs will not only contain steps taken
towards emission reductions, but also aim to
address steps taken to adapt to climate
change impacts, and what support the
country needs-or will provide to address
climate change.
In February 2015, Switzerland became the
first nation to submit its INDC to reduce
greenhouse gas emissions, later followed by
the European Union.
sources: the hindu, pib, bs.
1. Question

Which of the following is/are not some of the


recommendations made by the Bibek Debroy
Committee in its report on Indian Railways?
1. Accounting reforms
2. Streamlining recruitment processes
3. It must distance itself from non-core
activities such as running schools and
hospitals
Select the correct answer using the codes given
below:
a) 1 Only
b) 1 and 2 Only
c) 3 Only
d) All are recommendations by the
committee
Ans: d.
http://pib.nic.in/newsite/PrintRelease.aspx?
relid=128524

http://articles.economictimes.indiatimes.com/201504-02/news/60756506_1_indian-railways-interimreport-services/2

2. Question

Which of the following can be classified as


Occupational Diseases?
1. Sleep disorder due to noise pollution
2. Silicosis
3. Asbestosis
4. Pneumoconiosis
Select the correct answer using the codes given
below:
a) 1, 2 and 3 only
b) 2 and 3 only
c) 2 and 4 Only
d) All the above
Ans: d.
http://pib.nic.in/newsite/PrintRelease.aspx?
relid=128488

http://www.nioh.org/projects.html

3. Question

Which article in the Indian Constitution prohibits


trafficking in human beings?
a) Article 21
b) Article 23
c) Article 43
d) There is no such article in the Indian
Constitution
Ans: b.

4. Question

Which of the following laws do not have


provisions associated with human trafficking?
1. The Minimum Wages Act
2. The Protection of Children from Sexual
Offences Act
3. The Juvenile Traffic (Prevention) Act
4. The Sexual harassment of Women at
Workplace (Prevention , Prohibition and
Redressal) Act
Select the correct answer using the codes given
below:
a) 1 and 2 only
b) 1, 2 and 3 Only
c) 2, 3 and 4 Only
d) 3 Only
Ans: d.
There is no Juvenile Traffic (Prevention) Act. And
you certainly dont have to go through all the laws!
This is an important issue today in India and
worldwide, so having a general idea about
provisions associated with it would be of benefit.
You can go through the link provided below for the
list of laws associated with trafficking.
You can guess that the 2nd law mentioned above
could have provisions associated with trafficking.
There you can eliminate 3 options. You will be left
only with option d.
http://pib.nic.in/newsite/PrintRelease.aspx?
relid=128489

http://stophumantraffickingmha.nic.in/forms/Sublink1.aspx?lid=256

5. Question

The balance sheet syndrome with Indian


characteristics creates a web of difficult
challenges that could hold back private
investment. In this context, consider the
following statements:
1. Unlike many other countries with high
debt to equity ratios currently, Indias
debt is almost exclusively financed by

Current Affairs 83 October-2015

public sector banks


2. Public sector in India is exposed to
corporate risk
Which of the above statements are incorrect?
a) 1 Only
b) 2 Only
c) Both 1 and 2
d) Neither 1 or 2
Ans: d.
Page 72: Economic Survey 2014-15, Volume 1
http://www.thehindu.com/news/national/bankskilled-road-sector-official/article7735927.ece

6. Question

Which among the following are some of the


flaws in the existing design of Public-PrivatePartnerships?
1. Unmanageable risks, e.g., traffic risk in
highways, are transferred to
concessionaires
2. Compulsory allocation of both,
construction and maintenance
responsibilities to the concessionaire
3. The Government has no leverage in case
of non-performance by concessionaire
4. There are no ex-post structures for renegotiation of contracts
Select the correct answer using the codes given
below:
a) 1 and 2 only
b) 1 and 3 Only
c) 2 and 4 Only
d) All of the above
Ans: b.
There are no ex-ante structures for re-negotiation of
contracts.
Even if you havent read the information in Box 4.1,
Volume 1 of the Economic Survey (2014-15), you
can hazard a guess. Statement 2 can be eliminated,
and you will be left only with option b.
It is better to continue combining construction and
maintenance responsibilities to incentivise building
quality. In many projects, especially highways,
maintenance costs depend significantly on
construction quality. If a single entity is responsible
for both construction and maintenance, it takes
lifecycle costs into account. Separating these
responsibilities provide an incentive to increase
profits by cutting corners during construction.
Suggestions to let the public sector build assets and

have the private sector maintain and operate them


ignore this linkage.
http://www.thehindu.com/news/national/bankskilled-road-sector-official/article7735927.ece

7. Question

Consider the following statements about the


1951 United Nations Convention relating to the
status of refugees:
1. It is the key legal document in defining
who is a refugee, their rights and the legal
obligations of states
2. India is not a signatory to the convention
Which of the above statements are correct?
a) 1 only
b) 2 Only
c) Both 1 and 2
d) Neither 1 or 2
Ans: c.
Though India offers asylum to a large number of
refugees, it is not a signatory to the 1951 Refugee
Convention.
India to oppose uniform migration policy at
BRICS meet in Sochi Page 15, The Hindu (8th
September)

http://www.unhcr.org/pages/49da0e466.html

8. Question

Which of the following statements about the


Micro Solar Dome is/are correct?
1. It is a zero-fuel device requiring
maintenance, the frequency of which
depends upon its usage
2. It captures sunlight which can be
transmitted through Polyvinyl Chloride
pipes
Choose the correct answer using the codes given
below:
a) 1 Only
b) 2 Only
c) Both 1 and 2
d) Neither 1 or 2
Ans: b.
http://pib.nic.in/newsite/PrintRelease.aspx?
relid=128557

http://www.thehindu.com/news/cities/kolkata/trappi
ng-sunshine-letting-it-flow-in-darkhomes/article7096493.ece

Current Affairs 84 October-2015

Insights into Editorial: Time to Refresh Afghan


Relationship
21 October 2015
Present situation in Afghanistan, Central and West Asia makes it the right time for India to take a fresh
initiative that reaches out to old friends in northern Afghanistan along with anti-Taliban elements in the
southern parts. The changing equations of balance of power in Afghanistan and the current status of
Talibans capacities in southern Afghanistan have given India this opportunity. Following Talibans takeover
of Kunduz, the geopolitical situation on Afghanistans borders is rapidly evolving.
Why should Afghans neighbours be worried about this situation?
It is because the Taliban in Kunduz comprises not only Afghans, but an ensemble of Uzbeks, Tajiks,
Turkmen, Kyrgyz, Kazakhs, Uighurs, Chechens, Dagestanis along with the Al Qaeda and Pakistanis.
Why should Russia be worried?
The Russian government is concerned that local Daesh (Islamic State) elements, comprising mainly
Central Asian jihadis , could, along with the Taliban, further fuel instability.
There is also fear that the Daesh could retaliate against Russian targets in response to the happenings
in Syria.
Russia loses 50,000 youth to narcotics, which makes its way through the porous Central Asian border.
Why the US should be worried?
The U.S. had recently decided to withdraw its last troop from Afghanistan. But, following the Kunduz
incident, the US decided to wait for some more time. The halt in troop withdrawal until 2017 is meant to buy
time until the U.S. finds a better roadmap to peace in Afghanistan. It is said to be examining various
possibilities for securing peace.
Iran is also worried because of the following reasons:
The instability in Afghanistan could give a boost to the Daesh.
An unstable Afghanistan will also lead to an increase in the illicit drug trade on Irans eastern border.
Instability could lead to an influx of refugees.
Current scenario:
In the aftermath of the Kunduz attacks, it has not been possible for Taliban to regain control over major cities
in its traditional stronghold of southern Afghanistan. Kunduz was meant to legitimize the new Taliban under
Mansours leadership. However, its operations in the south Afghanistan give the appearance of not being as
robust. Kandahar has been the traditional base of Taliban. However, in recent times it has not been able to
retain its control over the Kandahar and other regions. Some groups within the Taliban are opposed to the
new leadership.
Indias role:
India has played a pivotal role in the resistance against the Taliban through the Northern Alliance in 90s.
That role was predicated on the existence of facilities and friends in the region. Now, India needs to reestablish contact not just with the old friends in the region but also factions within the Taliban and the antiTaliban forces in the south, to help Afghanistan gain strategic autonomy. It is also imperative for India that
Afghanistans southeastern regions are prevented from exporting terror into India. Given the situation, India
needs to come out with a new and coordinated initiative to help the Afghanistan.
Conclusion:
The geopolitical changes around Afghanistan coupled with the ongoing organizational struggles of the
Taliban provide an opportunity for Russia, Iran and India to reassess their roles in Afghanistan. This will
materialize only in co-ordination with the Afghanistan unity government for which it will first need to
convince itself that over-reliance on a Pakistan-led solution is a non-starter.

21 October 2015
Paper 1 Topic: Poverty and developmental
issues, urbanization

Gap widening between rural and urban


India
According to the recently released data by HSBC
global research, the gap between rural and urban
inflation has more than doubled over the last one

Current Affairs 85 October-2015

year. Although the inflation has been slowing both


in rural and urban areas of the country, there is a
widening difference between the two as rural
inflation is decelerating at a much slower pace.
Other important observations:
Inflation has fallen strikingly over the last
several months, but the gains are not equally
distributed.
Rural inflation is running higher than urban
inflation and its underlying trend is higher
than the RBIs target.
Urban inflation momentum has slowed to
4.5%, which is lower than the Reserve Bank
of Indias target level of 6%. The trend in
price gains in rural India, however, is
running at 6.5%.
The difference between rural and urban
inflation is most stark for fuel and
transportation, followed by core and to a
lesser extent food.
Despite two successive years of drought,
overall food inflation in India has remained
tepid, as low global prices have made it
possible to import food products that are in
short supply. Rural Indians, however, do not
seem to have benefitted as much from food
imports as their urban counterparts.
Why there is excess inflation in rural areas?
The excess inflation in rural India is arising
from food, fuel, transportation as well as
from core inflation.
Rural India has some structural
disadvantages vis-a-vis urban India. Urban
India is benefitting from lower global prices
while rural India, partly because of its
structural ailments, is not being able to
partake with equal vigour.
Structural bottlenecks in rural India are
harsher. Transport networks are also sparser
and distribution channels are insufficient.
Higher inflation for perishables such as meat
and fish point towards lack of availability of
cold storage facilities.
What should be done to improve the situation?
Higher investment in rural infrastructure and
meaningful agricultural reforms are needed to make
growth weather proof and put it on a higher, more
sustainable path to enable the RBI to meet its 4%
inflation target sustainably.
sources: the hindu.
Paper 3 Topic: Infrastructure.

Road developers may shy away from


capital infusion
The Union Cabinet has approved a one-time capital
infusion by the National Highways Authority of
India to get stalled road projects moving.
This infusion of funds would be a one-time
dispensation for all such projects that have
been languishing as on November 1, 2014.
All such cases and the amount of bridge
fund required in each case shall be approved
by NHAI, on a case to case basis.
However, according to credit rating agency India
Ratings and Research (Ind-Ra), lenders and
developers may not be keen on taking advantage of
this infusion. Why?
This is because the proposal comes with the
clause of first charge by NHAI on the
toll/annuity receivables of these projects
(structural subordination) over the senior
lenders debt service.
In structural subordination, the toll/annuity
receivables of the project would be ensured
for NHAI through execution of the tripartite
agreements between the senior lender,
concessionaire and the NHAI.
What else has the agency said?
The rating agency has also made it clear that
simply pumping in funds will not be enough
to get all projects moving. It is because
funding shortfall may not be the only reason
for languishing projects, but also delays in
getting the appropriate approvals and
clearances from various government
agencies.
One-time funding may not help all stranded
projects, but only those that needed
financing.
NHAI:
The National Highways Authority of India (NHAI)
is an autonomous agency of the Government of
India, responsible for management of a network of
over 70,000 km of National Highways in India. It is
a nodal agency of the Ministry of Road Transport
and Highways.
The NHAI was created through the
promulgation of the National Highways
Authority of India Act, 1988. In February
1995, the Authority was formally made an
autonomous body.
It is responsible for the development,
maintenance, management and operation of
National Highways.

Current Affairs 86 October-2015

sources: the hindu.


Paper 2 Topic: Health.

Prioritising infection control in Telangana


The Telangana government has directed teaching
hospitals in the state to start implementing measures
to control healthcare associated infections.
It was recently found that government
hospitals in Telangana have fallen behind in
implementing good practices to curb
infections.
Various measures taken by the state government:
Control committees:
To address concerns related to infections, the state
government has directed the government hospitals
to set up infection control committees to provide a
forum for multidisciplinary inputs, which would be
responsible for planning, implementation and
resource allocation of all matters related to infection
prevention and control.
What would these committees do?
The committees would develop infection
control policies and environmental
management practices to ensure adherence
to standard infection control precautions.
They also review and approve annual plan
prepared by the infection control team,
review epidemiological surveillance data
and identify areas of intervention.
The other responsibilities of such a
committee is to ensure capacity building for
infection prevention, control and safety,
prepare standard operating procedures for
various medical departments and frequently
review infectious risks associated with
medical equipment.
Hospital Infection Control Team (HICT):
The government has also asked each
government hospital in Telangana to
constitute a Hospital Infection Control Team
(HICT) to implement the infection control
measures and provide solutions on infection
control on 24 hours basis.
The team would also be responsible for
maintenance of medical equipment,
investigating outbreaks of infections, take up
surveillance, advice staff on waste
management, develop antibiotic policies and
train hospital staff.
Specialized staff:
It has been decided to have a dedicated
Infection Control Officer (ICO), a
microbiologist or epidemiologist, in-charge

of infection control practices, provide trends


for hospital acquired infections to patient
care units, and investigate outbreaks of viral,
bacterial or fungal ailments and training.
It has also been decided to have an Infection
Control Nurse (ICN), a nurse with an
academic education and practical training to
act as a specialist advisor in all aspects
relating to infection control.
An exclusive biomedical waste management
committee in hospitals that would be in
charge of implementing good practises in
management of hospital waste will also be
setup.
sources: the hindu.
Paper 1 Topic: Art and culture:

A festival to remember
The Telangana government recently celebrated the
eagerly awaited Festival of Flowers
(Bathukamma) for which the State is known.
About the festival:

The festival known as Bathukamma is


Telanganas floral festival, traditionally
celebrated by women across the state.
Every year this festival is celebrated as per
Telugu version of Hindu calendar in the
Bhadrapada Amavasya, also known as
Mahalaya Amavasya, usually in September
October of Gregorian calendar.
The festival is celebrated for nine days
during Durga Navratri.
It represents cultural spirit of Telangana.
In Telugu, Bathukamma means Mother
Goddess come Alive.
It is the state festival of Telangana.

Current Affairs 87 October-2015

sources: the hindu, wiki.


1. Question

The Central Government has recently given tax


sops to sugar mills in order to boost ethanol
production. With reference to ethanol, which of
the following statements are incorrect?
1. Ethanol can be derived not only from
sugarcane, but also corn and sorghum
2. In India, 10 per cent blending of ethanol
with petrol is mandatory
3. Bootleggers mix it with country-made
liquor to increase its potency
Select the correct answer using codes given
below:
a) 1 and 2 Only
b) 2 and 3 Only
c) 1 and 3 Only
d) All are correct
Ans: b.
Ethanol is a renewable, domestically produced
alcohol fuel made from plant material, such as corn,
sugar cane, or grasses. In India, up to 10 per cent
blending of ethanol with petrol is allowed.
Although 10 per cent blending of ethanol with
petrol is allowed, oil marketing companies (OMCs)
will achieve 3.5 per cent in 2014-15 season.
Bootleggers mix methanol with country-made
liquor to increase its potency.
http://mrunal.org/2012/10/economy-ethanolblending-in-petrol.html#19

http://www.thehindu.com/business/tax-sops-onmolasses/article7739457.ece

2. Question

Consider the following statements:


1. India, Bhutan, Nepal and Bangladesh
have recently signed a Motor Vehicles
Agreement
2. The Treaty of Peace and Friendship of
1950 between India and Nepal is still in
force
Which of the above statements are correct?
a) 1 only
b) 2 only
c) Both
d) None
Ans: c.
Aug 2014: The two Prime Ministers agreed to
review, adjust and update the Treaty of Peace and
Friendship of 1950 and other bilateral agreements.
Both sides agreed that the revised Treaty should
better reflect the current realities and aim to further
consolidate and expand the multifaceted and deep

rooted relationships in a forward looking manner.


http://pib.nic.in/newsite/PrintRelease.aspx?
relid=122516

http://www.thehindu.com/news/international/nepals
-crisis-is-an-outcome-of-its-internal-discordindia/article7739462.ece

3. Question

It is known that agroforestry can not only


alleviate poverty and reduce malnutrition, but
also help develop resilience to climate change.
With reference to agroforestry in India, consider
the following statements:
1. India is the first nation in the world to
have adopted an agroforestry policy
2. Various elements of agroforestry lie
scattered across various existing
missions, programme and schemes under
different ministries
3. All policies on agroforestry will be
subsumed under the National
Agroforestry Policy
Which of the above statements is/are incorrect?
a) 1 Only
b) 2 Only
c) 3 Only
d) None
Ans: c.
The National Agroforestry Policy is the first and
only policy on Agroforestry in India. However,
various elements of agroforestry lie scattered across
various existing missions, programme and schemes
under different ministries Agriculture, Rural
Development and Environment.
http://www.thehindu.com/news/national/involvefarmers-in-fight-against-climatechange/article7739732.ece

http://www.downtoearth.org.in/news/indiabecomes-first-country-to-adopt-an-agroforestrypolicy-43518

4. Question

It is said that India has the most progressive


agroforestry policy in the world. Which of the
following statements about Indias National
Agroforestry Policy is/are correct?
1. Its National Agroforestry Board will be
under the Ministry of Agriculture
2. Apart from promoting agroforestry
research, it also seeks to usher in land
reforms
Select the correct answer using the codes given
below:

Current Affairs 88 October-2015

a) 1 only
b) 2 Only
c) Both
d) None
Ans: c.
http://mrunal.org/2014/11/national-agroforestrypolicy-2014-salient-features-benefits.html#89

http://www.thehindu.com/news/national/involvefarmers-in-fight-against-climatechange/article7739732.ece

5. Question

The Supreme Court recently decided to set up


another Constitution Bench to have a relook at
the question whether right to privacy is a
fundamental right or not. Consider the following
statements about the Constitution Bench:
1. The minimum number of Judges who are

to sit in it shall be five


2. Article 32 of the Constitution contains a
provision dealing with setting up of the
Constitution Bench
Which of the above statements are incorrect?
a) 1 Only
b) 2 Only
c) Both 1 and 2
d) Neither 1 or 2
Ans: b.
Article 145 has a provision dealing with setting up
of the Constitution Bench. To be specific, it is
Article 145(3).
http://indiankanoon.org/doc/1537130/

http://www.thehindu.com/todays-paper/tpnational/after-60-years-sc-to-take-a-relook-at-rightto-privacy/article7740412.ece

Insights into Editorial: Measures for Judicial Reform


22 October 2015
Striking down the NJAC bill, the supreme court has clearly expressed its lack of confidence and faith in the
political class to preserve and protect the independence of the judiciary. However, the court has also
acknowledged the deficiencies of the collegium system and is looking forward for some reforms.
Quick look at collegium system and its history:
The Constitution does not envisage a collegium of judges to select judges. It was virtually proposed by the
lawyer community and the public who were distinctly uncomfortable with the intrusions into the
independence of the judiciary in the 1970s and 1980s. When the Supreme Court devised collegium system, it
was widely welcomed.
However, public and lawyer community were unhappy with this system too. It was proved, in some cases,
that even judges also can be men of straw. The lawyers and the public realised that like any normal human
being, several of the members of the collegium did not rise above their religion, caste, gender, language,
family, friends and other affinities.
Is difference of opinion between various organs unhealthy?
By rejecting NJAC, the SC has expressed its lack of faith in the political class. There could be different
perceptions on whether such lack of faith between the major organs of the state is warranted or is healthy in
a vibrant democracy. Honest differences do exist. But such differences should not lead to recrimination or
name-calling. The common man, the consumer of justice, is more interested in efficacious resolution of
disputes expeditiously. The three organs of the government judiciary, legislature and the executive should
take swift and decisive action in this regard in discharge of their constitutional obligations.
Which are the other areas that need immediate attention?
Vacancies in the Supreme Court and in the High Courts need to be filled up. Most High Courts are
functioning with half or one third the sanctioned strength.
Persons of doubtful integrity who might have been appointed by the mistake of the collegium have to
be weeded out. A method has to be found without the process of impeachment, and voluntary
retirement could be an option.
The infrastructure in the courts needs improvement there will not be enough court halls, chambers,
or staff, if all the vacancies are filled.
There needs to be appointment of ad hoc or additional judges to clear pending cases the reluctance
of the collegium to appoint retiring judges as ad hoc judges is baffling.
How the present collegium system can be improved?
Accepting applications for appointments as High Court judges should be followed. This is followed
Current Affairs 89 October-2015

in the U.K. and can be adopted in India too.


There must be full and complete disclosure of relationships and affiliations of applicants to sitting
and retired judges.
Minimum eligibility criteria for consideration need to be laid down, including appearances in
important cases.
Parliament should also enact changes to provide a uniform retirement age for judges of the Supreme
Court and the High Courts, so that the present practice of some of the judges seeking to be in the
good books of the existing or prospective members of collegiums in the Supreme Court is avoided.
This will also obviate the argument of expectation based on seniority for appointment as judges of the
Supreme Court.
The retirement age may be raised uniformly to 70 with a condition that no judge retiring at 70 shall
be appointed as a member of any Tribunal.
The continuation as a judge after the age of 65 should be subject to being found not unfit by the
Permanent Commissions.
A minimum tenure of two years should be provided to the Chief Justice of India and the Chief Justice
of High Courts.
No judge who is more than 68 years should be made a Chief Justice.
Court management should not be vested with Judicial Officers but assigned to trained managers.
All the three organs of the state should also introspect as to why there has been no or inadequate
representation in the higher judiciary from amongst women.
Proposal for a Permanent commission:
There is also a proposal to create a permanent commission to allay the fears of intrusion into the
independence of judiciary. It should be a three member Permanent Commission which will scrutinise
the credentials of candidates and recommend names.
The Commission may consist of three retired Chief Justices of India for appointment of judges to the
Supreme Court.
Four such similar Commissions may be constituted for the four regions of India with a retired judge
of the Supreme Court as a Chair Person and two retired Chief Justices of the High Courts as
members.
The tenure of the Chair person and members of the Commission should be three years.
The recommendations of the collegiums in the High Court may be forwarded to the Regional
Permanent Commission which shall then send its recommendations to the collegium in the Supreme
Court.
The selection of these permanent commissions should be made by a committee consisting of the
Chief Justice of India, two senior most judges of the Supreme Court, the Prime Minister and the
Leader of the Opposition in the Lok Sabha.
The collegium in the High Court may recommend a panel which is twice or thrice the number of
existing and expected vacancies and, on scrutiny, the Commissions can recommend a pruned list of
names to the Supreme Court Collegium.
These Permanent Commissions should also be vested with the power to scrutinise complaints of
dishonesty and lack of integrity of judges, to make recommendations to the collegiums to withdraw
work from those judges pending impeachment.
vehicles for a period of five years and some other
products imported from various other countries.
Paper 3 Topic: Effects of liberalization on the
Other products on which anti dumping duties are
economy.
imposed:
Chinese items imported into India at prices
FinMin imposes anti-dumping duty on
less than even cost of production, such as
Chinese auto parts
front-axle beam and steering knuckles meant
Following the recommendations of the directoratefor heavy and medium commercial vehicles.
general of anti-dumping and allied duties, the
Fully-drawn or fully-oriented yarn, spinfinance ministry has imposed anti-dumping duties
drawn yarn, and flat yarn of polyester (nonon imported Chinese parts used in commercial

22 October 2015

Current Affairs 90 October-2015

textured) imported from China and Thailand.


Plain medium density fibre board of
thickness 6 mm and above, imported from
China, Malaysia, Thailand and Sri Lanka.
Hexamine, which is used in making of
phenolic resins, if imported from China and
the United Arab Emirates.
What is anti dumping duty?
Anti dumping is a measure to rectify the situation
arising out of the dumping of goods and its trade
distortive effect. The purpose of anti dumping duty
is to rectify the trade distortive effect of dumping
and re-establish fair trade. The use of anti dumping
measure as an instrument of fair competition is
permitted by the WTO. It provides relief to the
domestic industry against the injury caused by
dumping. It is levied on distrustfully low-priced
imports, so as to protect the domestic
manufacturers.
sources: BS, finmin.
Paper 2 Topic: Government policies
and interventions for development in
various sectors and issues arising out of
their design and implementation.

Cabinet decisions
The Cabinet has cleared two ordinances for
expeditious settlement of commercial disputes that
would improve ease of doing business in the
country. It gave its nod to ordinances to amend the
Arbitration and Conciliation Act and bring into
force the Commercial Courts, Commercial Division
and Commercial Appellate Division of High Courts
Bill, 2015 pending before a Parliamentary standing
committee.
Nod to amend the Arbitration and Conciliation Act:
The ordinance to amend the Arbitration and
Conciliation Act is aimed at making India a
favourable place to settle disputes through
arbitration.
The Cabinet move has come amid
companies such as Vodafone choosing
overseas courts for this purpose.
The amendments include specifying and
restricting the term Public Policy of India
on whose ground an arbitral award could be
challenged.
According to the amendments, an award
would be construed as against the Public
Policy of India if it is induced or affected by
fraud or corruption, or is in contravention of
the fundamental policy of Indian law or is in
conflict with the most basic notions of

morality or justice.
The amendments, based on the
recommendations of the Law Commission,
would also put various timelines for order of
arbitration panel.
There would also be a change in the existing
law to the effect that mere filing of an
application for challenging the award would
not automatically stay execution of the
award. The award can only be stayed where
the court passed any specific order on an
application filed by the party.
The Commercial Courts, Commercial Division and
Commercial Appellate Division of High Courts Bill,
2015:
The Commercial Courts, Commercial
Division and Commercial Appellate
Division of High Courts Bill, 2015 would
enable setting up of commercial benches in
high courts for expeditious resolution of
disputes.
The Department Related Standing
Committee on Law and Personnel will table
its report on the Bill in Parliament by
November 30.
After being referred to a Rajya Sabha Select
Committee during the United Progressive
Alliances tenure, the Bill was sent to the
Law Commission.
Based on the law panels recommendations,
the National Democratic Alliance
government re-drafted the Bill as part of its
ease of doing business initiative.
sources: PIB.
Paper 2 Topic: Statutory, regulatory
and various quasi-judicial bodies.

Sebi notifies 12 entities as stock


exchanges, creates 7 departments
With commodities derivatives regulation under its
fold, the Securities and Exchange Board of India
(Sebi) has notified 12 commodity bourses and
associations as stock exchanges and has also created
seven departments for effectively regulating this
market.
To fulfil its additional responsibility of
regulating the commodity derivatives
market, Sebi has created additional seven
departments like legal affairs, surveillance
investigations and enforcement divisions.
It has also created departments for
commodity derivatives market regulation,

Current Affairs 91 October-2015

market intermediaries regulation and


supervision and economic policy and
analysis.
Sebi was recently merged with the Forward Market
Commission (FMC). Sebi was set up in 1988 as a
non-statutory body for regulating the securities
markets, while it became an autonomous body in
1992 with fully-independent powers.
sources: bs.
Paper 2 Topic: India and its
neighborhood- relations.

In the context of transnational crime and its


linkages to terrorism, the proposed Treaty will
provide a broad legal framework for bilateral
cooperation with Maldives in investigation
and prosecution of crime as well as in tracing,
restraint and confiscation of proceeds, and
instruments of crime as well as funds meant to
finance terrorist acts.
About Mutual Legal Assistance Treaty (MLAT):
MLAT is an agreement between two or more
countries for gathering and exchanging
information to enforce public or criminal
laws.

Signing of a treaty between India and


Maldives on Mutual Legal Assistance in
criminal matters
The Union Cabinet has given its approval for
signing of a Mutual Legal Assistance Treaty in
Criminal Matters between India and the Maldives.
The Treaty aims to enhance effectiveness of
both countries in investigation and
prosecution of crime, through cooperation
and mutual legal assistance in criminal
matters.

Under the agreement, mechanisms have


been developed among nations for
requesting and obtaining evidence for
criminal investigations and prosecutions.
sources: pib.
1. Question

Consider the following statements associated


with the plant Opium Poppy:
1. India is sandwiched between two
producing regions of it the Golden
Triangle and the Golden Crescent
2. Its seeds are used in Indian cuisine
Which of the above statements are incorrect?
a) 1 Only

Current Affairs 92 October-2015

b) 2 Only
c) Both 1 and 2
d) Neither 1 or 2
Ans: d.
Opium is the source of many drugs,
including morphine. For curiositys sake: In Indian
cuisine white poppy seeds are added for thickness,
texture and also give added flavor to the recipe.
Commonly used in the preparation of korma,
ground poppy seed, along with coconut and other
spices, are combined as the masala to be added at
the end of the cooking step. It is quite hard to grind
them when raw, so they are normally dry fried, and
then mixed with a little water to get the right paste
consistency.

http://pib.nic.in/newsite/PrintRelease.aspx?
relid=128547

https://en.wikipedia.org/wiki/Golden_Triangle_(Sou
theast_Asia)

2. Question

Which of the following statements about the


Asia Europe Meeting (ASEM) are correct?
1. India is a not a founder member
2. The ASEM dialogue addresses not only
economic, but also political and cultural
issues
3. The ASEM summit is a biennial meeting
Select the correct answer using the codes given
below:
a) 1 and 2 Only
b) 2 and 3 Only
c) 1 and 3 Only
d) All are correct
Ans: d.
http://www.aseminfoboard.org/about

http://pib.nic.in/newsite/PrintRelease.aspx?
relid=128593

3. Question

Gazette of India notifications are published by


a) Press Information Bureau, Ministry of
Information and Broadcasting
b) Publications Division, Ministry of
Information and Broadcasting
c) The Department of Publication,
Ministry of Urban Development
d) Publications Division, Ministry of
Statistics and Programme Implementation
Ans: c.
http://egazette.nic.in/Aboutus.aspx

http://pib.nic.in/newsite/PrintRelease.aspx?
relid=128570

4. Question

With reference to the High Risk Area (HRA) in


the Indian Ocean, consider the following
statements:
Assertion (A): Additional War Risk Premiums
imposed by Insurance providers on vessels
carrying Indian EXIM (export-import) along the
west coast are set to reduce, resulting in lower
costs to consumers
Reason (R): International Chamber of Shipping
and other international bodies have agreed to
push back the HRA from 78 W longitude to the
to the 65 W longitude
Select the correct answer using codes given
below:
a) A and R both are true, and R is the
correct explanation for A
b) A and R both are true, and R is the
NOT the correct explanation for A
c) A is correct, R is incorrect
d) A is incorrect, R is correct
Ans: c.
Beware of such traps. India lies in the Eastern
Hemisphere. International Chamber of Shipping and
other international bodies have agreed to shift the
HRA from 78 E longitude to the to the 65 E
longitude.
Note: Both statements mentioned above will be in
past tense by the August 2016.
http://pib.nic.in/newsite/PrintRelease.aspx?
relid=128577

5. Question

Why are there fewer cyclones over the Arabian


Sea (AS) as compared to the Bay of Bengal
(BB)?
1. AS is relatively colder than BB
2. Majority of the cyclones over BB weaken
over land after landfall
3. In addition to cyclones which develop insitu, BB gets an increased quota of them
due to high frequency of cyclones over
the Northwest Pacific
Select the correct answer using the codes given
below:
a) 1 only
b) 2 Only
c) 1, 2 and 3
d) The frequency of cyclones remains the
same (AS cyclones make landfall in West
Asia while BB cyclones, along the East
Coast of India)

Current Affairs 93 October-2015

Ans: c.
http://www.imd.gov.in/section/nhac/dynamic/faq/F
AQP.htm#q25

http://pib.nic.in/newsite/PrintRelease.aspx?
relid=128566

6. Question

Consider the following statements about the


Territorial Army of India:
1. It is a part of the regular army
2. They also known as the Green Army
due to their pioneering role in
preservation of the nations biodiversity
Which of the above statements is/are correct?
a) 1 Only
b) 2 Only

c) Both 1 and 2
d) Neither 1 or 2
Ans: a.
They are not known as the Green Army, but they
have played a pioneering role in preservation of
ecosystem and the nations biodiversity.
http://pib.nic.in/newsite/PrintRelease.aspx?
relid=128567

http://indianarmy.nic.in/Site/FormTemplete/frmTem
p1PLT3C.aspx?
MnId=7Zlke2QlpgL2YScSgUtUsg==&ParentID=g
mqe2Bk/dm5xsuTmf4xsyg==&flag=Rmpgcjbyjq3
NGC0+NMqNMA==

Insights into Editorial: Replacing Food with


Cash
23 October 2015
The central government gazetted the Cash Transfer of Food Subsidy Rules on 21 August 2015. With this, it
appears that the current government is deeply committed to substitute subsidised wheat and rice supplied
through the public distribution system (PDS) with direct cash transfers into bank accounts of targeted
households.
Cash transfers in lieu of PDS would involve the transfer of money directly into bank accounts of identified
card holders. The amount transferred would be the difference between the market and subsidized price of the
grain. Instead of going to their local ration shop to purchase subsidized grains, recipients would withdraw
this money to buy the food of their choice from the market.
However, the substitution of food transfers with cash is not being appreciated by food rights campaigners.
What PDS was opposed?
Some reports have shown that PDS is an inefficient mode of transfer of subsidies, prone to enormous
leakages into the black market, and high waste in costs of transferring subsidies in the form of food transfers.
Why cash transfer would be better?
Replacing food with direct cash transfers would greatly reduce corruption and leakages.
It would enable the poor to access goods currently denied them by a PDS beset by corruption.
It would enable people to buy better quality food of their choice from the open market and not be
restricted to items sold in the PDS, which are often inferior in quality and limited in range.
It would both bypass brokers as well as reduce the waste and holding costs of storing grains in
government silos.
The amount of grain actually required for Indias buffer stock needs could be held in better-quality
warehouses, eliminating waste and rotting.
It would also help reduce fiscal deficit by curbing expenditures earmarked for the PDS that are
siphoned off through corruption, as well as avoiding substantially higher costs of transferring food
rather than cash.
Why cash transfer may not be a great idea?
It is possible for people to spend cash transfers not on more nutritious food but instead on non-food
items, which would decrease the amount of household money left for buying food.
Research confirms that culturally decisions relating to cash in households tend to be made by men,
who may or may not spend the money on food.
Decisions relating to food are made by women in almost all cultures, and therefore food rather than
cash in a household is more likely to end up as food in a childs stomach.
Current Affairs 94 October-2015

Why PDS would be better than cash transfers?


There are worries about how genuinely inclusive of people in remote rural regions is Indias banking
system. Fair price shops exist in three of every four villages, and are therefore generally accessible.
PDS supplies rations at a constant price, irrespective of the fluctuations in market prices. This
therefore provides a shield against inflation, a benefit that cash transfers cannot match.
Conclusion:
It is also problematic to assume that cash transfers would in themselves bring about drastic reductions in
corruption and leakages in welfare programmes, as there is nothing intrinsic to cash transfers which renders
them less vulnerable to leakages. Irregularities are empirically found to be high in existing cash transfer
programmes like pension schemes. The difference between the corruption or probity of delivery of welfare
programmes is not dependent on whether cash or food is delivered, but on political and administrative will
and capacities, and public vigilance and organization. Studies have confirmed that many states have been
able to reform PDS and significantly reduce leakages.
These findings indicate that RSBY and other state
run insurance programmes have failed to provide
aper 2 Topic: Welfare schemes for vulnerable
financial risk protection.
sections of the population by the Centre and
Why they have failed?
States and the performance of these schemes.
According to the study, a major design flaw
in RSBY and other such state health
RSBY failing to provide risk cover
insurance programmes is their narrow focus
An evaluation of the Rashtriya Swasthya Bima
on secondary and tertiary care
Yojana (RSBY) has concluded that the RSBY
hospitalization.
scheme has had little or no impact on medical
The study also says that RSBY was being
impoverishment in India.
The assessment was conducted by the
used mostly by those who already had better
access (to healthcare services) and the most
Council for Social Development and the
marginalized sections were being excluded
report is titled, India: Social Development
further.
Report 2014.
RSBY
and
similar state run health schemes are
The authors of the study compared data
designed with the intention to address low-volume,
before and after the launch of RSBY to
high-value financial transactions that could result in
understand the emerging trends in out-ofcatastrophic expenditure and impoverishment of
pocket expenditure for medical care in India.
households. However, these evidences suggest the
The aim of the study was to study the impact
opposite trend.
of RSBY and evaluate how much financial
Rashtriya Swasthya Bima Yojana (RSBY):
risk protection it granted to beneficiaries.
RSBY was launched by the Ministry of Labour and
Important findings:
Employment, Government of India to provide
Despite high enrolment in RSBY,
health insurance coverage for Below Poverty Line
catastrophic health expenditures (when
(BPL) families.
medical expenses push a family into
Details:
poverty), hospitalization expenditure and the
Beneficiaries under RSBY are entitled to
percentage of total household outgo on outhospitalization coverage up to Rs. 30,000/of-pocket (OOP) expenses medicines and
for most of the diseases that require
other consumables that are not reimbursed
hospitalization.
by insurance have steadily increased, for
Government has even fixed the package
both in-patients and outpatients, over the last
rates for the hospitals for a large number of
two decades.
interventions.
Between 2004-05 and 2011-12,
Pre-existing conditions are covered from day
hospitalization expenses have increased at a
one and there is no age limit.
much higher rate (9.2%) compared to
Coverage extends to five members of the
outpatient expenses (4.5%) or medicines
family, which includes the head of
(4.85%).
household, spouse and up to three
The poorer income sections in RSBY have
dependents.
experienced a rise in catastrophic headcount.
Beneficiaries need to pay only Rs. 30/- as

23 October 2015

Current Affairs 95 October-2015

registration fee while Central and State


Government pays the premium to the insurer
selected by the State Government on the
basis of a competitive bidding.
How RSBY is different from other schemes?
Empowering the beneficiary RSBY
provides the participating BPL household
with freedom of choice between public and
private hospitals and makes him a potential
client worth attracting on account of the
significant revenues that hospitals stand to
earn through the scheme.
Business Model for all Stakeholders The
scheme has been designed as a business
model for a social sector scheme with
incentives built for each stakeholder.
Hospitals A hospital has the incentive to
provide treatment to large number of
beneficiaries as it is paid per beneficiary
treated.
Intermediaries The inclusion of
intermediaries such as NGOs and MFIs
which have a greater stake in assisting BPL
households.
Information Technology (IT) Intensive For
the first time IT applications are being used
for social sector scheme on such a large
scale. Every beneficiary family is issued a
biometric enabled smart card containing
their fingerprints and photographs. All the
hospitals empanelled under RSBY are IT
enabled and connected to the server at the
district level. This will ensure a smooth data
flow regarding service utilization
periodically.
Safe and foolproof The use of biometric
enabled smart card and a key management
system makes this scheme safe and
foolproof. The key management system of
RSBY ensures that the card reaches the
correct beneficiary and there remains
accountability in terms of issuance of the
smart card and its usage. The biometric
enabled smart card ensures that only the real
beneficiary can use the smart card.
Portability The key feature of RSBY is that
a beneficiary who has been enrolled in a
particular district will be able to use his/ her
smart card in any RSBY empanelled hospital
across India. This makes the scheme truly
unique and beneficial to the poor families
that migrate from one place to the other.
Cards can also be split for migrant workers

to carry a share of the coverage with them


separately.
Cash less and Paperless transactions A
beneficiary of RSBY gets cashless benefit in
any of the empanelled hospitals. He/ she
only needs to carry his/ her smart card and
provide.
Sources: the Hindu, pib.
Paper 1 Topic: urbanization.

Centre clears first batch of flagship urban


projects
The Urban Development Ministry has cleared the
first batch of projects under the flagship Atal
Mission for Rejuvenation and Urban
Transformation (AMRUT) for 89 cities worth
Rs.2,786 crore.
The States that are getting benefited from
this push are Andhra Pradesh, Gujarat and
Rajasthan.
This is for the first time in Indias urban
governance that the Urban Development
Ministry is sending money to States in
massive chunks so that the development
projects are run without financial delays.
This is also the first time that the Urban
Development Ministry approved State level
plans unlike the past practice of appraising
and approving individual projects.
The focus of these approved urban renewal
projects would be on establishing
infrastructure that could ensure adequate
water supply and robust sewerage networks.
Atal Mission for Rejuvenation and Urban
Transformation (AMRUT):
AMRUT is the new avatar of the Jawaharlal Nehru
National Urban Renewal Mission (JNNURM).
It adopts a project approach to ensure basic
infrastructure services relating to water
supply, sewerage, storm-water drains,
transportation and development of green
spaces and parks with special provision for
meeting the needs of children.
Under this mission, 10% of the budget
allocation will be given to states and union
territories as incentive based on the
achievement of reforms during the previous
year.
It is being implemented in 500 locations
with a population of one lakh and above.
It would cover some cities situated on stems
of main rivers; a few state capitals and

Current Affairs 96 October-2015

important cities located in hilly areas, islands


and tourist areas.
Under this mission, states get the flexibility
of designing schemes based on the needs of
identified cities and in their execution and
monitoring.
States will only submit state annual action
Plans to the centre for broad concurrence
based on which funds will be released.
Central assistance will be to the extent of
50% of project cost for cities and towns with
a population of up to 10 lakhs and one-third
of the project cost for those with a
population of above 10 lakhs.
Under the mission, states will transfer funds
to urban local bodies within 7 days of
transfer by central government and no
diversion of funds to be made failing which
penal interest would be charged besides
taking other adverse action by the centre.
Sources: the Hindu, pib.
Paper 3 Topic: Security challenges and
their management in border areas;
linkages of organized crime with
terrorism.

Spy cam project fails to click


A pilot project to install high-resolution surveillance
cameras along the China border, undertaken in 2013
to monitor the movement along the Line of Actual
Control (LAC) has failed to give the desired results
and the government is now rethinking its strategy.
Background:
The project was initiated in 2014 in the wake
of the 21-day face-off with the Peoples
Liberation Army (PLA) of China at Depsang
Valley in the Ladakh region.
Initially, it was planned to install
surveillance cameras along the unmanned
pockets on the China border. It was decided
that the cameras would be put up at 50
locations in Himachal Pradesh, Jammu and
Kashmir, Sikkim and Tawang in Arunachal
Pradesh.
The cameras were to relay live-feed in a 2025 km range to help the security personnel
deployed there to plan patrolling in
vulnerable areas more effectively.
The pilot project was taken up at Thakung
post, which is a high-altitude terrain.
However, the weather is not favourable there
as high-velocity winds and frost tend to blur

the images. It has not been possible to


establish proper links.
The official are looking for better technology
now.
The Indian Army, which is the second line of
defence along the China border, does its own
surveillance with the help of unmanned
aerial vehicles but this also has its
limitations.
China has a robust surveillance system on its side.
They have put up a well-knit surveillance network
on their side. India has always maintained that
incidents of transgressions occur due to difference
in perception regarding the border. After the NDA
government came to power, the frequency of
patrolling along the China border has increased.
Sources: the Hindu.
Paper 2 Topic: disaster management.

India China Joint Exercise Hand-in-Hand


: 2015 Concludes
The fifth round of India-China joint military
exercises was recently concluded in the Chinese city
of Kunming with the two countries holding a
unique mock exercise depicting counter-terror
operations at the Sino-Indian border.
The 10-day exercises codenamed Hand-inHand was supervised by a joint delegation
of observers, who expressed satisfaction at
the level of assimilation achieved, said the
statement.

The exercise featuring troops from both sides


at the border was regarded significant
considering recent tensions between the two
militaries in the Ladakh region, where Indian
troops objected to attempts by their Chinese
counterparts to claim areas on the Indian side
of the Line of Actual Control (LAC).

Current Affairs 97 October-2015

Earlier editions of the Sino-Indian military drills


mainly focused on countering terrorism, but
humanitarian assistance and disaster relief emerged
as a top priority during the latest round of drills. The
devastating earthquake in Nepal, to which troops
from India and China were among the first
responders, appears to have imparted fresh urgency
among military planners of the two countries to
include disaster relief on the agenda of joint military
exercises.
Sources: the Hindu, pib.
Paper 1 Topic: Art and culture.

Sangeet Natak Akademi Fellowships and


Awards for 2014
The President of India, Shri Pranab Mukherjee
recently conferred the Sangeet Natak Akademi
Fellowships and Sangeet Natak Akademi Awards
for the year 2014 at a function held in Rashtrapati
Bhavan.
About the akademi:
The Sangeet Natak Akademi Indias national
academy for music, dance and drama is the first
National Academy of the arts set-up by the Republic
of India. It was created by a resolution of
Government of India. It was set up in 1952.
The academy functions as the apex body of
the performing arts in the country to
preserve and promote the vast cultural
heritage of India expressed in music, dance
and drama.
It also works with governments and art
academies in states and territories of the
country.
The academy Renders advice and assistance
to the government of India in the task of
formulating and implementing policies and
programmes in the field. It carries a part of
the responsibilities of the state for fostering
cultural contacts between regions in the
country, as well as between India and the
world.
Awards:
The Akademi Awards are the highest
national recognition conferred on eminent
artistes.
Each year the Academy awards Sangeet
Natak Akademi Fellowships, Ratna Sadsya,
to distinguished individuals for their
contribution to the field of arts, music, dance
and theatre.
Ustad Bismillah Khan award is given to

young artists for their talent in the fields of


music, dance and drama.
Sources: the Hindu, wiki.
1. Question

Consider the following pairs:


Directions
Neighbouring
State/Country
1. East
Bangladesh
2. West
Bangladesh
3. South
Myanmar
4. North
Meghalaya
With reference to borders shared by the Indian
state of Tripura, which of the above pairs are
correctly matched?
a) 1 and 2 Only
b) 2 and 3 Only
c) 2,3 and 4 Only
d) 1,3 and 4 Only
Ans: a.
http://pib.nic.in/newsite/PrintRelease.aspx?
relid=128579
Even though being the third smallest State in the
country, Dr. Jitendra Singh said, Tripura has its own
strategic importance and shares its borders with
Bangladesh in North, South and West, and can thus
become an important exit point of export and trade
with the countries on the eastern border
Take a look at the political map of India. Tripura is
not square-shaped. A portion of its eastern borders
too are shared by Bangladesh, rest being shared by
Mizoram and a very small portion by Assam.

2. Question

Consider the following statements:


1. The Agricultural Produce Market
Committee (APMC) Act controls only the
first sale of the agricultural produce
2. The Essential Commodities (EC) Act
empowers the central and state
governments concurrently to control the
production of certain commodities
3. EC Act also covers commodities such as
coal and textiles
Which of the above statements are incorrect?
a) 1 Only
b) 1 and 3 Only
c) 3 Only
d) All statements are correct
Ans: d.
Page 118-119: Economic Survey 2014-15, Volume 1
http://pib.nic.in/newsite/PrintRelease.aspx?
relid=128585

3. Question

The United Nations Security Council Resolution

Current Affairs 98 October-2015

1953
a) Urges all States, including non-State
Parties to the International Criminal
Court (ICC) to cooperate fully with the
ICC
b) Affirms the urgent need to reach
within 12 months a peaceful solution to
the situation in the Middle East
c) Paved the way for the Palestinian flag
to be raised at the United Nations
d) None of the above
Ans: d.
Trick question! Those who had a vague idea of this
articles contents, may have selected option a.
It is important to note that United Nations Security
Council resolution 1593 urged all States, including
non-States Parties to the ICC as well regional and
other international organisations, to cooperate fully
with the ICC. This includes arresting Mr. Al-Bashir
UNSC resolution 1953 deals with Cyprus,
reaffirming all resolutions on the situation there as
of now, there is no reason for you to know this.

4. Question

India was recently asked by the International


Criminal Court (ICC) to arrest one of the guests
expected to attend the India-Africa Summit.
With reference to the ICC, which of the
following statements is/are incorrect?
1. It is independent of the United Nations
2. India is not a signatory to it
3. It can request non-signatories (to the

ICC) to cooperate with it, which among


others, includes acts of arrest
Select the correct answer using codes given
below:
a) 1 only
b) 2 Only
c) 3 Only
d) All statements are correct
Ans: d.
This question has been framed from this article.

5. Question

The transfer of which of the following nontangible goods/services can be classified as


invisible trade?
1. Customer services
2. Intellectual property
3. Consulting
4. Tourism
5. Shipping services
Select the correct answer using codes given
below:
a) 1, 2 and 5 only
b) 1,3 and 4 Only
c) 2,4 and 5 Only
d) All of the above
Ans: d.
Invisible trade has been indirectly defined in the
question itself! This article also appeared in the
Hindu.
http://www.investopedia.com/terms/i/invisibletrade.asp

Insights into Editorial: India and Pakistan can


Start Opening up to Trade
24 October 2015
The trade between India and Pakistan has been minuscule for years now. The official bilateral trade between
the two countries in 2014 is barely worth $3 billion. Despite their nearly seven decades of rivalry, the two
countries have a bottom-line interest in increasing commerce between them. If existing trade barriers are
removed, the trade could increase by almost 10 times. Presently, both countries lose millions in potential
customs revenue to smuggling. Eliminating trade barriers could also make goods cheaper for consumers and
factories more competitive.
Other implications:
Deeper commercial ties could also help help ease geopolitical tensions between the two countries.
Trade volumes in the range of $10 billion to $15 billion annually could create a lobby in Indian and
Pakistani business circles influential enough to steady relations.
Developments so far:
India has granted Pakistan most favored nation trading status since 1996, when both countries
joined the World Trade Organization.
Pakistan has also drastically reduced the number of Indian imports banned or subject to high duties
since 1996.
Current Affairs 99 October-2015

Pakistan also wants India to reciprocate by lowering so-called non-tariff barriers that allegedly disadvantage
Pakistani producerseverything from strict licensing and inspection rules to subsidies for Indian farmers.
Recent developments:
India has planned to bypass Pakistan entirely and reach Central Asia via the Iranian port of Chabahar.
Pakistan has gone ahead with China withs a $46 billion economic corridor project linking its own port at
Gwadar to Chinas massive market. These moves have increaseed the tension between the two countries and
have widened the existing gap. Political talks have been stalled for months, with India insisting they focus
only on eliminating terrorism and Pakistan demanding that the status of Kashmir be on the table as well.
What should both the nations do to enjoy the full benefits of open trade and improve the existing situation?
Work jointly at improving logistics, untangle red tape, build new roads, ports and rail connections.
Allow businesspeople to travel and invest more freely, and to remit their profits.
A jointly run special economic zone along the border that divides the Indian and Pakistani Punjabs
can be created.
To ease security concerns, goods, workers and executives going into and out of the zone could be
monitored.
For companies that are setting up shops, rules governing financing and remittances could be eased,
visa restrictions and other bureaucratic barriers can be lifted.
Factories should be given a steady supply of power.
Other countries can also help in easing the situation. The US and the European Union, for instance,
might offer tariff-free access to any goods exported from the zone.
Foreign companies that currently manufacture in both countries could be encouraged to consolidate
their operations.
Hence, in the short run, both countries should look for a more limited opening rather than strive for a major
breakthrough.
instrumenting and monitoring sub-seafloor
environments.
Paper 3 Topic: Science and Technology.
Why subseafloor?
The Earth deep below the seafloor contains a unique
Manipal varsity scholar in deep sea
record of our planets history and structure.
research team
Scientists study the subseafloor to better understand
Gurumurthy G.P., a Post-doctoral Research Fellow
Earths components, processes, and phenomena.
at the Manipal Centre for Natural Sciences
This research helps answer questions about
(MCNS), Manipal University, has become the first
fundamental aspects of our planet such as the past
participant from the university and the first from
global environment, the deep biosphere, plate
any private university in the country to participate
tectonics, and deep fluid flow.
in the prestigious deep sea International Ocean
sources: the hindu, iodp.
Discovery Programme (IODP).

24 October 2015

Gurumurthy was among the 30 scientists


from 26 countries to sail in Joides
Resolution, a deep sea drilling research
vessel.
About the deep sea International Ocean Discovery
Programme (IODP):
The International Ocean Discovery Program
(IODP) is an international research
collaboration that coordinates seagoing
expeditions to study the history of the Earth
recorded in sediments and rocks beneath the
ocean floor.
IODP is dedicated to advance scientific
understanding of Earth by sampling,

Paper 2 Topic: RPA; Features of


Constitution

Adnan Sami will be an Indian citizen soon


The union government has decided to give Indian
citizenship to Adnan Sami, a Pakistani singer.
The decision was taken after AttorneyGeneral Mukul Rohatgi gave a favourable
opinion to the Ministry of Home Affairs
(MHA).
Adnan Sami has been staying in India since
2001. In May this year, the MHA granted
him the right to stay in India indefinitely on
humanitarian grounds after his Pakistani

Current Affairs 100 October-2015

passport expired the same month.


He was also exempted from deportation
proceedings under Section 3 of the
Foreigners Act.
He will be given Indian citizenship under Section 6
of the Indian Citizenship Act, 1955.
What does Section 6 say?
Section 6 of the Indian Citizenship Act, 1955 allows
citizenship, through a certificate of naturalisation,
to applicants with distinguished contribution in the
field of science, philosophy, art, literature, world
peace or human progress.
sources: the hindu.
Paper 2 Topic: Statutory, regulatory and
various quasi-judicial bodies.

Basu is Atomic Energy Commissions new


chief
Sekhar Basu was recently appointed as the new
Chairman of Atomic Energy Commission (AEC).
Mr. Basu will also be Secretary, Department of
Atomic Energy (DAE).
He was till now Director, Bhabha Atomic
Research Centre (BARC), Trombay,
Mumbai. He was earlier Chief Executive of
the Nuclear Recycle Board.

MWt Pressurised Water Reactor (PWR) which


propels Arihant.
He was also responsible for the design,
development, construction and operation of
plutonium reprocessing plants at Tarapur in
Maharashtra and Kalpakkam in Tamil Nadu.
Basu played an important role in the
development of the India-based Neutrino
Observatory (INO) slated to come up in
Tamil Nadu.
He has also been guiding the setting up of a
nuclear fuel cycle park which includes
research reactors, fuel fabrication plants and
reprocessing facilities on the Visakhapatnam
campus of BARC.
About India Atomic Energy Commission:
The Indian Atomic Energy Commission was
first setup in August 1948 in the Department
of Scientific Research.
Later on, in accordance with a Government
Resolution, the Atomic Energy Commission
(AEC) was established in the Department of
Atomic Energy. The Department of Atomic
Energy (DAE) was setup on August 3, 1954
under the direct charge of the Prime Minister
through a Presidential Order.
According to the Resolution constituting the
AEC, the Secretary to the Government of
India in the Department of Atomic Energy is
ex-officio Chairman of the Commission.
The other Members of the AEC are
appointed on the recommendation of the
Chairman, AEC and after approval by the
Prime Minister.
Important functions of the Atomic Energy
Commission are:
1. to organise research in atomic scientists in
the country.
2. to train, atomic scientists in the country.
3. to promote nuclear research in commissions
own laboratories as well as in India.
4. to undertake prospecting of atomic minerals
in India and to extract such minerals for use
on industrial scale.
sources: the hindu, wiki.
Paper 3 Topic: Public Distribution
System objectives, functioning,
limitations, revamping.

Basu made a name for himself as the Project


Director of the Nuclear Submarine
Programme under which Arihant was built.
He was responsible for developing the 80

POS machines to plug pilferage at ration


shops
To cut pilferage in the public distribution system

Current Affairs 101 October-2015

(PDS), the Karnataka state government has decided


to set up biometric system-based point of sales
(POS) machines in all fair price ration shops across
the State.
The POS machines provide real time details
about ration. At present, there is no system
to know real time details of ration in PDS
shops during the month end.
Based on the successful execution of a pilot
project in Udupi district, the POS scheme
has been extended to all PDS shops across
the State.
How POS machines operate?

This machine is sim card based. Those who


purchase ration will have to give a thumb
impression on this machine. It then
authenticates the biometric details of the
ration card holder before generating the bill. It
also sends information to the central server so
that there is no scope for pilferage.
Why POS scheme was necessary?
There are possibilities of shop owners selling the
remaining food items to others at a higher price,
instead of distributing it to the beneficiaries. With
the installation of POS machines, as soon as any
person gets the ration in the shop, the data is
immediately sent to the server and uploaded on the
website. Hence, there will be no scope for diversion
of food items by shop owners.
Other details:
The state government has instructed owners
of PDS shops to install the machines by this
year end. Owners themselves have to bear
the cost of the machine, which is in the

range of Rs. 15,000 to Rs. 20,000.


There are about 20,500 governmentcontrolled ration shops in the State, serving
over 1.03 crore BPL card holders and 11
lakh APL card holders.
sources: the hindu.
Paper 2 Topic: mobilization of
resources.

GOLD DEPOSIT SCHEME


The Reserve Bank of India has issued guidelines for
the gold monetisation scheme that allow banks to
fix their own interest rates on gold deposits.
The Prime Minister Narendra Modi will
formally launch the scheme on November 5.
Important guidelines:
Banks will be free to set interest rate on such
deposit, and principal and interest of the
deposit will be denominated in gold.
Redemption of principal and interest at
maturity will, at the option of the depositor
be either in Indian rupee equivalent of the
deposited gold and accrued interest based on
the price of gold prevailing at the time of
redemption, or in gold. The option in this
regard shall be made in writing by the
depositor at the time of making the deposit
and shall be irrevocable.
The interest will be credited in the deposit
accounts on the respective due dates and will
be withdrawable periodically or at maturity
as per the terms of the deposit.
Gold Monetisation scheme:
Through the Gold Monetisation Scheme,
gold in any form can be deposited with
banks for a period of one to 15 years. This
gold will earn interest and redemption will
be at the prevailing market value at the end
of the tenure of deposit.
The scheme also provides for incentives to
the banks, while individuals and institutions
can deposit as low as 30 gm of gold, while
the interest earned on it would be exempt
from income tax as well as capital gains tax.
The scheme is aimed at mobilising a part of
an estimated 20,000 tonnes of idle precious
metal with households and institutions.
This scheme was actually announced in the
Budget for 2015-16.
sources: the hindu, pib.
1. Question

Consider the following statements about female


labour force participation in India

Current Affairs 102 October-2015

1. It has been declining sharply in India


2. Changes in measurement methodology
across survey rounds may have
contributed to its decline
3. Indias inability to compete with cheaper
labour provided by countries like
Bangladesh too has contributed to the
decline
Which of the above statements are correct?
a) 1and 2 Only
b) 1 and 3 Only
c) 1,2 and 3
d) All statements are incorrect
Ans: c.
Reading just the summary on this page will suffice.
http://www.thehindu.com/news/national/raisingwages-can-boost-growth-in-indiailo/article7747672.ece

2. Question

With reference to the Qutb Minar and its


monuments, consider the following statements:
1. Although Qutbud-Din Aibak laid the
Qutb Minars foundation, he did not
construct all 5 storeys
2. Quwwat-ul-Islam Mosque is the earliest
extant mosque built by the Delhi Sultans
3. Qutbud-Din Aibaks tomb is not present
in the complex
Which of the above statements are incorrect?
a) 2 and 3 Only
b) 1 and 3 Only
c) 2 Only
d) All statements are correct
Ans: d.
http://asi.nic.in/asi_monu_whs_qutbminar.asp

http://www.thehindu.com/todays-paper/tpnational/asi-to-seek-nofly-zone-over-qutubminar/article7748261.ece

3. Question

The Quartet Roadmap is


a) A peace plan Proposed by USA, the
EU, Russia and the UN to resolve the
Israeli-Palestinian conflict
b) A plan formulated by USA, Russia,
Israel and Palestine to resolve the IsraeliPalestinian conflict
c) A plan formulated by the P5 grouping,
India, Germany and Brazil to tackle
terrorism
d) A peace plan formulated by USA, Iran,
Saudi Arabia and UAE to end the Yemeni
conflict

Ans: a.
http://pib.nic.in/newsite/PrintRelease.aspx?
relid=128630

https://www.globalpolicy.org/component/content/art
icle/189-israel-palestine/38357-the-road-map.html

4. Question

Consider the following statements:


Assertion (A): The farmers of the Bundelkhand
region have largely depended on just the rabi
season; the area of crops sown during kharif is
much smaller
Reason (R): The region suffers due to Anna
Pratha
Select the correct answer using the codes given
below:
a) A and R both are true, and R is the
correct explanation for A
b) A and R both are true, and R is the
NOT the correct explanation for A
c) A is correct, R is incorrect
d) A and R both are incorrect
Ans: a.
The rabi crop is good here and high value crops
like gram, lentil, pea etc are grown in the area in
large quantities. However, the farmers of the region
have largely depended on just one crop; the area
sown during kharif is much smaller. The primary
reason given is that the region suffers due to Anna
Pratha. This is a traditional system under which
people leave their milch cattle unfettered to graze in
the fields. These cattle eat up the crops and
therefore the farmers are not inclined to sow their
lands during the season of Kharif. This does not
happen during the rabi season as he farmers keep
their cattle at home.
PIB Features 8th October, Open Sesame! by
Amit Mohan Prasad
http://www.pib.nic.in/newsite/efeatures.aspx

5. Question

Consider the following statements:


1. Cattle do not eat this crop
2. It is a crop meant for the rain fed areas
3. Very few countries cultivate it
Which of the above statements about the
Sesame or Til crop is/are true?
a) 1 only
b) 2 and 3 Only
c) 1,2 and 3 Only
d) None of the above
Ans: c.
The economics of sesame cultivation is revealing.

Current Affairs 103 October-2015

It is a crop meant for the rain fed areas. There is no


dearth of market for the produce as there are very
few countries cultivating sesame whereas almost
every country consumes it in some form or the
other. Sesame oil is used for cooking, making
sweets, massage, medicinal and cosmetic purposes
and many other uses. Sesame seeds are used for
preparing sweets like gajak, rewari, laddoos and
tilkut in India, tahini sauce in Middle East and on
top of buns and burgers in the west. It is also used
for different rituals on religious occasions. Sesame
gets a better price for the farmers than the other
oilseeds in the market.
PIB Features 8th October, Open Sesame! by
Amit Mohan Prasad
http://www.pib.nic.in/newsite/efeatures.aspx

6. Question

. Consider the following list of cow breeds:


1. Tharparkar
2. Rahiwal
3. Jersey
4. Rathi
Which of the above are native to India?
a) 1, 2 and 3 only
b) 2,3 and 4 Only
c) 1,2 and 4 Only
d) These are not cow breeds
Ans: c.
http://www.thehindu.com/business/towards-robustnative-breeds/article7750143.ece

Insights into Editorial: Growing Forests in the


Air
26 October 2015
The Union Environment Ministry, on 2nd October, submitted its Intended Nationally Determined
Contributions (INDCs) to the UN Framework Convention on Climate Change (UNFCCC).
The Ministry has proposed to:
1. Reduce emissions intensity of its GDP by 33 to 35% by 2030 from 2005 level.
2. Achieve about 40% electric power installed capacity from non-fossil fuel based energy resources by
2030 with help of transfer of technology and low cost international finance.
3. Create an additional carbon sink of 2.5 to 3 billion tonnes of CO2 equivalent through additional
forest and tree cover by 2030.
The ministry has proposed to achieve the 3rd target through existing programmes and schemes such as the
National Afforestation Programme (NAP), Joint Forest Management (JFM), the Green India Mission (GIM)
and compensatory afforestation (CA).
Performance of various afforestation programmes in India:
The history of afforestation programmes and their indifferent outcomes in India is not promising. India has
had numerous centrally sponsored plantation programmes such as the National Afforestation Pragramme
(NAP).
According to various reports the survival rate of trees planted under various afforestation
programmes in India is only 10-20%. This is worrisome and calls into question the very schemes
expected to deliver these contributions.
Various surveys also show that growing and maintaining forests has proven difficult for Indias Forest
bureaucracy.
The Thirty-Sixth Report of the Lok Sabha Secretariat Committee on Estimates (Fifteenth Lok Sabha)
on National Afforestation Programme in February 2014 observed that the outcome of the NAP,
launched in 2002, had been negative. The committee observed that even after spending thousands of
crores of rupees, the total area under forest cover had declined by 367 sq km.
Why the 3rd proposed INDC target appears unrealistic?
The INDCs propose to rely on existing mechanisms without a review of its earlier outcomes. Several
reports, even from the government, have questioned the efficacy of earlier programmes.
Fixing concrete targets for afforestation also open up uncomfortable questions for the government
such as the availability of land for greening.
The proposed target is isolated from the other sectoral growth targets such as those under clean
energy through nuclear, clean coal and hydro power projects. The commitments to expand these
Current Affairs 104 October-2015

sectors do not acknowledge their footprint on forest areas. Almost all such energy projects would
require the diversion of forest land under the Forest (Conservation) Act. And such forest diversion is
not accounted for in the growth projections made.
The current rate of forest diversion to other uses like coal mining, power generation, construction of
roads or ports is approximately 35,000 ha annually. There are pending project proposals that seek the
diversion of 3414.84 ha of forest land. These diversions are not taken into account.
The government has suggested that the participation of the private sector will green degraded forests.
However, the private sector has only shown a propensity for deforestation.
Even the scientifically trained bureaucracy has so far not been able to achieve national forestry
targets.
Even if the target is achieved, there is no guarantee that these areas will not be diverted for non-forest
use if the latter seems more beneficial in monetary terms.
Conclusion:
Indias intended contribution on forests to mitigate climate change ignores the rich history of landscape
management practices involving communities living in forests and is uninformed of the impact of the
growing energy sector on forests. Without both these, Indias INDCs will not create any forests with roots on
the ground.
2. Diogenes canaliculatus: This species is light
brown or tan and is named after the
Paper 3 Topic: Biodiversity.
longitudinal furrows on the outer surface of
the arm of the left chelate leg. These are
Four new crab species found in Kerala
narrow bodied and live inside a shell shaped
Researchers at the Department of Aquatic Biology
like an elephant tusk. Even this belongs to
and Fisheries, University of Kerala, have reported
the family Diogenidae.
the discovery of four new species of crab from the
3.
Pagurus spinossior: It was collected from
Kerala coast. These discoveries highlight the
Neendakara, Kollam. It belongs to another
crustacean diversity in the State.
hermit crab family Paguridae known as right
About the species:
handed crabs and is tan in colour. The name
1. Paguristes luculentus: This hermit crab
spinossior refers to the strong armature on
species was collected off the coast of
the clawed legs of the species.
Kollam. It represents the ninth of the genus
4. Afropinnotheres ratnakara: This is a new
known from Indian waters. The species
species of pinnotherid crab. It was found
name luculentus (meaning colourful) refers
inside the brown mussel ( Perna perna ) at
to the livid living colour of the crustacean. It
Kovalam. The species is named ratnakara
belongs to the family Diogenidae, which are
which means Indian Ocean in Sanskrit, as
left handed hermits because the left claw is
the genus was reported for the first time
larger.
from the Indian Ocean.

26 October 2015

Hermit crabs:
Hermit crabs are ubiquitous animals often
not considered to be true crabs as they lack
an external shell on their soft abdomen
which leaves them vulnerable to predators.
To protect themselves, they live in

abandoned gastropod (snail) shells and often


select larger shells as they grow up.
Their last two pairs of legs are small and
modified and, along with their uropods
(appendages at the end of the abdomen), are
used to clamp onto the internal whorls of the

Current Affairs 105 October-2015

shell.
More than 40 species of hermit crabs were
documented from the Kerala coast during the
research project.
sources: the hindu.
Paper 3 Topic: Indian Economy and
issues relating to planning, mobilization
of resources, growth, development and
employment.

Sin tax for alcohol, tobacco industries in


GST regime
Alcohol and tobacco industries will soon have to
pay more taxes towards an additional sin tax under
the proposed GST structure.
There is a provision in the proposed GST bill
under which the sinful industries such as
alcohol and tobacco will have to pay an
additional tax. However, the rate at which
this tax would be levied under the proposed
GST regime is not yet decided.
What is Sin tax?
Sin tax is a globally prevalent practice
under which products like alcohol and
tobacco attract higher rates of tax. Typically,
sin tax is an excise tax that is levied on
products and services considered to be bad
for health or society such as alcohol, tobacco
and gambling.
These additional taxes are also seen as
efforts to discourage people from use of such
products or services. Besides, such taxes are
often the most common measures by the
governments to shore up their tax revenues
as people generally refrain from opposition
to such levies as they are indirect in nature
and affect only their end users.
GST:
GST is being seen as one of the biggest tax
reforms in the country. It will subsume
various taxes like excise, service tax, sales
tax, octroi, etc, and will ensure a single
indirect tax regime.
The Finance Ministry is currently seeking
inputs from the industry and other
stakeholders at national, state and local
levels on the Goods and Services Tax (GST)
law.
While the Constitution Amendment Bill to
roll out GST law has been passed in Lok
Sabha, it is awaiting clearance from the
Rajya Sabha where the ruling NDA lacks a

majority.
The government is currently undertaking the
preparatory work necessary for GST
implementation.
sources: BS, pib.
Paper 3 Topic: Disaster and disaster
management.

NDRF ties up with 30 PSUs like ONGC


and GAIL to tackle disasters
The NDRF, countrys elite disaster mitigation
combat force, has tied up with about 30 PSUs in
order to develop mutual understanding and enhance
capabilities to jointly tackle man-made or natural
emergencies.
A coordination meeting in this regard was
held recently between the National Disaster
Response Force (NDRF) and prominent
Public Sector Undertakings including
ONGC and GAIL.
The NDRF has proposed to these large
establishments that it could extend its skill
modules like on tackling nuclear, biological
and chemical disasters to train their
personnel. Once trained, the workforce of
the PSUs can effectively act to help the
NDRF as force multipliers during big
challenges and they can render a relief and
rescue task independently when any state
government or local administration seeks
their help.
This is the first of its kind initiative, which
will ensure that in case of any emergency or
disaster the two sides can come together and
act as one strong unit and also ensure that in
case of small or mid-level exigencies these
units can function as first responders till the
NDRF arrives on the scene.
NDRF:
The Disaster Management Act has made the
statutory provisions for constitution of National
Disaster Response Force (NDRF) for the purpose of
specialized response to natural and man-made
disasters.
Two national calamities in quick succession
in the form of Orissa Super Cyclone (1999)
and Gujarat Earthquake (2001) brought
about the realization of the need of having a
specialist response mechanism at National
Level to effectively respond to disasters.
This realization led to the enactment of the
DM Act on 26 Dec 2005.

Current Affairs 106 October-2015

NDRF has been proving its importance by


highly skilled rescue and relief operations,
regular and intensive training and retraining, capacity building & familiarization
exercises within the area of responsibility,
carrying out mock drills and joint exercises
with the various stakeholders.
Important functions:
Specialized response during disasters.
Proactive deployment during impending
disaster situations.
Impart basic and operational level training to
State Response Forces (Police, Civil
Defence and Home Guards).
Community Capacity Building Programme.
Public Awareness Campaign.
Why it is said to be UNIQUE?
It is the only dedicated disaster response
force of the world.
It is the only agency with comprehensive
response capabilities having multidisciplinary and multi-skilled, high-tech,
stand alone nature.
It consists of experienced paramilitary
personnel specially trained and equipped for
disaster response.
It has the capabilities for undertaking
disaster response, prevention, mitigation and
capacity building.
sources: et, pib, ndrf.
Paper 1 Topic: Role of women and
womens organization.

MoD Approves Induction of Women as


Fighter Pilots
The Ministry of Defence has approved the induction
of women into the Fighter (Combat) stream of the
IAF. With this decision to open up induction of
women in the fighter stream, women have become
eligible for induction in all branches and streams of
the IAF.
This progressive step was taken keeping in
view the aspirations of Indian women and is
in line with contemporary trends in Armed
Forces of developed nations.
Inducting women into the fighter stream
would provide them with an equal
opportunity to prove their mettle in combat
roles as well.
The first women pilots would be selected
from the batch which is presently
undergoing flying training at Air Force

Academy. After successful completion of abinitio training, they would be commissioned


in the fighter stream in June 2016.
The IAF is presently inducting women in the
Transport and Helicopter stream of the
flying branch, Navigation, Aeronautical
Engineering, Administration, Logistics,
Accounts, Education and Meteorology
branches.
Role of women in other forces:
Presently, the Indian Army is inducting
women into the Signals, Engineers, Army
Aviation (Air Traffic Control), Army Air
Defence, Electronics & Mechanical
Engineers, Army Service Corps, Army
Ordinance Corps, Intelligence Corps, Army
Education Corps and Judge Advocate
Generals Branches/Cadres.
The Indian Navy is inducting women in the
Judge Advocate Generals, Logistics,
Observer, Air Traffic Controller, Naval
Constructor and Education branches/cadres.
The Ministry of Defence has taken up a
comprehensive review pertaining to induction of
women in Armed Forces, both in short service
commission and permanent commission (SSC &
PC) and once finalised more and more branches
would be opened up for induction of women to give
them the space which they deserve in the Armed
Forces of the country.
sources: pib.
1. Question

It is said that sensitivity of the agriculture sector


to the monsoon has been moderating. What may
be the reasons for the same?
1. Rising share of irrigation
2. Sharp increase in the share of small-sized
loans in agricultural credit
3. Improvement in yield
4. Sharp increase in the share of credit being
used for capital formation in agriculture
Select the correct answer using the codes given
below:
a) 1, 2 and 4 Only
b) 1, 3 and 4 Only
c) 1 and 3 Only
d) All of the above
Ans: c.
http://www.thehindu.com/business/deficiency-ofrainfall-may-hit-rural-income/article7750219.ece
Statements 2 and 4 are wrong in themselves. Refer
to Box 5.2, Economic Survey 2014-15 Volume 1 for
the correct statements. And this box provides an

Current Affairs 107 October-2015

analysis of agricultural credit patterns for the past


two decades not the reasons for reduced
sensitiveness of Indian agriculture to the monsoons.

2. Question

Despite a deficient monsoon for the second year


in a row, food inflation has been on the decline
this year. What may be the reasons for the same?
1. Restricted hikes in Minimum Support
Prices
2. A global supply glut of agri-commodities
Select the correct answer using the codes given
below:
a) 1 Only
b) 2 Only
c) Both 1 and 2
d) Neither 1 or 2
Ans: c.
http://www.thehindu.com/business/deficiency-ofrainfall-may-hit-rural-income/article7750219.ece

3. Question

Consider the following statements associated


with the plant Opium Poppy:
1. India is sandwiched between two
producing regions of it the Golden
Triangle and the Golden Crescent
2. Its seeds are used in Indian cuisine
Which of the above statements are incorrect?
a) 1 Only
b) 2 Only
c) Both 1 and 2
d) Neither 1 or 2
Ans: d.
Opium is the source of many drugs,
including morphine. For curiositys sake: In Indian
cuisine white poppy seeds are added for thickness,
texture and also give added flavor to the recipe.
Commonly used in the preparation of korma,
ground poppy seed, along with coconut and other
spices, are combined as the masala to be added at
the end of the cooking step. It is quite hard to grind
them when raw, so they are normally dry fried, and
then mixed with a little water to get the right paste
consistency.

http://pib.nic.in/newsite/PrintRelease.aspx?
relid=128634

https://en.wikipedia.org/wiki/Golden_Triangle_(Sou
theast_Asia)

1. Bonded and child labour


2. Right to clean environment
3. Right to health
4. Rights of prisoners
5. Rights of women
Select the correct answer using the codes given
below:
a) 1, 4 and 5 only
b) 2 and 3 only
c) 1, 3, 4 and 5 only
d) All are correct
Ans: d.
Any form of human rights violations can be
addressed by the commission. It may have been
caused due to violations of the Right to Health or
Right to a clean environment or any other human
right.
http://pib.nic.in/newsite/PrintRelease.aspx?
relid=128648

4. Question

Violations concerned with which of the


following issues can the National Human Rights
Commission sometimes address?

5. Question

Consider the following statements with regard to


Exercise Malabar, a naval military exercise
involving India and the USA:
1. It is an annual exercise
2. 2015 marks the first year in which it
transformed into a multilateral exercise
3. It is always held off Indias western coast
Which of the above statements is/are incorrect?
a) 1 and 3 only
b) 3 Only
c) 2 and 3 Only
d) All are correct
Ans: c.
Exercise Malabar is an annual bilateral naval
exercise involving the United States and India.
Participation has been expanded in some years to
include Japan and/or Australia and/or Singapore.
When the 2007 edition of the exercises in the Bay
of Bengal were expanded to include Japan and
Australia, Beijing lashed out at what looked like the
so-called quadrilateral security dialogue or quad
between the United States, Australia, Japan and
India operating at sea. India had included Japan also
in the 2009 and 2014 exercises held in the northwestern Pacific
http://www.thehindu.com/news/international/notworried-about-malabar-drillsbeijing/article7754163.ece

http://thediplomat.com/2015/07/us-official-callsfor-permanent-expansion-of-malabar-exerciseswith-india/

Current Affairs 108 October-2015

6. Question

Participating in naval exercises is one way to


manage a new geopolitical reality Chinas
growing naval profile in the Indian Ocean.
Which of the following naval exercises has the
Indian Navy conducted or participated in?
1. Milan
2. Varuna
3. Konkan
4. Simbex
5. IBSAMBAR
6. Malabar
Select the correct answers using the codes given
below:
a) 1, 2, 3 and 6 only
b) 3, 4, 5 and 6 only
c) 2, 4, 5 and 6 only
d) All the above
Ans: d.

India has not been averse to multilateral and


bilateral exercises in themselves. Indeed, over the
years it has conducted and participated in many.
IBSAMBAR (India, Brazil & South Africa), Varuna
(India & France), Milan (16 countries from the
Indian Ocean Region), Simbex (India & Singapore),
KONKAN (India & U.K.) are just a few in a long
list of naval exercises that have held in the three sea
bodies surrounding India. These exercises have not
drawn protests from China as most of the countries
involved have not been claimants in the South
China Sea
http://thediplomat.com/2015/07/indias-multilateralnaval-reluctance/

http://www.thehindu.com/news/international/notworried-about-malabar-drillsbeijing/article7754163.ece

Insights into Editorial: We Need to Talk About


the Brahmaputra
27 October 2015
China recently operationalized its Zangmu Hydropower Project on the Yarlung Zangbo River
(Brahmaputra River). The Hydropower Station is located in Gyaca county of the Shannan prefecture in
China, and is considered to be Tibets largest such facility. It has also raised fresh concerns in downstream
India, especially in Arunachal Pradesh and Assam. There are also other upcoming projects such as Dagu,
Jiacha and Jiexu in different stages of planning/construction, which have further raised concerns in India.
India had prior information about Chinas plans of developing hydropower on the Yarlung Zangbo.
However, India and China do not have any formal river water sharing agreements. India is worried
mainly because these projects could be used to regulate and control water downstream, leading to
scarcity in the northeastern states.
What should be done to reduce the mistrust?

India and China need to move towards a framework of engagement and dialogue on the Brahmaputra,
as a precursor to any negotiation.
The dialogue needs to be inclusive, providing a platform to various stakeholders and identify new
approaches to address the common problem.
The dialogue must address the concerns of various stakeholders and sub-national units within the
respective riparian countries (China, India, Bangladesh and Bhutan).
Current Affairs 109 October-2015

An attempt must be made through the dialogue to bring together the interconnected research on
rivers, infrastructure building and other related aspects, ranging from politics, engineering, geology,
economics, social scientists, hydrologists, environmentalists, activists forums, local stakeholders,
which is now missing.
Expert-Level Mechanisms currently existing between India and China on hydrological data must be
revived.
The success of the internal dialogue processes will mainly depend on the voice of sub-national units such as
Arunachal Pradesh and Assam.

Conclusion:
Current Affairs 110 October-2015

Given the huge population growth in China, India and Bangladesh, the water may become even scarcer in
the coming years. Hence, India and China need to engage purposively in co-managing the rivers of the
region, and thereby ensure that the development of the region is not impeded by unnecessary posturing on
the sensitive issue of water, which can impact other bilateral issues. Sub-regional cooperation groupings such
as the Bangladesh-China-India-Myanmar (BCIM) Cooperation Framework can also create an enabling
environment for mutually inclusive sub-regional participation and water resources sharing.
enables armed forces to resort to the provisions of
AFSPA.
Who declares an area as disturbed?
I will continue my fight alone:
The choice of declaring any area as
Sharmila
disturbed vests both with state and central
Disappointed by the absence of supporters at a very
government.
crucial stage in her campaign, Irom Sharmila
Special powers provided to armed forces:
recently said that she would continue her campaign
After an area comes under the ambit of AFSPA, any
against AFSPA all alone and would not allow it to
commissioned officer, warrant officer, nonbe sabotaged.
commissioned officer or another person of
Who is Irom Sharmila?
equivalent rank can use force for a variety of
reasons while still being immune to the prosecution.
Ambit:
The act was passed on 11 September 1958
by the parliament of India to provide special
legal security to the armed forces carrying
out operations in the troubled areas of
Arunachal Pradesh, Assam, Meghalaya,
Manipur, Mizoram, Nagaland, Tripura
(seven sisters).
In 1990 the act was extended to the state of
Jammu and Kashmir to confront the rising
insurgency in the area.
In Manipur, despite opposition from the
Central government, state government
withdrew the Act in some parts in Aug,
2004.
The government can declare AFSPA in the
following conditions:
When the local administration fails to deal
with local issues and the police proves
inefficient to cope with them.
When the scale of unrest or instability in the
state is too large for the police to handle.
She is the woman crusader who has been on a
Legal provisions of AFSPA:
fast-unto-death since November 2000
In an area declared, disturbed an army
demanding the repeal of the Armed Forces
officer is legally free to carry out following
(Special Powers) Act in Manipur.
operations:
Background:
Fire upon or otherwise use force, even to the
The High Court in Delhi is hearing an attempt to
causing of death, against any person who is
commit suicide case against her. However, the
acting in contravention of any law against
district and sessions court in Imphal had ordered
assembly of five or more persons or
Ms. Sharmilas release finding her not guilty of the
possession of deadly weapons.
charge.

Destroy any shelter (private or govt.) from


Armed Forces (Special Powers) Act:
It is an Act empowering armed forces to deal
which armed attacks are made or likely to be
effectively in Disturbed Areas. Any area which is
made or attempted to be made.
declared Disturbed under the disturbed areas act

27 October 2015

Current Affairs 111 October-2015

Arrest any person without warrant who has


committed a cognizable offence or against
whom a reasonable suspicion exists that he
has committed or is about to commit a
cognizable offence.
Enter and search, without warrant, any
premises for purpose of arrest or to recover
any person, arms, explosives.
Search and seize any vehicle suspected to be
carrying an offender or any person against
whom any reasonable suspicion exists that
he has or is about to commit an offence.
Provide legal immunity to the army
personnel found involved in any violation or
ethical breach i.e., they cannot be sued or
prosecuted.
Common people see it as Right to Kill Act. Since
its inception many Human Rights organizations and
civil societies have been opposing it for the
following reasons:
It makes no distinction between a peaceful
gathering of five or more people and a
berserk mob.
The law also states that, no prosecution can
be initiated against an officer without the
previous sanction of the Central
government.
The decision of the government to declare a
particular area disturbed cannot be
challenged in a court of law.
In 2005 the Jeevan Reddy Commission said that
AFSPA should be repealed and the clauses that are
required should be included in other Acts.
sources: the hindu, pib.
Paper 2 Topic: Government policies
and interventions for development in
various sectors and issues arising out of
their design and implementation.

After IAF, Navy opens its doors to women


pilots
Following the Air Force, even the Navy has decided
to throw open its doors to women fighter pilots. All
flying areas in the Navy will be opened for women,
except where it requires staying overnight on
carriers and ships.
However, until the infrastructural needs are
addressed, they will be shore-based.
The army and the Navy are also looking at
appointing women in combat roles.
These moves are aimed at giving women an almost
equal status as long as there are no logistical,

infrastructure and training issues. The Defence


Ministry had recently announced that the first batch
of women fighter pilots would be serving the Indian
Air Force by June 2017.
Issue of Permanent Commission:
In September 2025, in a landmark
judgement, the Delhi High Court had
granted Permanent Commission for women
and had pulled up the Defence Ministry and
the Navy for sexist bias and blocking
womens progress.
However, the defence minister has observed
that there was no gender bias in the Navy.
What is a permanent commission?
A permanent commission means a career in the
Army/Navy till one retires. A permanent
commission also entitles 20 years of service and a
pension.
Why women officers are demanding for Permanent
Commission?
Due to their limited service span, women officers
are not eligible for pension, which requires a
minimum 20 years of service. Currently, in the navy
women officers are entitled only to short service
commissions for a maximum of 14 years.
sources: the hindu, pib, bs.
Paper 2 Topic: Issues relating to
development and management of Social
Sector/Services relating to Health,
Education, Human Resources.

Increase of Financial Assistance under the


Rashtriya Arogya Nidhi (RAN)
To further ease access to financial assistance under
Rashtriya Arogya Nidhi (RAN), the Government
has decided to enhance the financial powers
delegated to designated Central Government
hospitals/institutes from Rs. 2 lakh to Rs. 5 lakh for
providing financial assistance in cases where
emergency surgery is to be conducted.
Rashtriya Arogya Nidhi (RAN):
Rashtriya Arogya Nidhi was set up under the
Ministry of Health & Family Welfare in
1997.
The scheme provides for financial assistance
to patients, living below poverty line who is
suffering from major life threatening
diseases, to receive medical treatment at any
of the super specialty hospitals/institutes or
other Govt. hospitals.
The financial assistance to such patients is
released in the form of one time grant to
the Medical Superintendent of the hospital in

Current Affairs 112 October-2015

which the treatment is being received.


Under the scheme, Central Government also
provides Grant in aid to States/Union
Territories (with legislature) to set up state
illness fund to the extent of 50% of
contribution made by State Govt/Union
Territories.
Financial Assistance is given to patients
living in their respective States/UTs under
State illness fund up to Rs.1.5 lakhs in an
individual case. However, in cases where the
quantum of financial assistance is likely to
exceed Rs.1.5 lakhs, those are referred to
RAN for consideration.
sources: pib, mohfw.
1. Question

Consider the following statements:


Assertion (A): Indias Blood Safety Programme
is the primary responsibility of Department of
pharmaceuticals, Ministry of Chemicals and
Fertilisers
Reason (R): As per the Indian laws, human blood
is treated as a drug under the Drugs &
Cosmetics Act
Which of the above statements is/are true?
a) A and R both are true, and R is the
correct explanation for A
b) A and R both are true, and R is the
NOT the correct explanation for A
c) A is incorrect, R is correct
d) A and R both are incorrect
Ans: c.
Access to safe blood is mandated by law, and is the
primary responsibility of National Aids Control
Organisation (NACO). The specific objective of the
Blood Safety Programme is to ensure reduction in
the transfusion associated with HIV transmission to
0.5 percent, while making available safe and quality
blood within one hour of requirement in a health
facility.
http://www.thehindu.com/news/national/misleading
-ad-on-blood-still-on-air/article7754442.ece

2. Question

A well organised Blood Transfusion Service is a


vital component of any health care delivery
system. In this context, which of the following
statements is/are incorrect?
1. Indias lack of a central blood collection
agency
2. A Supreme Court rulinghas banned paid
donors and unlicensed blood banks
Select the correct answer using codes given
below:

a) 1 Only
b) 2 Only
c) Both 1 and 2
d) Neither 1 or 2
Ans: d.
http://www.thehindu.com/news/national/misleading
-ad-on-blood-still-on-air/article7754442.ece

http://www.bbc.com/news/business-30273994

3. Question

The State shall endeavour to secure for the


citizens a uniform civil code throughout the
territory of India which article in the Indian
Constitution places this duty upon the State?
a) Article 43
b) Article 44
c) Article 243
d) The Indian Constitution makes no such
provision
Ans: b.
http://www.thehindu.com/todays-paper/tpnational/apex-court-tells-centre-to-work-on-civilcode/article7755110.ece

http://indiankanoon.org/doc/1406604/

4. Question

Consider the following statements about ASCI


(Advertising Standards Council of India):
1. It comes under the Ministry of Consumer
Affairs, Food and Public Distribution
2. Any consumer can lodge complaints
(with ASCI) on what they may consider
to be misleading advertisements
Which of the above statements are incorrect?
a) 1 only
b) 2 only
c) Both 1 and 2
d) Neither 1 or 2
Ans: a.
ASCI is a voluntary, self-regulatory body. It has
partnered with the Department of Consumer Affairs
to tackle misleading advertisements.
http://www.ascionline.org/

http://www.thehindu.com/news/national/misleading
-ad-on-blood-still-on-air/article7754442.ece

5. Question

Consider the following statements:


1. Meloxicam affects cattle as well as
vultures the same way as diclofenac
2. Ketoprofen affects cattle the same way as
diclofenac, but is harmless for vultures
Which of the above statements is/are correct?

Current Affairs 113 October-2015

a) 1 only
b) 2 Only
c) Both 1 and 2
d) Neither 1 or 2
Ans: d.
Its the other way round.
Meloxicam affects cattle the same way as
diclofenac, but is harmless for vultures. Ketoprofen
affects cattle as well as vultures the same way as
diclofenac.
http://www.thehindu.com/todays-paper/taking-cuefrom-centre-state-bans-a-drug-to-savevultures/article7755262.ece

6. Question

Sankalp, launched by the Department of


Pension & Pensioners Welfare is an initiative
a) for enhancing efficiency in
Government systems
b) to channelize the energy, experience
and skill of retiring Government servants

into meaningful voluntary work


c) to monitor and redress, in a timely
manner, any delay in grant of pension and
other retirement benefits
d) Both a and b
Ans: b.
Approximately 40,000 employees retire every year
from the Central Government Civil establishments
alone. Besides the average life expectancy today has
increased to 69.2 years. A pensioners cognitive
skills by and large remain intact for ten to fifteen
years. These groups of personnel have maturity,
experience and stability.
The voluntary or other organisations working
towards building society may be looking for
expertise, skill and maturity which can be offered
by these retired government servants.
Accordingly, an activity called Sankalp has been
initiated and a web portal of the same name has
been launched.
http://pib.nic.in/newsite/mbErel.aspx?relid=128672

Insights into Editorial: The Age of Intolerance


28 October 2015
Few recent incidents- murders of two prominent iconoclasts (a person who attacks or criticizes
cherished beliefs or institutions.), ban on the sale of meat during the Jain Festival of Paryushan and
lynching in Dadri- show that Indias image as a pluralist and secular democracy is under threat. These
incidents also point out at the sharply divided polity in the country. Such incidents, if ignored, could
spell danger, particularly in a multi-layered, multi-religious and multi-ethnic country like India.
All these incidents point out the danger posed by majoritarianism [It is a traditional political philosophy or
agenda which asserts that a majority (sometimes categorized by religion, language, social class or some
other identifying factor) of the population is entitled to a certain degree of primacy in society, and has the
right to make decisions that affect the society wikipedia.], at the expense of minorities of every hue. These
incidents have also led to an impression that an atmosphere of intolerance overlaid with shades of
authoritarianism pervades the country and question the inclusive character of India. It is therefore time for
introspection.
How the government has reacted?
Reaction from the government has been weak and tardy. Condemnations of these incidents by the
government authorities have been inadequate, considering the incendiary nature of some of them.
There has also been the tendency in official circles to paint the events with a political brush. Such
inactivity has further polarised the atmosphere.
President Pranab Mukherjee appealed to the nation that the core values of Indias civilization that
celebrate diversity, plurality and tolerance should not be allowed to wither away. However, there
does not appear to be any genuine desire for reconciliation.
Why it is a cause for concern?
In this age of connectivity, sharing of individual experiences among lakhs of people using the Internet could
lead to a mighty ground-swell of protest that could deal a blow to the institutions of state much like what
happened following the gang rape of a young girl in Delhi in December 2012. If the different streams of
dissent and discontent coalesce into a single stream, it could transform the protests into an uncontrolled
crisis. Hence government should intervene and take measures to prevent such events before it is too late.
Conclusion:
It is vital to preserve the pluralistic fabric of India. Hence, leaders across the political spectrum should work
Current Affairs 114 October-2015

together to sustain the inherent quality of a nation that has welcomed people of all faiths and all
denominations for centuries. Hence, it is the right time to take certain visionary steps rather than attempt to
keep opponents off-balance.

28 October 2015
Paper 1 Topic: Art and culture.

Ancient temples in Mandya district unearthed


The Archaeological Survey of India, in its recent excavations in Karnataka, has found five temple complexes
defining Jaina identity.
These temples were found at Chikkabetta of Artipura in Mandya district, Karnataka.
Archaeologists say that these temples were built during the Western Ganga dynasty rule.
Significance of these findings:
These temples are believed to be the oldest
known archaeological find belonging to the
Western Ganga dynasty. Western Ganga dynasty
was said to have thrived from 4th century BC.
However, there were no evidences to prove that.
The earliest archaeological evidence available
was only from the 10th century inscriptions of
Shravanabelagola in Hassan district. But, the
latest archaeological discovery at Artipura
brings to light the temple relics of the early, first
quarter of the Ganga Dynasty in 925 AD or the
beginning of the 10th century.
Western Ganga Dynasty:
It was an important ruling dynasty of
ancient Karnataka. Some experts believe
that the dynasty lasted from about 350 to
1000 AD.
They are known as Western Gangas to
distinguish them from the Eastern
Gangas who in later centuries ruled over
Kalinga (modern Odisha).
The Western Gangas began their rule
during a time when multiple native clans
asserted their freedom due to the
weakening of the Pallava empire in
South India.

Current Affairs 115 October-2015

After the rise of the imperial Chalukyas of Badami, the Gangas accepted Chalukya overlordship and
fought for the cause of their overlords against the Pallavas of Kanchi. The Chalukyas were replaced
by the Rashtrakutas of Manyakheta in 753 AD as the dominant power in the Deccan. The Western
Gangas accepted Rashtrakuta overlordship and successfully fought alongside them against their foes,
the Chola Dynasty of Tanjavur. The defeat of the Western Gangas by Cholas around 1000 resulted in
the end of the Ganga influence over the region.
They showed tolerance towards all faiths. They are most famous for their patronage toward Jainism
resulting in the construction of monuments in places such as Shravanabelagola and Kambadahalli.
sources: the hindu, wiki.
Paper 3 Topic: Indian Economy.

India moves up in ease of doing business ranking


India now ranks 130 out of 189 countries in the ease of doing business 2016, according to a World Bank
report.
The original ranking for 2015 had been pegged at 142, which would give India a jump of 12 ranks,
but the WBs mid-year revision had bumped up Indias rank to 134.
The improvement in two indicators, starting a business and getting electricity, pushed India up the
ladder, according to the report.
The report also commended the legislative changes that eliminated the minimum capital requirement
and the requirement to obtain a certificate to start business operations.
Performance of other countries:
Singapore remains the easiest place to do business.
China is ranked 84 and Pakistan is at 138th place. Pakistan has slipped 10 spots from 128 last year
while China has moved six spots in a year from 90 since the last report.
Current Affairs 116 October-2015

New Zealand remained in the number-two position, followed by Denmark (3), South Korea (4), Hong
Kong (5), Britain (6) and the United States (7). Sweden moved up a notch to number eight, switching
places with Norway. Finland kept its 10th place.
The World Bank Doing Business report was started in 2002. The report reviews business regulations and
their enforcement across 189 countries. The report looks at the regulatory environment for small and
medium-sized companies to see how it hampers or helps them conduct business, from starting up and paying
taxes to registering property and trading across borders.
sources: the hindu.
Paper 2 Topic: transparency and accountability.

Info panel turning down more RTI requests now


Data from the Central Information Commissions website show that CIC has admitted fewer and fewer cases
every month this year, under the Right to Information Act.
Hence, RTI activists have asked for greater transparency in the process of turning down requests.
Data show that from September 2014 to June this year, the CIC admitted between 2,500 and 3,500
cases every month. However, since June, the number of cases the CIC admits has crashed
precipitously, falling to just 119 last month.
What the CIC says?
The CIC says it is not rejecting requests but is returning them for technical deficiencies, including
lack of proper identification.
However, it has not made public details of these requests being returned, nor why the need to return requests
has suddenly arisen.
Cases come before the CIC in two ways:
One, if an applicant is not satisfied with the response to his or her request for information from a
Central government authority, and with the verdict of the first appeal made to the authority
concerned, he or she can approach the CIC for the second appeal.
Second, if a citizen has a complaint his or her request was not taken or wrong information was
given or he or she has faced threats he or she can come directly before the CIC.
CIC:
The Central Information Commission (CIC) is set up under the Right to Information Act and is the
authorized body, established in 2005, under the Government of India.
The Chief Information Commissioner heads the Central Information Commission.
CIC hears appeals from information-seekers who have not been satisfied by the public authority, and
also addresses major issues concerning the RTI Act.
sources: the hindu, cic.
Paper 2 Topic: Government policies and interventions for development in various sectors and
issues arising out of their design and implementation.

Income Tax Dept sets up panel to simplify tax law


The Income Tax Department has set up a panel to help simplify the Income Tax Act, 1961, as part of the
governments move to improve the ease of doing business.
The committee will be chaired by Justice R.V. Easwar, a former judge of the Delhi High Court and
former president Income Tax Appellate Tribunal.
Objective of the committee is to study and identify the provisions or phrases in the Act that are
leading to litigation due to different interpretations, impacting the ease of doing business, and those
that can be simplified.
The committee is also tasked with suggesting alternatives to these provisions or phrases to bring
about predictability and certainty in tax laws without substantial impact on the tax base and revenue
collection.
Members of the committee include accountants, advocates and current and former bureaucrats from
the Indian Revenue Service.
sources: the hindu.
Current Affairs 117 October-2015

Paper 2 Topic: Transparency and accountability.

Centre backs publishing CMs photos in govt. advertisements


The Centre has sought a review of the Supreme Court judgment banning the publication of photographs of
political leaders and Chief Ministers in government-issued advertisements.
Centres arguments:
People have the right to know about government welfare work in a participative democracy, and
hence the ban should be lifted.
It was contrary to the federal structure of governance envisioned in the Constitution to prohibit the
publication of the pictures of Chief Ministers and Governors.
Background:
In May 2015, the Supreme Court issued guidelines for the publication of government advertisements
and held that publication of photographs of politicians and government functionaries, including Chief
Ministers, defeated the public interest behind advertising welfare schemes and encouraged
personality cults.
However, the court had exempted the President, the Prime Minister and the Chief Justice of India
from this ban, leaving it to their discretion to decide whether they wanted their pictures published in
the advertisements or not.
sources: the hindu.
Paper 2 Topic: Government policies and interventions for development in various sectors and
issues arising out of their design and implementation.

Maharashtra withdraws sedition order


The Maharashtra government recently told the Bombay High Court that it had withdrawn the controversial
circular on sedition.
The circular had left the field open for the police to register sedition charge against critics of the
government. Hence, it was widely opposed.
Cartoonist Aseem Trivedi, who was arrested in 2012 under the sedition charge for his political
cartoons and advocate Narendra Sharma had sought to quash the circular on the ground that it
violated the constitutional rights of people and was liable to be misused.
About the circular:
The Maharashtra government through a circular had given police the powers to take action against
those critical of the state or central government if it deemed such critiques to be particularly
offensive.
The order allowed the police to invoke a colonial era sedition clause 124-A of the Indian Penal
Code against any person who by words, either spoken or written, or by signs or by visible
representation or otherwise, brings or attempts to bring into hatred or contempt, dissatisfaction and
provoking violence against the central or the state government.
According to the circular those who lawfully try to change the government without invoking anger or
disaffection should not be charged with sedition. But it is still left to the police to determine whether
someone is employing hatred and contempt while democratically protesting against the
government.
sources: the hindu.

Current Affairs 118 October-2015

Insights into Editorial:


Role of Civil Society Practitioners in Improving
Relationship Between India and Pakistan
29 October 2015
Frequent ceasefire violations along the Line of Control and exchange of fiery speeches between the
two countries have once again put on hold the High level talks between India and Pakistan. This has
given rise to a feeling that the situation between the two countries can never be improved. With
government to government ties standstill, many believe that the situation can only be improved by the
participation of citizens of both the countries. By working together outside the government, Indian and
Pakistani civil society practitioners can build meaning and constructive connections between the two
countries.
Many civil society-led initiatives trade expositions, cultural festivals, and Track Two dialogues (Track II
diplomacy refers to non-governmental, informal and unofficial contacts and activities between private
citizens or groups of individuals, sometimes called non-state actors'. It contrasts with track I diplomacy,
which can be defined as official, governmental diplomacy that occur inside official government channels.)
could play a vital role in building links between Indians and Pakistanis. Taking the form of literary
festivals, womens dialogues, youth exchanges, joint chambers of commerce, security forums, student
conferences, digital ideas hubs, and so on, such initiatives have only increased in the recent times.
Various initiatives that are aimed at improving India Pakistan relations:
Influential corporate leaders from both the countries convene through the Pakistan India Joint
Business Forum to explore ways of jump-starting cross-border commerce.
The Ottawa and Chaophraya Dialogues and a newer initiative called South Asian Voices regularly
bring defence practitioners together, in person and online, to formulate workable solutions to
intractable political and military problems.
Exchange for Change has linked thousands of Indian and Pakistani students and enabled scores to
travel between the two countries.
The India-Pakistan Regional Young Leaders Forum enables young adults to collaborate on media and
public service projects.
Importance of civil society-led initiatives:
Civil society-led efforts can keep both the countries talking even when official lines of communication are
closed. Public support born of a robust civil society can generate pressure and momentum for reconciliation.
Such efforts may not be a panacea for immediate peace, but over the longer term, they lay a sturdy
foundation of goodwill between the governments and among large segments of the public in both countries.
Such initiatives also make governments further works easier.
Conclusion:
However, civil society will not, on its own, bring peace to the subcontinent. Ultimately, the governments
have to find their way to the negotiating table and start chipping away mistrust. Civil society can only
continue to lay the groundwork for a more peaceful future.
.
Concepts:
Tracks of diplomacy
Traditionally, the term diplomacy referred to interaction between nation-states. More recently, however,
scholars have delineated several levels of diplomacy. Tracks 1 and 2 are the most frequently used terms. A
composite term is multitrack diplomacy.
Track 1 diplomacy: Official discussions typically involving high-level political and military leaders and
focusing on cease-fires, peace talks, and treaties and other agreements.
Track 2 diplomacy: Unofficial dialogue and problem-solving activities aimed at building relationships and
encouraging new thinking that can inform the official process. Track 2 activities typically involve influential
academic, religious, and NGO leaders and other civil society actors who can interact more freely than highranking officials. Some analysts use the term track 1.5 to denote a situation in which official and nonCurrent Affairs 119 October-2015

official actors work together to resolve conflicts.


Track 3 diplomacy: People-to-people diplomacy undertaken by individuals and private groups to encourage
interaction and understanding between hostile communities and involving awareness raising and
empowerment within these communities. Normally focused at the grassroots level, this type of diplomacy
often involves organizing meetings and conferences, generating media exposure, and political and legal
advocacy for marginalized people and communities.
Multitrack diplomacy: A term for operating on several tracks simultaneously, including official and
unofficial conflict resolution efforts, citizen and scientific exchanges, international business negotiations,
international cultural and athletic activities, and other cooperative efforts. These efforts could be led by
governments, professional organizations, businesses, churches, media, private citizens, training and
educational institutes, activists, and funders.
Source: USIP

29 October 2015
Paper 1 Topic: Mechanisms, laws,
institutions and Bodies constituted for
the protection and betterment of these
vulnerable sections.

Govt. limits surrogacy to infertile Indian


couples
The Centre recently told the Supreme Court that it
does not support rent-a-womb commercial
surrogacy and would make such exploitation of
women and children wombs punishable under the
law. However, the centre has said that it would
allow only infertile Indian married couples to go
for surrogacy.
This provision will be made part of The
Assisted Reproductive Techniques
(Regulation) Bill, 2014, which is currently
awaiting the comments of states.
A surrogacy arrangement or surrogacy agreement
is the carrying of a pregnancy for intended parents.
The Assisted Reproductive Technology (ART)
Regulation Bill, 2014:
The proposed legislation aims at proper
regulation and supervision of ART clinics
and banks in the country and to prevent
misuse of this technology, including
surrogacy, and for safe and ethical practice
of these services.
Under the provisions of the bill foreign
nationals will be barred from
commissioning surrogacy in India.
The Bill has stern rules for surrogates. It
says that any woman agreeing to act as a
surrogate shall be duty-bound not to engage
in any act, including unprotected sex that
may harm the foetus during pregnancy and
the child after birth, until the time the child
is handed over to the designated
commissioning couple.

Need for such legislations:


7,000-8,000 surrogacy clinics operate in
Delhi alone. Most of them operate illegally,
according to estimates.
Around 10,000 foreign couples visit India a
year to commission surrogacy with nearly
30% being single or homosexual.
Rs 2.5-6 lakh is the amount charged by
surrogates in India.
$400mn is the surrogacy industrys value in
India as per estimates.
sources: the hindu, indiatoday, pib.
Paper 3 Topic: Awareness in IT.

Zero Rating will enhance Net access:


Zuckerberg
During his recent visit to India, Facebook CEO
Mark Zuckerberg said that Facebooks Free Basics
and Zero Rating do not violate Net neutrality,
instead they are aimed at enhancing access to the
Internet in the developing world, including India.
He also said that most of the folks who are
pushing for Net neutrality have access to
the Internet already. But the people who are
not yet on the Internet cant sign an online
petition pushing for increased access to the
Internet.
He also tried to distinguish zero-rating
services from other instances when
companies charge different rates for
services or when a telecom operator tries to
give an advantage to its own services.
According to him, it was for the latter that
Net neutrality regulations are required.
What is Zero rating?
Zero Rating is a practice by which Internet
operators offer free data for specific applications.
Advocates of Zero Rating services have argued that
this enables those offline to try online services,

Current Affairs 120 October-2015

thereby bridging the digital divide.


Free basics:
It was earlier called as internet.org. It has partnered
with Reliance Communications as the telecom
service provider in India. Internet.org had come
under intense criticism from Net neutrality activists
in India that it favours Facebooks own services.
Other notable points:
Facebook is the worlds second largest
Internet company in terms of market
capitalisation and the largest social media
firm.
India provides the second largest user-base
of Facebook after the U.S. with 130 million
users, nearly one-tenth of the overall users.
However, Facebooks annual report of 2014
says that its average revenue per user for
the U.S. and Canada was nearly six times
than that for the Asia-Pacific region
(including India), suggesting the
importance of greater Internet access in
India for Facebook.
sources: the hindu.
Paper 3 Topic: Government policies
and interventions for development in
various sectors and issues arising out of
their design and implementation.

Nutrition bureau axed, anti-poverty


schemes starved
The National Nutrition Monitoring Bureau
(NNMB) has been shut down by the Union Health
Ministry.
Why it was shut down?
It is being said that since the bureau was
running in project mode and Government
programmes that run in a project mode for
this long are not sustainable, it was shut
down. The bureau was also among many
research organisations hit by the
governments cash crunch.
The Union Science and Technology
Ministry had also recently asked scientific
research organisations under the Council of
Scientific and Industrial Research to start
self-financing projects, turning research
projects into for-profit ventures over the
next two years.
About NNMB:
It was established in 1972 by the Indian
Council of Medical Research (ICMR) with
a mandate to generate data on the

nutritional status of socially vulnerable


groups.
The bureau had been critical in informing
the governments poverty alleviation
interventions with periodic assessments of
nutrient deficiency among tribal
communities, pregnant women, adolescents
and at-risk elderly population in India.
Significance of NNMB:
In countries such as India where nutrition
has a cultural significance, organisations
like NNMB provide a good understanding
of what people eat and what, therefore, can
be culturally accepted nutritional
interventions.
NNMB plays a very important role in
projecting data in terms of what people are
eating. The data gathered by the NNMB
informs the policy intervention to address
under-nutrition.
The bureau was the only organisation
involved in continuous collection of data on
actual dietary intakes of households as a
whole as well as by individuals belonging
to different age and physiological groups in
different States.
sources: the hindu, nnmb.
Paper 2 Topic: Welfare schemes for
vulnerable sections of the population
by the Centre and States.

First affordable housing projects under


Housing for All Mission
Chattisgarh has become the first state to get central
governments approval for taking up affordable
housing projects in urban areas under Prime
Ministers Awas Yojana (PMAY) launched in June
this year.
The Centre has approved state
governments proposals for building 26,034
houses in 11 cities and towns in the state.
Of these, 12,670 are for beneficiaries
belonging to Economically Weaker Sections
(EWS) and 13,364 for Low Income Group
(LIG).
The centre has also approved the release of
first installment of Rs.76 cr to Chattisgarh
government. State Urban Development
Agency is the nodal agency for housing
mission while Chattisgarh Housing Board is
the implementing agency.
Housing for All:

Current Affairs 121 October-2015

It is scheme which proposes to build 2 crore houses


across the nation by 2022.
The components of the scheme are as follows:
Slum rehabilitation of Slum Dwellers with
participation of private developers using
land as a resource.
Promotion of affordable housing for weaker
section through credit linked subsidy.
Affordable housing in partnership with
Public & Private sectors.
Subsidy for beneficiary-led individual
house construction or enhancement.
Details of the scheme:
The scheme will be implemented as a
Centrally Sponsored Scheme except the
credit linked subsidy component, which
will be implemented as a Central Sector
Scheme.
The scheme prescribes certain mandatory
reforms for easing up the urban land market
for housing, to make adequate urban land
available for affordable housing.
Houses constructed under the mission
would be allotted in the name of the female
head of the households or in the joint name
of the male head of the household and his
wife.
A Technology Sub-mission under the
Mission would be set up to facilitate
adoption of modern, innovative and green
technologies and building material for
faster and quality construction of houses.
The Technology Sub-Mission will also
facilitate preparation and adoption of layout
designs and building plans suitable for
various geo-climatic zones. It will also
assist States/Cities in deploying disaster
resistant and environment friendly
technologies.
The Technology Sub-Mission will
coordinate with various regulatory and
administrative bodies for mainstreaming
and up scaling deployment of modern
construction technologies and material in
place of conventional construction. The
Technology Sub-Mission will also
coordinate with other agencies working in
green and energy efficient technologies,
climate change etc.
In the spirit of cooperative federalism, the
Mission will provide flexibility to States for
choosing best options to meet the demand

of housing in their states.


sources: pib.
Paper 2 Topic: Agreements involving
India.

Agreement on digital preservation and


conservation of rare written treasures
Indian and France have signed a Memorandum of
Understanding (MoU) to conserve, develop and
publicize written heritage.
Details:
The MoU will assist a programme on
digitization of old manuscripts and
documents, begun in France seven years
ago, as it will make it possible for gaining
their experience and high quality expertise
in this field.
This MoU will also entail development of
digital co-operation, sharing of technical
and other expertise, competency and skillbuilding and cultural co-operation between
the two countries.
France is keen to avail Indias assistance in
sorting out, deciphering and gaining a better
insight into their collection of the several
thousands of Indian documents, especially
in Sanskrit and Tamil languages,that are
archived in their possession.
Ministry of Culture is also planning to build a
National Virtual Library in India which will store
and share numerous manuscripts, archives,
artworks etc. This MoU will help in efficient
implementation of this project. The National
Virtual Library will link and share all knowledge
resources lying within the possession of various
government institutes and other organizations of
both the countries.
sources: pib.
Trivia:

Nepal gets first woman President


Nepals Parliament has elected communist
lawmaker
Bidhya
Bhandari as
the countrys
first woman
President after
the adoption of
a landmark
Constitution
last month.

Current Affairs 122 October-2015

1. Question

Which of the following statements about the


India Development Foundation of Overseas
Indians (IDF-OI) is/are incorrect?
1. It is a not-for-profit trust set up by the
Ministry of Overseas Indian Affairs
2. It engages with the concerned authorities
of other nations in enhancing the skills
of blue-collared Overseas Indians
Select the correct answer using the codes given
below:
a) 1 Only
b) 2 Only
c) Both 1 and 2
d) Neither 1 or 2
Ans: b.
India Development Foundation of Overseas
Indians (IDF-OI) is a not-for-profit trust registered
by the Ministry of Overseas Indian Affairs,
Government of India, to provide a credible window
for Overseas Indian Philanthropy in Indias Social
Development. The objective of the Foundation is to
facilitate philanthropic activities by Overseas
Indians through innovative projects and
instruments such as micro credit for rural
entrepreneurs, self help groups for economic
empowerment of women, best practice
interventions in primary education and technology
interventions in rural health care delivery.
http://pib.nic.in/newsite/PrintRelease.aspx?
relid=128679

2. Question

H1N1 virus is sometimes mentioned in the news


with reference to which one of the following
diseases?
a) AIDS
b) Bird Flu
c) Dengue
d) Swine Flu
Ans: d.
http://pib.nic.in/newsite/PrintRelease.aspx?
relid=128677

3. Question

With the presence of more than 4,200 startups,


India has become the third largest startup base
worldwide. What may be the reasons for the
growth of the entrepreneur ecosystem in India?
1. Investment in start-ups by business
tycoons
2. The Start-Up India initiative has seen
rapid success
3. A sharp increase in the availability of

risk capital
Select the correct answer using the codes given
below:
a) 2 and 3 Only
b) 1 and 2 Only
c) 1 and 3 Only
d) All of the above
Ans: c.
The Start-Up India initiative was announced by
PM Modi in August this year. By August 2016, it
may become a rapid-success story!
http://www.thehindu.com/business/Industry/indiabags-third-slot-in-global-startupspace/article7758031.ece

4. Question

Which of the following statements about writs


mentioned in the Indian Constitution is/are
incorrect?
1. Mandamus, unlike Prohibition,
directs inactivity
2. Certiorari is both preventive as well as
curative, unlike prohibition, which is
only curative
3. Mandamus can also be issued against
the Government asking it to perform
duties which it has failed or refused to
perform
Select the correct answer using codes given
below:
a) 1 and 3 only
b) 1 and 2 only
c) 2 and 3 only
d) 3 Only
Ans: b.
Statements 1 and 3 are contradictory. Hence, if you
had read this article, you would have realised that
statement 3 is correct. You would be left only with
option a and b. The correct answer between
these two options though, you can hazard a guess if
you have read Laxmikanth/D.D. Basu.
Page 7.18-19, Indian Polity by M. Laxmikanth

5. Question

With reference to the Pradhan Mantri Khanij


Kshetra Kalyan Yojana (PMKKKY), which of
the following statements is/are correct?
1. It is being administered by the Ministry
of Labour and Employment
2. This scheme will be binding on state
governments
Select the correct answer using the codes given
below:
a) 1 only
b) 2 Only

Current Affairs 123 October-2015

c) Both 1 and 2
d) None of the above
Ans: b.
Pradhan Mantri Khanij Kshetra Kalyan Yojana,
13th October
http://pib.nic.in/newsite/efeatures.aspx

6. Question

Consider the following statements:


Assertion (A): The Pradhan Mantri Khanij
Kshetra Kalyan Yojana (PMKKKY)
earmarks 50% of expenditure of the District
Mineral Foundation (DMF) funds for high
priority areas, 30% towards other priority
areas and 20% towards low priority areas
Reason (R): PMKKKY provides guidelines for
utilization of funds accruing to DMF
Select the correct answer using codes given
below:
a) A and R both are true, and R is the
correct explanation for A
b) A and R both are true, and R is the
NOT the correct explanation for A
c) A is correct, R is incorrect
d) A is incorrect, R is correct
Ans: d.
The scheme earmarks 60% of expenditure of the
DMF fund for high priority areas, and 40%
towards other priority areas.
Pradhan Mantri Khanij Kshetra Kalyan Yojana,
13th October
http://pib.nic.in/newsite/efeatures.aspx

7. Question

Consider the following statements about


Strategic Partnerships in the context of
international relations:
1. Its appearance can be traced to the end
of the cold war
2. These partnerships entail only the
following areas of co-operation:
political-diplomatic ties, defence ties,
and economic relations
Which of the above statements is/are incorrect?
a) 1 Only
b) 2 Only
c) Both 1 and 2
d) Neither 1 or 2
Ans: c
Strategic partnerships are commonly associated
with defence or security related issues, but a survey
of formal strategic partnerships around the world
reveal they can also be quite a hold-all, covering a
wide range in bilateral relations, from defence to

education, health and agriculture, and quite


commonly, economic relations, including trade,
investment and banking.
Some scholars of international relations theory
have argued against a set definition, arguing that
each agreement belongs to a specific time and
context, and thus has its own meaning. Some have
even argued that the phrase is nothing more than
nomenclature, and parties use it to project a higher
status to their ties
http://www.businessstandard.com/article/economy-policy/indiasingapore-to-sign-strategic-partnership-pactduring-pms-visit-115101300787_1.html

http://www.thehindu.com/opinion/op-ed/in-thepromiscuous-world-of-international-relationselements-of-a-strategicpartnership/article2806158.ece

8. Question

Consider the following statements about the


India-Africa Forum Summit:
1. The first such summit will be held in
New Delhi in 2015
2. This biennial summit is to be held on a
rotational basis between India and Africa
Which of the above statements is/are correct?
a) 1 Only
b) 2 Only
c) Both 1 and 2
d) Neither 1 or 2
Ans: d.
It is not a biennial summit.
http://www.thehindu.com/todays-paper/tpopinion/reversing-the-continentaldrift/article7758734.ece

9. Question

The Government recently announced that the


Price Stabilisation Fund (PSF) of the Centre
will be used to cool prices of an agricommodity. Which of the following statements
about the PSF is/are incorrect?
1. It will support market interventions
for price control of onions and potatoes
initially; other commodities may be
added later
2. States have to set up a revolving fund to
receive grants under this scheme
Select the correct answer using the codes given
below:
a) 1 Only
b) 2 Only
c) Both 1 and 2

Current Affairs 124 October-2015

d) Neither 1 or 2
Ans: b.
States have to set up a revolving fund to receive
interest-free loans under this scheme. A revolving
fund is a fund that is continually replenished as
withdrawals are made.
http://www.thehindu.com/business/Economy/gover
nment-announces-buffer-stocks-forpulses/article7762552.ece

http://pib.nic.in/newsite/PrintRelease.aspx?
relid=117762

10. Question

In the Wholesale Price Index, which one of


the following is accorded the highest weight?
a) Food articles
b) Fuel and Energy
c) Manufactured products
d) Primary articles
Ans: c.
http://www.thehindu.com/business/Economy/wpicontraction-eases-to-45/article7762270.ece

11. Question

The Global Hunger Index is calculated and


released by the
a) World Bank
b) International Food Policy Research
Institute
c) Food and Agriculture Organisation
d) United Nations Childrens Fund
Ans: b.
https://www.ifpri.org/topic/global-hunger-index

12. Question

To reflect the multidimensional nature of


hunger, the Global Hunger Index (GHI)
combines three equally-weighted indicators in
one index number:
1. Proportion of undernourished
2. Proportion of under-5 underweight
children
3. Mortality rate of under-5 children
4. Adult female literacy rate
5. Proportion of population living below $1
in Purchasing-Power-Parity terms per
day
Which, from the above list, are the indicators?
a) 1, 2 and 3 only
b) 1, 2 and 4 only
c) 1, 4 and 5 only
d) 3, 4 and 5 only
Ans: a.
https://www.ifpri.org/topic/global-hunger-index

Insights into Editorial: NJAC An AntiConstitutional Judgment?


By striking down the 99th constitutional amendment, the Supreme Court of India has clearly indicated that
NJAC (National Judicial Appointments Commission) is not a credible alternative to the Supreme Courts
collegium.
However, some experts are not satisfied with the Courts judgment. Why?
They are upset by the fact that the verdict upholds an extra-constitutional forum, created by the
Supreme Courts own members to serve its own ends, in the place of a system lawfully enacted by a
popularly elected Parliament.
According to them, the judgment fails to adequately answer the fundamental question at the root of
the controversy: how is judicial primacy in making appointments to the higher judiciary a part of our
Constitutions basic structure?
How proponents of the NJAC view the collegium?
According to them, the supreme court, in the second judges case, 1993, wrongly interpreted the word
consultation used in Articles 124 and 217, to mean concurrence. The court then held that the executive was
bound by the advice of the CJI in making appointments to the higher judiciary.
Current Affairs 125 October-2015

And hence, it is being alleged that the collegium is a pure creation of the supreme court through second
judges case.
What the constitution says?
The constitution accords to the President the power to appoint judges to the Supreme Court and to the
various High Courts. In performing this function, the executive is required to compulsorily consult
with certain persons.
To make appointments to the Supreme Court, the Chief Justice of India (CJI) must always be
consulted. In elevating persons to a High Courts bench, in addition to the CJI, the Chief Justice of
that High Court and the Governor of the State concerned, acting through his or her Council of
Ministers, must be mandatorily conferred with.
How proponents of NJAC defend it?
According to them the enactment of the 99th Amendment was intended at redressing the imbalance
created by the verdict of court in second judges case.
For them, NJAC would have been a more broad-minded forum, providing a genuine chance to
participate and influence the selection of our higher judiciary not merely to the Supreme Court and
the executive, but also to laypersons (eminent persons) outside the constitutional framework.
Why the court struck down NJAC act?
The court has held that the appointment of judges, coupled with primacy of judiciary and the CJI, was part of
the basic structure of the Constitution and that the parliament, through NJAC act, violated this basic
structure.
Conclusion:
There is an argument that the Constitution envisages and puts a system in place to ensure the balance of
power involving the executive, the legislature and the judiciary. The judiciary has the power and jurisdiction
to review the functioning of the executive and the legislature and thus, it is supremely important that the
appointment of judges remains above board. With evident loopholes in the collegium system, it appears that
a review of the verdict by a larger bench is necessary.

30 October 2015
Paper 3 Topic: S & T.

First Scorpene class submarine set afloat


Kalavari, the first of Scorpene class submarines
being manufactured at Mazagon Dock Shipbuilders
Ltd (MDL), was recently set afloat in the Mumbai
naval dockyard.
The submarine will now undergo rigorous
harbour trials and tests which will certify
each system to its fullest capacity.
Kalvari is first of the Indian Navys Scorpene class
stealth submarines being built under the Project 75,
under collaboration with M/s DCNS, France.
Scorpene-class submarine:
The Scorpene class submarines are a class
of diesel-electric attack submarine jointly
being developed by the French DCN and
the Spanish company Navantia and now by
DCNS.
It features diesel propulsion and an
additional air-independent propulsion (AIP)
system.
They are 67 meters long, 6.2 meters wide
and have 1,550 tonnes displacement.

The state-of-art features of the Scorpene include


superior stealth and the ability to launch an attack
on the enemy using precision-guided weapons. The
attack can be launched with torpedoes, as well as
tube launched anti-ship missiles, underwater or on
surface.
Designed to operate in all theatres including
the tropics, the submarines can undertake
multifarious missions including anti-surface
warfare, anti-submarine warfare,
intelligence gathering, operations by special
forces and mine laying etc.
What is Air-independent propulsion?
Air-independent propulsion (AIP) is any
technology which allows a non-nuclear
submarine to operate without the need to
access atmospheric oxygen (by surfacing or
using a snorkel).

Current Affairs 126 October-2015

It can augment or replace the diesel-electric


propulsion system of non-nuclear vessels.
It is based on the combustion of stored
oxygen and ethanol to augment batterypowered propulsion.
Significance of AIP:
Notably, a submarine is about stealth. It is a
weapons platform not visible to the naked
eye. AIP significantly improves stealth
because it enables a submarine to generate
electricity for services and battery charging
and propulsion while completely
submerged.
AIP systems also generate electricity,
powering a submarines to operate and also
generate oxygen, lighting and amenities for
crew.
They enable conventional diesel-electric
submarines to remain submerged for two to
three weeks at a time. This significantly
increases the risk of detection.
Another advantage is that the Non-nuclear
submarines running on battery power or
AIP can be virtually silent.
sources: the hindu, pib.
Paper 3 Topic: Money laundering and
its prevention.

Meet to devise new strategy to combat


money laundering
The meeting of all International groups, combating
money laundering rackets funding terror activities
and organised crimes, will be held in the first week
of November in New Delhi.
This meeting is aimed at building greater
inter-regional coordination in law
enforcement and intelligence-sharing.
The meeting is part of Networking the
Networks, an initiative of the United
Nations Office on Drugs and Crime
(UNODC). It is being held in coordination
with the Indian government.
The meeting will also address a wide range
of issues pertaining to trans-border illicit
financial transactions by organised criminal
syndicates and terror outfits.
Participants:
Among the key participants are
representatives of top international groups,
including Interpol, the Egmont Group of
Financial Intelligence Units, Financial
Action Task Force, the Eurasian Group on

combating money laundering and financing


of terrorism and the Asia/Pacific Group on
Money Laundering.
The Directorate of Revenue Intelligence
and the Enforcement Directorate will also
attend the meeting.
Focus areas of the meeting:
The meeting will focus on establishing
close cooperation among these international
groups to build a global network for an
effective fight against money laundering.
Issues related to sharing of resources of all
such anti-money laundering groups which
will ensure expeditious action against transborder crimes, will also be discussed.
The participants at the meeting are expected
to highlight the best practices being
followed by these groups and law
enforcement agencies, to be followed by
member countries for more coordinated
action.
Procedures to facilitate confiscation of the
proceeds of crime in foreign jurisdictions
a major issue facing India will also be
discussed.
The members would also make
recommendations to strengthen the South
Asia Regional Information and
Coordination Center for more concerted
action in the region.
About UNODC:
Established in 1997 through a merger between the
United Nations Drug Control Programme and the
Centre for International Crime Prevention,
UNODC is a global leader in the fight against
illicit drugs and international crime.
UNODC operates in all regions of the
world through an extensive network of field
offices.
UNODC relies on voluntary contributions,
mainly from Governments, for 90% of its
budget.
UNODC is mandated to assist Member
States in their struggle against illicit drugs,
crime and terrorism.
In the Millennium Declaration, Member
States have resolved to intensify efforts to
fight transnational crime in all its
dimensions, to redouble the efforts to
implement the commitment to counter the
world drug problem and to take concerted
action against international terrorism.

Current Affairs 127 October-2015

The three pillars of the UNODC work programme


are:
1. Field-based technical cooperation projects
to enhance the capacity of Member States
to counteract illicit drugs, crime and
terrorism.
2. Research and analytical work to increase
knowledge and understanding of drugs and
crime issues and expand the evidence base
for policy and operational decisions.
3. Normative work to assist States in the
ratification and implementation of the
relevant international treaties, the
development of domestic legislation on
drugs, crime and terrorism, and the
provision of secretariat and substantive
services to the treaty-based and governing
bodies.
sources: the hindu, unodc.
Paper 1 Topic: Population and
associated issues.

As workforce ages, China ends its onechild policy


China has dropped its controversial one-child
policy, allowing all couples to have two children
for the first time in more than three decades.
The policy change is an advance over the
2013 ruling, which allowed couples, where
one of the parents was a single child, to
have two children.
Why such move?
It was due to the mounting pressure on the
authorities to ease the family-size norms after it
became evident that the one-child policy was
leading to severe labour shortages and an ageing
population. One child policy was meant to restrain
a burgeoning population in the 1970s.
What the reports indicate?
According to U.N. estimates, nearly 440
million people in China would be over 60
by 2050, signalling a sharp decline in the
labour pool.
Last year, the working population between
the ages 15 and 59 slid by 3.71 million.
The Chinese Academy of Social Sciences
recently said that China should permit all
couples to have a second child because the
total fertility rate had dropped to 1.4,
signifying the average number of children
to be born of a woman in her entire
lifespan. This was well short of the 2.1mark that should be reached to ensure a

balanced population profile.


sources: the hindu.
Paper 3 Topic: space.

Chinas first moon rover sets record for


longest stay
Chinas first lunar rover, Yutu, has been operating
on the moon for almost two years, setting a record
for the longest stay by a rover.
About Yutu:
Yutu was deployed and landed on the moon
via Chinas Change-3 lunar probe in 2013,
staying longer than the Soviet Unions 1970
moon rover Lunokhod 1, which spent 11
months on the moon.
The scientific objectives of Change-3
mainly included lunar surface topography
and geological survey, lunar surface
material composition and resource survey,
Sun-Earth-Moon space environment
detection, and lunar-based astronomical
observation.

Brief history of Chinas space programme:


The launch of Dongfanghong-1, Chinas
first satellite, in 1970 made China the fifth
country to launch a domestic satellite using
a domestic rocket, following the Soviet
Union, the US, France and Japan.
China launched its manned space
programme in the 1990s and successfully
sent Yang Liwei, the countrys first
astronaut, into orbit on the Shenzhou-5
spacecraft in 2003.
China is also planning to be the first
country to land a lunar probe on the far side
of the moon, or dark side of the moon
which is never visible to Earth. The mission
will be carried out by Change-4, a backup
probe for Change-3.
sources: the hinduu.

Current Affairs 128 October-2015

Paper 3 Topic: Science and


Technology- developments and their
applications and effects in everyday
life.

Google to help Indonesia expand Net


connectivity
Alphabet, parent company of tech giant Google,
has announced tie-ups with three major Indonesian
telecom operators to expand web connectivity in
the countrys remote areas using hundreds of netbeaming balloons.
The initiative is part of Project Loon by
Google X, Alphabets research division,
which works on ambitious ideas, including
self-driven cars.
The initiative has already been tried out in
Brazil, New Zealand and Australia with a
single service provider and has been
successful.
According to the Google, around 1,000
Internet balloons have already been
deployed worldwide, and have flown nearly
20 million km while some of them had
circled the world 20 times.
Indonesia is the worlds fourth most populous
country with more than 255 million people, twothirds of whom have no access to the web.
About Project Loon:
Project Loon is a research and development
project being developed by Google X with
the mission of providing Internet access to
rural and remote areas.
The project uses high-altitude balloons
placed in the stratosphere at an altitude of
about 32 km to create an aerial wireless
network with up to 3G-like speeds.
How it operates?
The balloons are maneuvered by adjusting
their altitude to float to a wind layer after
identifying the wind layer with the desired
speed and direction using wind data from
the National Oceanic and Atmospheric
Administration (NOAA).
Users of the service connect to the balloon
network using a special Internet antenna
attached to their building.
The signal travels through the balloon
network from balloon to balloon, then to a
ground-based station connected to an
Internet service provider (ISP), then onto
the global Internet.
Why stratosphere was chosen?

Google asserts that the stratosphere is


advantageous because of its relatively low wind
speeds and minimal turbulence. Google also claims
that it can model, with reasonable accuracy, the
seasonal, longitudinal, and latitudinal variations in
wind speeds within the 1825 km stratospheric
layer.
Significance of this project:
The technology designed in the project
could allow countries to avoid using
expensive fiber cable that would have to be
installed underground to allow users to
connect to the Internet.
This will also greatly increase Internet
usage in developing countries in regions
such as Africa and Southeast Asia that cant
afford to lay underground fiber cable.
The project also brings Internet access to
remote and rural areas poorly served by
existing provisions, and improves
communication during natural disasters to
affected regions.
sources: the hindu, wiki.
Paper 3 Topic: environmental
pollution.

Punjab to set up bio-ethanol refinery


The Punjab State government has signed a
Memorandum of Understanding (MoU) for setting
up a bio-ethanol refinery with a consortium of Beta
Renewables, Novozymes and CVC India
Infrastructure Pvt. Ltd.
This is being seen as a major step towards
tackling the menace of wheat and paddy
straw burning in Punjab.
Estimated project cost is Rs. 950 crore.
How these refineries would help?
The bio-refinery will solve the chronic problem of
straw burning and also help the farmers who would
be paid for the agro-based produce. It would also
help in containing the loss of fertility and damage
to environment.
What is Bioethanol?
Bioethanol is a form of quasi-renewable energy
that can be produced from agricultural feedstocks.
It can be made from very common crops such as
sugarcane, potato, cassava and corn. It is also made
from corn, potatoes, milk, rice, beetroot and
recently grapes, banana and dates depending on the
countries agricultural strength.
Important uses:
It is blended with petrol to make a truly
sustainable transport fuel.

Current Affairs 129 October-2015

It is used in cosmetic and other


manufacturing processes.
sources: the hindu, wiki.
Paper 2 Topic: UN organs.

Pakistan loses UNHRC seat


Pakistan has failed to win a re-election to the top
UN human rights body, UNHRC. It garnered just
105 votes in the 193-member General Assembly.
A total of 18 members were elected to the
UN Human Rights Council through a secret
ballot.
Pakistans current term is set to expire on
December 31 and it was seeking re-election
to the 47-member Council.
Pakistan lost the seat in the Asia-Pacific
category in which five seats were vacant.
India is also a member of the Council and
its term will end in 2017.
The new members, who will start their three-year
terms from January 1 next year, are Belgium,
Burundi, CdIvoire, Ecuador, Ethiopia, Georgia,
Germany, Kenya, Panama, Kyrgyzstan, Mongolia,
Philippines, Republic of Korea, Togo, Slovenia,
Switzerland, UAE and Venezuela.
UNHRC:
It is an inter-governmental body within the United
Nations system responsible for strengthening the
promotion and protection of human rights around
the globe and for addressing situations of human
rights violations and make recommendations on
them.
It meets at the UN Office at Geneva.
The Council is made up of 47 United
Nations Member States which are elected
by the UN General Assembly.
The term of each seat is three years, and no
member may occupy a seat for more than
two consecutive terms.
The council works closely with the Office
of the High Commissioner for Human
Rights and engages the United Nations
special procedures.
The General Assembly can suspend the
rights and privileges of any Council
member that it decides has persistently
committed gross and systematic violations
of human rights during its term of
membership. The suspension process
requires a two-thirds majority vote by the
General Assembly.
sources: the hindu, unhrc.

Paper 2 Topic: Welfare schemes for


vulnerable sections of the population
by the Centre and States and the
performance of these schemes.

New initiative to spread awareness about


Beti Bachao Beti Padhao Scheme
The Union Ministry of Women and Child
Development has launched a new initiative to
spread awareness about the Beti Bachao Beti
Padhao Scheme of the Ministry.
About the initiative:
The initiative has been launched in
partnership with Celltick Mobile Media
India Pvt. Ltd. to generate awareness about
BBBP Scheme to an estimated 100 million
mobile users in India.
Through the Celltick managed mobile
services based on its patented Live Screen
platform, available to over 100 million
users in India, the user will be able to get
messages that they can interact with on the
homescreen of their device, enabling ease
of use in accessing information about
BBBP.
The platform provides a means to reach
users based on their location as well as their
language of preference.
The service will direct users to access more
information about it.
About BBBP:
This campaign is aimed at promoting
gender equality and the significance of
educating girls.
The Ministry of Women and Child
Development is the nodal ministry for the
Scheme.
The Scheme is targeted at improving the
Child Sex Ratio through multi sectoral
interventions including prevention of
gender biased sex selection and promoting
girls education and her holistic
empowerment.
sources: pib.
1. Question

Which of the following sections of the


Information Technology Act grants Internet
Service Providers (or intermediaries) immunity
from liability, in certain cases, for the content
they host?
a) Section 66A
b) Section 79

Current Affairs 130 October-2015

c) Section 69A
d) There is no such immunity
Ans: b.
Most countries have a system that allows them to
ask intermediaries to block or take down specified
illegal content. In India, the Indian IT Act has
created such a system (included in Section 79),
enabling the government or any third party to ask
for the blocking of content and set up a process for
content removal by intermediaries.
While in case of an emergency, the State police
directly write to the intermediary concerned, such
as Facebook and Twitter, to block a website under
Section 79 (3)(b) of the Information Technology
Act. The other way of blocking objectionable
content is through a committee comprising CERTin officials, which takes a call after wider
consultation.
http://www.thehindu.com/news/national/centralgovernment-blocks-two-website-of-islamicstate/article7762786.ece

http://indiankanoon.org/doc/844026/

2. Question

Consider the following statements:


1. An exclusive rail-and-road-way will be
open at Kolkata and pass through
Bangladesh to end at Kaladan in Tripura
2. It will provide an alternate route of
goods movement between the Northeast
Indian states and mainland India
3. It will reduce pressure on the Siliguri
Corridor
Which of the above statements about the
Kaladan Multi Modal Transit Transport
Project are correct?
a) 1 and 2 only
b) 2 and 3 only
c) 1 and 3 only
d) All are incorrect
Ans: b.
http://pib.nic.in/newsite/PrintRelease.aspx?
relid=128699

3. Question

The White Cane Day also known


internationally as the White Cane Safety Day
was organized by the Ministry of Social Justice
& Empowerment recently to sensitize the
common man about the challenges being faced
by
a) Visually challenged people
b) Aged persons
c) The physically challenged

d) Both a and b
Ans: a.
http://pib.nic.in/newsite/PrintRelease.aspx?
relid=128726

4. Question

Programme for International Student


Assessment (PISA) is a survey conducted by
a) Pratham (An Indian NGO)
b) Organisation for Economic
Cooperation and Development
c) Global Partnership for Education
d) World Bank
Ans: b.
. However, the overall standard of education is
well below global standards: that PISA 2009+
results ranked Tamil Nadu and Himachal Pradesh
72 and 73 out of 74 participants, higher only than
Kyrgyzstan, exposes the gaps in our education
system. PISA, which measures the knowledge and
skills of 15-year-olds with questions designed to
assess their problem-solving capabilities, rates
these two states at the bottom, with the scores in
mathematics and science falling way behind the
OECD (Organisation for Economic Cooperation
and Development) average
The most recently published PISA results are from
the assessment in 2012. India did not participate in
PISA 2012.
http://www.thehindu.com/opinion/lead/oecd-studythe-perils-of-efixation/article7762198.ece

5. Question

With reference to the relationship between the


State of Jammu and Kashmir and India, which
of the following statements is/are correct?
1. An amendment made to the Indian
Constitution does not apply to the state
unless it is extended by a presidential
order
2. Part IV and Part IV-A of the Indian
Constitution are applicable to the state
with some exceptions and conditions
3. While Article 1 of the Indian
Constitution is applicable to the state,
Article 3 is not
Select the correct answer using the codes given
below:
a) 1 only
b) 1 and 2 only
c) 1 and 3 only
d) All of the above
Ans: c.

Current Affairs 131 October-2015

Part III of the Indian Constitution is applicable to


the state with some exceptions and conditions. Part
IV and IV-A are not applicable to the state.

http://www.thehindu.com/opinion/editorial/theimportance-of-article-370/article7762192.ece
Chapter 32, Indian Polity by M Laxmikanth

Insights into Editorial: Reaching out to Africa


31 October 2015
The 3rd India-Africa Forum Summit, which concluded recently in New Delhi, was the largest gathering of
foreign dignitaries in New Delhi since the 1983 Non-Aligned Summit. 41 heads of state and government
from 54 countries in Africa were present at the summit. This summit has taken the relationship between
India and Africa to a higher level with a demonstrated resolve and a clearly laid-down road map. During this
summit, India offered a new line of credit worth $10 billion to Africa to strengthen economic cooperation
and has also called for a unified stance for the reform of the UN Security Council.
The India-Africa Forum Summit serves as the official platform for the African-Indian relations. It will be
held once in every three years and was first held in 2008.
Why Africa is important for India?
A rapidly growing India not only needs more commodities from Africa, but also needs its vast market
to pay for them. Africa is an important trade partner for India. India-Africa trade was worth almost
$70 billion in 2014-15, and Indian companies invested some $30-35 billion in the continent over the
past decade.
Indian energy companies have assets in African countries, and India exports consumer and capital
goods and medicines to Africa.
Energy imports from Africa have increased dramatically from mid-2000s and Africa now accounts
for about 18% of Indias oil imports. Nigeria is Indias largest source of oil in Africa followed by
Angola.
Apart from oil, Africa is also an important source of coal, natural gas and uranium.
Bilateral trade in agricultural goods has also grown rapidly. Agricultural goods currently account for
about 11% of Indias total exports to sub-Saharan Africa and about 7% of Indias total imports from
sub-Saharan Africa.
Beef, sugar and fish are other important agricultural exports from India to Africa. About 15% of
Indias beef exports are destined to Africa.
Africa has also emerged as an important source of cash crops such as shelled cashew, vegetables, nuts
(fresh and dried), coffee, tea, and spices for India.
The present pace of growth of Africa will ensure that most African countries will be middle income by 2025.
The projected GDP of Africa in 2050 is $29 trillion, placing it in the same range as Indias projected 2050
GDP.
Indias main concerns:
While trade between the two countries has improved in the last decade, it is still much less than
Africas trade with China, which was $200 billion in 2014-15.
Indias investments in Africa are less compared to China. China has invested more than $180 billion
in Sub-Saharan Africa alone in areas ranging from energy to infrastructure during the period 20052015.
Conclusion:
From the above points it is evident that deeper engagement with Africa is imperative for India. Economic
relations between India and Africa have intensified in the last decade but there is an urgent need to consider
the role of other big players such as China and Brazil in Africa, particularly in the energy and agriculture
sector. Indias approach towards Africa needs to be much more structured, development cooperation in
Africa must be closely linked to Indias economic interests, and financial packages to Africa must be more
generous.

Current Affairs 132 October-2015

Paper 3 Topic: conservation.

31 October 2015
Paper 2 Topic: Bilateral, regional and
global groupings and agreements
involving India and/or affecting Indias
interests.

Nuclear suppliers may admit India


The chairperson of Nuclear Suppliers Group
(NSG) Rafael Grossi, during his recent visit to
India, said that the NSG would begin consultations
on Indias membership next month. He thinks it is
possible to find a way for Indias membership to be
accepted.
India sought membership of the NSG in
2008, but its application hasnt been
decided on, primarily because signing the
NPT or other nuclear moratoriums on
testing is a pre-requisite.
India has received a special waiver to
conduct nuclear trade with all nuclear
exporters.
Indias push for the NSG membership in the
next year is likely to be complemented by
its application to other nuclear and missile
control regimes like MTCR, Wassenaar and
Australian groups.
It will also get a boost from the completion
of the India-Australian civil nuclear
agreement likely to be announced in the
next few months, with the possibility of the
India-Japan civil nuclear deal also making
progress.
About NSG:
Nuclear Suppliers Group (NSG) is a multinational
body concerned with reducing nuclear proliferation
by controlling the export and re-transfer of
materials that may be applicable to nuclear weapon
development and by improving safeguards and
protection on existing materials.
Interestingly, the NSG was set up in 1974 as
a reaction to Indias nuclear tests to stop
what it called the misuse of nuclear material
meant for peaceful purposes.
Currently, it has 48 members.
Once admitted, an NSG member state gets:
Timely information on nuclear matters
Contributes by way of information
Has confirmed credentials
Can act as an instrument of harmonization
and coordination
Is part of a very transparent process.
sources: the hindu, wiki.

Sunderbans to get a student army of


conservationists
An ambitious project has been started in West
Bengal under which Schoolchildren in the
Sunderbans area will learn about tiger conservation
and pass on the experience to their elders.
Details:

Under this project, two fully equipped


edutainment boats carrying a projector, a
sound system, generators, a library, films
related to conservation and wildlife
photographs will be launched in the
Sunderbans which will help students in
understanding the importance of this area.
The students will go back home and impart
what they have learnt to their parents and
elders, thus making it easier to spread
awareness among people living in the fringe
areas of the forests.
Sundarbans:
The Sundarbans is a natural region in West
Bengal and Bangladesh. It is the largest
single block of tidal halophytic mangrove
forest in the world.
The Sundarbans covers approximately
10,000 square kilometres (3,900 sq mi) of
which 60% is in Bangladesh with the
remainder in India.
It is a UNESCO World Heritage Site.
Mangroves in India:
Mangroves in India account for about 3%
of the worlds mangrove vegetation.
Mangrove cover in India is 4,662 sq. km,
which is 0.14% of the countrys total
geographical area.
Sundarbans in West Bengal accounts for
almost half of the total area under
mangroves in the country. Mangrove in

Current Affairs 133 October-2015

India is famous for its rich variety of flora


and fauna.
Composition of Mangroves in India: The very
dense mangrove comprises 1,403 sq. km (30.10%
of the total mangrove cover), moderately dense
mangrove is 1,658.12 sq. km (35.57 %) while open
mangroves cover an area of 1,600.44 sq. km
(33%).
sources: the hindu, pib.
Paper 2 Topic: Important aspects of
governance, transparency and
accountability.

Give NOTA option in civic polls: Gujarat


HC
The Gujarat High Court has criticised the State
Election Commission (SEC) for its reluctance to
implement the NOTA (none of the above) option in
the coming municipal and panchayat elections in
the State.
The court has directed the SEC to
implement it and the State government to
give the SEC necessary help.
The court said the SEC must provide the
option as directed by the Supreme Court,
rejecting the SECs contention that it was
not possible to include the NOTA button in
the electronic voting machines in 20 days
because the polls are scheduled for
November 22 and 29.
The Court also observed that the right to
vote in favour of none of the candidates is
one of the rights of the electorate, and it is
so identified and strengthened by the
Supreme Court.
NOTA:
NOTA is designed to allow the voter to indicate
disapproval of all of the candidates in a voting
system. It is based on the principle that consent
requires the ability to withhold consent in an
election, just as they can by voting no on ballot
questions.
The Supreme Court had upheld the right of
voters to reject all candidates contesting the
elections, saying it would go a long way in
cleansing the political system of the
country.
The court had directed the Election
Commission to have an option of None Of
The Above (NOTA) on the electronic
voting machines (EVMs) and ballot papers
in a major electoral reform.
The EVMs have the NOTA option at the

end of the candidates list. Earlier, in order


to cast a negative ballot, a voter had to
inform the presiding officer at the polling
booth. A NOTA vote doesnt require the
involvement of the presiding officer.
In the Indian general election, 2014, NOTA
polled 1.1% of the votes, counting to over 6
million.
Before NOTA:
Before the NOTA option came in existence,
people casting negative votes were required
to enter their names in a register and cast
their vote on a separate paper ballot.
Under Section 49 (O) of the Conduct of
Elections Rules, 1961, a voter could enter
his electoral serial number in Form 17A and
cast a negative vote. The presiding officer
would then put a remark in the form and get
it signed by the voter. This was done to
prevent fraud or misuse of votes.
This provision was, however, deemed
unconstitutional by the SC as it did not
protect the identity of the voter.
sources: the hindu, pib, wiki.
Paper 3 Topic: Biodiversity.

More Indian birds enter list of


threatened species
The Red List of birds released by the International
Union for Conservation of Nature (IUCN) for 2015
shows that a total of 180 bird species in India are
now threatened, as against 173 last year. Only one
species in the list has moved out of the Red List.
Details:
Of the new additions, five have been
uplisted from the Least Concerned to the
Near Threatened category, a sign of
increased threat. These include Northern
Lapwing (a grassland bird) and four
wetland birds, namely Red Knot, Curlew
Sandpiper, Eurasian Oystercatcher and BarTailed Godwit.
Two other wetland birds, Horned Grebe and
Common Pochard have been uplisted from
Least Concerned to Vulnerable.
Steppe Eagle (a raptor from grasslands),
which is a regular winter visitor to the
Indian subcontinent, has been uplisted from
Least Concerned to Endangered.
Destruction of grasslands, wetlands, forests
and other habitats is considered the most
common reason for this development.
Apart from habitat loss, the other reason for

Current Affairs 134 October-2015

decline of birds like Steppe Eagle, which


mostly scavenges on animal carcasses, is
the use of veterinary drug diclofenac used
to treat livestock. This drug causes renal
failure in these birds.
Three vulture species, namely Whitebacked, Slender-billed and Long-billed
have also been severely affected by
diclofenac.
IUCN:
IUCN was founded in October 1948 as the
International Union for the Protection of
Nature (or IUPN) following an international
conference in Fontainebleau, France.
It was renamed as International Union for
Conservation of Nature and Natural
Resources in 1956 with the acronym IUCN.
It is the worlds first global environmental
organization. Today it is the largest
professional global conservation network
The Unions HQ is located in Gland, near
Geneva, in Switzerland.
It demonstrates how biodiversity is
fundamental to addressing some of the
worlds greatest challenges such as climate
change, sustainable development and food
security.
Red list:
The IUCN Red List is set upon precise
criteria to evaluate the extinction risk of
thousands of species and subspecies. These
criteria are relevant to all species and all
regions of the world.
The aim is to convey the urgency of
conservation issues to the public and policy
makers, as well as help the international
community to try to reduce species
extinction.
Species are classified by the IUCN Red List
into nine groups, set through criteria such as
rate of decline, population size, area of
geographic distribution, and degree of
population and distribution fragmentation.
sources: the hindu, iucn.
1. Question

Through the Universal Immunisation


Programme, India has achieved the elimination
of which of the following diseases?
1. Polio
2. Measles
3. Small Pox
4. Maternal and Neonatal Tetanus
Select the correct answer using the codes given

below:
a) 1 and 2 only
b) 1, 2 and 3 only
c) 1 and 3 only
d) 1, 3 and 4 only
Ans: d.
Unlike polio and smallpox, tetanus cannot be
eradicated (tetanus spores are present in the
environment worldwide), but through
immunization of pregnant women and other
women of reproductive age and promotion of more
hygienic deliveries and cord care practices, MNT
(Maternal and Neonatal Tetanus) can be eliminated
(defined as less than one case of neonatal tetanus
per 1000 live births in every district)
http://indianexpress.com/article/opinion/columns/le
ts-leave-no-child-behind/

2. Question

Dr. Abdul Kalams birth place (Rameswaram)


was recently included in the Atal Mission for
Rejuvenation and Urban Transformatin
(AMRUT) mission for providing basic urban
infrastructure. Which among the following
is/are the thrust areas of the AMRUT mission?
1. Pedestrian facilities
2. Upgrading green spaces and parks
3. Storm-water drains
4. Parking spaces
Select the correct answer using the codes given
below:
a) 1 Only
b) 1 and 2 only
c) 1, 2 and 3 only
d) 1, 2, 3 and 4
Ans: d.
http://amrut.gov.in/writereaddata/Thrust
%20Areas.pdf

3. Question

The International Tourism Mart is organized


by the Ministry of Tourism, Government of
India, in association with
a) North Eastern States
b) Northeastern States and West Bengal
c) Member-countries of the South Asian
Association for Regional Cooperation
d) Representatives from the 5 statezones of India
Ans: b.
The 4th International Tourism Mart 2015 spread
over three days (15th-17th October, 2015) is seeing
participation of fifty two (52) international
delegates comprising of tour operators and media

Current Affairs 135 October-2015

persons from 23 countries.


The Ministry of Tourism, Government of India, in
association with the North Eastern States and West
Bengal has been successfully organising the
International Tourism Mart over the last three years
in various North Eastern States with the objective
of highlighting the tourism potential of the region
in the domestic and international markets.
http://pibphoto.nic.in/documents/rlink/2015/oct/p2
015101501.pdf

4. Question

Consider the following pairs:


Stage
Reactors
1. First:
Pressurised Light Water
Reactors
2. Second: Fast Breeder Reactors
3. Third: Thorium-based Reactors
Which of the above pairs, with respect to Indias
3-stage nuclear power programme, are correctly
matched?
a) 1 and 2 Only
b) 1 and 3 Only
c) 2 and 3 Only
d) 3 and 4 Only
Ans: c.
Referring to various important ongoing
programmes, Dr. Jitendra Singh mentioned the 3stage nuclear power programme with Pressurized
Heavy Water Reactors (PHWRs) in the first stage,
Fast Breeder Reactors in the second stage and
Thorium based reactors to be put up in the third
stage. He stressed that although the programme is
indigenous, India has also entered into agreements
with several countries like France, Russia and USA
to establish a large generation capacity.
http://pib.nic.in/newsite/PrintRelease.aspx?
relid=128781

5. Question

Which of the following diseases is/are caused


by parasites?
1. Elephantiasis
2. Malaria
3. River Blindness
4. Visceral Leishmaniasis
Select the correct answer using the codes given

below:
a) 1 Only
b) 1 and 2 only
c) 1, 2 and 3 only
d) 1, 2,3 and 4
Ans: d.
http://www.businessstandard.com/article/opinion/devangshu-datta-thediscoveries-that-saved-lives-115101501415_1.html
River blindness is a parasitic infection that can
cause intense itching, skin discoloration, rashes,
and eye disease that often leads to
permanent blindness. It is spread by the bites of
infected black flies that breed in rapidly
flowing rivers.
Elephantiasis refers to a parasitic infection that
causes extreme swelling in the arms and legs.
The disease is caused by the filarial worm, which is
transmitted from human to human via the female
mosquito when it takes a blood meal.
Kala Azar is globally known as Visceral
Leishmaniasis, caused by protozoan parasites of
the Leishmania genus infection whose carrier is
the sand fly found in the eastern UP, Bihar,
Jharkhand and West Bengal, usually strikes
during the monsoon months. It is the second
deadliest vector borne disease after malaria.
Presently, its incidence is concentrated to about 54
districts, with Bihar most affected. Ninety percent
of visceral leishmaniasis cases occur in 5 countries:
Bangladesh, India, Nepal, Sudan and Brazil.

6. Question

The Indian Constitution deals with the


citizenship from
a) Articles 5 to 11
b) Article 5 only
c) Article 11 only
d) Article 13 only
Ans: a.
CH-6, Indian Polity by M Laxmikant
http://www.thehindu.com/news/national/geetagovt-may-invoke-section-13-to-grant-citizenshipto-geeta/article7771059.ece

Current Affairs 136 October-2015

Você também pode gostar